You are on page 1of 96

Seminar on Valuation of Properties - 2011 1

EDUCATIVE FIELD EXPERIENCES AND JUDICIAL DECISIONS


SUPPORTING PRINCIPLES OF VALUATION
R.K. Gandhi

1.00 Availability of evidence is proof of existence. he was a trespasser occupant on the property.
But non availability of evidence does not Legally the entire property belonged to the
necessarily mean proof of non-existence. lessor and not to the lessee.

1.10 We learn many valuation principles from theory 2.10 This field experience opened his eyes. He now
books. Some we understand but some we do understood the difference between Value of
not understand in its proper perspective or with land and buildings and Value of rights of a
its overall view. Only when we find evidence person in land and building. He also realised
through our field experience that we say that that his experience supports the fundamental
such a valuation principle exists and valid principle of valuation that If there are no rights
though lack of proof does not mean that in the property, there is no value.
principle does not exist. On finding proof we
thoroughly understand the principles as our 3.00 A valuer was called upon to value 70 years
experience supports or endorses the old ancestral bungalow at Versowa Gaothan
concerned valuation principles or valuation for assessing value of rights of 3 brothers. Each
terminology. co-owner held 1/3 undivided share in the
property. Only one co-owner occupied the
2.00 A young enterprising valuer was called to value house and other two co-owners were residing
a factory complex at Dharavi for Bank loan elsewhere. Sitting co-owner had to pay to
purpose. Valuer surveyed the property. He other two co-owners for their rights in the
found plot area at 11,450 Sq.Yds. and built property.
up area of factory and godowns worked out
3.10 Considering 1500 Sq.ft. builtup area of 2
to 54,000 Sq.ft. He quickly worked out the
storeyed bungalow and considering 200
figures and informed the owner about the likely
Sq.yds. plot area, valuer valued the property
value. Then he casually asked the owner for
at Rs.9,00,000/- on land and building method.
title deeds of the property. On the following
day on perusal of documents in his office, the Sitting co-owner said that the value is too high.
valuer was shocked to find that property which Other two brothers also agreed that value was
he considered worth millions was in reality high. Valuer replied that he has considered fair
worthless i.e. of Nil value. The title document value of the property in open market. He also
showed that it was leasehold property and clarified that he has assumed vacant
lease term had expired 3 years back. There possession of the bungalow because in case
was no renewal clause in the agreement and of sale in open market vacant possession is
land and building were to vest back to the required to be given (For owner occupied
Lessor free of cost. On date of valuation the portion) to the purchaser as per provisions of
Lessee owner possessed no legal rights and Transfer of Property Act.

R.K. Gandhi, Practising Valuer, Mumbai.


2 Seminar on Valuation of Properties - 2011

3.20 The three co-owners requested the valuer to 125 ITR 665, the court further stated We
rework out the value with the assumption that held that it would not be correct to say that
the premises was not available vacant but was the rental value method has no application at
available with continued occupation by sitting all in determining the valuation of the self
co-owner. Valuer reworked out the value and occupied residential house.
arrived at value of the property at Rs.450,000/
- by applying Rental Method of valuation. There is an old English Court judgement also
Rental value of owner occupied premises was not on valuation methods but on accounting
estimated and Reversionary Value of land was methods for determining profits. Courts view
also considered. Instead of open market, is interesting. In case of Patrick (IOT) V/s.
closed market was considered. This revised Broad Stone Mills Ltd. (1954) (1 ALLELR
value was acceptable to all the three co- 163) the Court held There may be one or two
owners. or three methods of arriving at the profits for
the relevant period. The one which shows most
3.30 This field experience proves the great accurately the position between the revenue,
importance of assumptions made by the valuer on the one hand, and the tax payer, on the
while estimating value of the property. If other, is the one which ought to be adopted.
assumption changes the value would also In other words most appropriate method has
change. It also supports the basic principle of to be applied.
valuation that various methods of valuation are
tools of the valuer. If one method fails, other All the above judgements clearly indicate that
method should be tried by the valuer. There Valuer can use any valuation method that the
are no water tight compartments that owner fact and the circumstances of the given case
occupied property must be valued by land and demands.
building method only and that rental premises
should be valued by rental method only. Most 4.00 A renowned Real Estate Co. of Delhi owned
appropriate and suitable method of valuation 400 acres of land at Gurgaon. Purchase cost
has to be applied by the valuer after was less than Rs.8,000 Crores. In March 2008
considering facts and circumstances of each a valuer was appointed to revalue this 400
case. Acres of undeveloped land. Valuer made
market inquiry. He found that there was not
3.40 There are Court judgements also endorsing this much demand for plots or for flats in the said
view. In the wealth tax case of V.C. locality. He also learned that rate of developed
Ramchandran V/s. C.W.T. Karnataka, land in Gurgaon was Rs.32 Crore/Acre in
(1980)126 I.T.R. 157, Karnataka High Court 2008. The company expected valuation of
held If there are more than one valuation of Rs.12,800 Crores based on Discounted cash
same property, the one which is reasonable flow method but valuer valued whole property
and nearer to correct market value, having due at Rs.9,000 Crore only. Giving reasons valuer
regard to all the relevant facts and stated that looking to poor demand, 400 Acres
circumstances of the case alone should be of land even if fully developed, will take more
accepted. Referring to courts earlier ruling than 20 years period for sale. Hence only 40
in case of S. Neelaveni V/s C.I.T. (1980) Acres of land was valued as developed land
Seminar on Valuation of Properties - 2011 3

by DCF Method, remaining 360 Acres land lacs only. Giving reasons valuer stated in the
was valued as undeveloped agricultural land report that out of total 16,871 Sq.ft. built up
as per prevalent rate for undeveloped land. area, an area of 6479 Sq.ft. was unauthorisedly
built up in the property .Actual construction
This approach also supports the fact that on the plot had exceeded permissible limit of
various methods of valuation are tools of the 0.5 F.S.I. This was the reason for great
valuer and combination of methods can also difference between actual cost amount and
be applied in a given case. reported market value of newly constructed
bungalow. This field example high lights another
5.00 A reputed builder of Bangalore sold 60% of very important valuation concept that COST
total flats in a new building at the rate of and VALUE of the property are two different
Rs.3800/Sq.ft. in July 2007. Remaining 40% features of the property. Even for a newly
flats were retained for sale on completion of completed house its Value may be less, more
the building. In the meantime a Township or equal to the Cost.
project was declared in nearby area by another
reputed builder. Booking rate for flat in 7.00 A valuer was asked to prepare a Valuation
township was fixed at Rs.2800/Sq.ft. in Report of a leasehold property in a damage
January 2008. Builder was therefore forced claim suit for 120 Crores, which was before
to sell remaining 40% flats at Rs.2900/Sq.ft. Arbitration Tribunal, headed by 3 Supreme
in January 2008 instead of expected rate of Court judges. Claimant was Lessee and
Rs.4,000/Sq.ft. This case shows, how Respondant was Developer who contracted
important it is to consider this very strong to develop the property in 2006 for Rs.14.75
market force viz. Demand and Supply, while Crore. It was a leasehold plot of 2087 Sq.Mts.
undertaking valuation assignment of the Real area in prime industrial locality of Sakinaka,
Property. Even anticipated increased supply Andheri. Lease was for 99 years period from
of flats in the near future will bring down the 1958 with lease rent of Rs.150/month. Lease
price of present ready stock. was renewable for further 99 years period at
same old rent. Right to Sublease was also given
5.10 The above field example proves one more to Lessee. Valuer valued Lessees rights in the
fundamental valuation principle which says that property, for the year 2008 at Rs.50 Crore.
Value of the property is inversely proportional Lessors interest was valued at Rs.26,000/-
to supply. Higher the supply lower is the value and was deducted from total value.
and lower the supply higher will be the value.
This supply aspect includes present supply as 7.10 In the cross examination the valuer was shown
well as assured supply in near future. two Supreme Court judgements for his opinion
on the issue of Value of Lessors interest in the
6.00 An industrialist constructed Dream Resort property.
House at Khandala in 1998/99 for total actual
cost of Rs.430 lacs. On completion of the (i) Inder Parshad V/s. Union of India (1994) 5
project in September 1999, the property was S.C.C. 239 . In this compensation case
offered for private mortgage. Mortgagees Supreme Court held that value of Lessees
valuer estimated its fair market value at Rs.266 share in property should be 75% and 25% of
total compensation should be paid to Lessor.
4 Seminar on Valuation of Properties - 2011

(ii) Union of India V/s. Ajitsingh (1997) 6 S.C.C. of land to Lessor . All these alternatives have
50 . In this case Supreme Court held that value to be properly examined.
of tenants (Lessee) share should be 60% and
40% of total value should be paid to Lessors . Unless and until all these factors are
Value of Share of Lessor as estimated by the considered in valuation , rights of rival parties
valuer in the arbitral case was only 0.005 % in the property can not be ascertained
vis--vis 25 % and 40 % awarded by the precisely. Any shares decided without
Supreme Court. considering all these relevant aspects, in favour
of Lessor or Lessee is bound to be arbitrary
7.20 With due respect to the Court it is submitted and without any basis.
that both these judgements are arbitrary and
adhoc. The Supreme Court has not considered The fault of such arbitrary awards by the
in both these cases several relevant factors and Court does not lie on the Judiciary but it fully
provisions which decides the value of rights of lies on valuers representing the rival parties. It
Lessor and Lessee in a given case. S.C. was the duty of valuers, who are expert in
judgement ought to have been based on valuation , to point out to the Court, various
following points. relevant points and deciding factors required
to be considered by the Court , while fixing
(i) What is the lease rent for land ? value of the Lessors right and the value of the
Lessees right in the leasehold property.
(ii) What is total lease period and what is
unexpired period of lease ? 7.30 Depending upon lease terms and provisions
which are favourable to Lessee or to Lessors,
(iii) Whether lease is renewable for further period the value of share of Lessee/Lessor in the
or not ? property could be 20:80, 50:50, 90:10 or even
99 : 1%. In the case cited above value of share
(iv) In case of renewal of lease, lease rent will of Lessor in property was as low as 0.005%
remain same as old or new rent will be fixed of the total value, rest of the value belonged to
based on market rental ? the Lessee.

(v) What amount will be payable to Lessor in case 8.00 There are several Court judgements which are
of sale/assignment of property ? very very educative to the valuers. They not
only lay down fine principles of valuation but
(vi) On maturity of lease land will vest back to they also critically analyse and clarify proper
Lessor but what will happen to building methodology and concepts of valuation. These
constructed by the lessee on the leased plot ? judgements are pretty old and they pertain only
Will building vest free of cost with the Lessor to basics of valuation but merely its reading
or will the Lessor has to pay its value to the makes our fundamentals very clear. Some of
Lessee? these judgements are given below.

Generally covenant provides that lessee will 8.01 Gold Coast Selection Trust Ltd. V/s.
have to demolish the building before surrender Humphrey. (1948) 2 All-ER 379 and (1949)
Seminar on Valuation of Properties - 2011 5

17 I.T.R. 19. In this case, justice Viscount judgement, the Court has held They are
Simon has stated that Valuation is not an exact directed to the sale being in such manner as
science. Mathematical certainty is not would obtain the best possible price in the
demanded, nor indeed it is possible. It is for market. It does not mean that the price to be
commissioners to express in terms of the fixed by valuation is the highest price that might
money value attributed by them to the asset, be obtained. It has been established time and
their estimate, and this is a conclusion of fact again in these Courts, that there is a range of
to be drawn from the evidence before them. price, in some circumstances wide, which
In such a nice and clear words the Court has competent valuers would recognise , while
stated that it is an Art and not exact science. estimating the price which the property would
In common mens language we can explain that fetch if sold in the open market.
in valuation, 5 + 5 = 10 or 5+ 5 = 9 or even 5
+ 5 = 11 also. However 5+ 5 cannot be 20 in 8.04 Ahmed G.H.Ariff & others V/s C.W.T.
valuation. Following judgement supports this Calcutta. (1970) 76 ITR 471 S.C. In this
view. case for Wealth Tax, Supreme Court held
Property is a term of the widest import and,
8.02 K.P. Varghese V/s. I.T.O. Ernakulam (1981) subject to any limitation which the context may
131 I.T.R. 597 (S.C.). require, it signifies every possible interest which
a person can clearly hold or enjoy. Court
In this case Supreme Court held It is well was of the view that right to receive an aliquot
known fact borne out by practical experience share of the net income from WAKF property
that the determination of fair market value of a did constitute an asset even if it was not
capital asset is generally a matter of estimate transferable and could not be sold in the open
based , to some extent, on guess work and market. Court also held It has been rightly
despite the utmost bonafides, the estimate of observed by the High Court that when the
the Fair Market Value is bound to vary from statute uses the words if sold in the open
individual to individual. market, it does not contemplate actual state
of the market, but only enjoins that it should
The Court further held that The postulate be assumed that there is an open market and
underlying Sec. 52(2) is that the difference the property can be sold in such a market, and
between one honest valuation and another may on that basis, the value has to be found out. It
range up to 15%. is hypothetical market which is contemplated
and the tax officer must assume that there is
Thus Court has endorsed the view that in an open market in which the asset can be
valuation 5 + 5 could be 10 or even 11. Both sold.
valuers could be correct so far as estimate of
value of an immovable property is concerned, 8.05 C.W.T. V/s. P.N. Sikand (1977) 107 I.T.R.
provided of course that variation in value is 922 S.C. In this case Supreme Court held that
not more than 15%. In determining the value of the leasehold
interest of the assessee (Lessee) in the land ,
8.03 Hayes Will Trust V/s. Ruth Hayes and others. the price which the leasehold interest would
(1971) 1 WLR 758. In this English Court fetch in the open market were it not
6 Seminar on Valuation of Properties - 2011

encumbered, would have to be reduced by 8.07 C.I.T. (West Bengal) V/s.Smt. Ashima Sinha
50% of the unearned increase in the value of (1979) 116 I.T.R. 26. In this case Calcutta
the land on the basis of the hypothetical sale High Court approved of the I.T. Tribunals
on the valuation date. In other words view that The said property having been sold
unearned increase payable to the Lessor as in two undivided half shares, a further
per lease deed should be deducted from the deduction of 10 % in value would have to be
total valuation. made

8.06 C.G.T. Bombay V/s Kusumben Mahadevia. In case of J.N. Bose V/s C.W.T. (1976) 104
(1979) 122 ITR 38. ITR 83 also Calcutta High Court approved of
10 % discount in value to account for
This case is not concerning valuation of undivided share of the co-owner in the
Building but it is a very interesting case on property. It was held by the Court that The
valuation of shares. Gift Tax officer worked tribunal has also not adverted to the fact that
out the value of the shares (Not quoted on the assessee was the owner of undivided half
exchange) by averaging the value arrived at share of the property. This is also relevant
by two different methods (Going concern profit factor affecting the valuation of the property
method and breakup value on liquidation in respect of market value.
method).
Both these judgement confirm that value of an
In this case Supreme Court held that The undivided share of the co-owner in a jointly
mere averaging of two results obtained by quite owned property will fetch atleast 10 % less
different basis of approach can hardly be said value in the open market.
to represent any logical approach, whatever
its merit as a compromise. Despite its evident 8.08 C.W.T. A.P. III V/s. Amatul Kareem (1980)
popularity in any quarter it has not been given 127 I.T.R. 549. In this Wealth Tax case,
judicial recognition in decisions involving the Andhra Pradesh H.C. allowed 50% rebate in
fixation of a value by the Court. The value due to pending litigation and risk and
combination of two methods advocated by the uncertainty of receiving compensation claim.
revenue has no sanction of any authority and High Court held The tribunal rightly took into
cannot be accepted as valid principle of consideration the hazard and risk involved in
valuation of shares. the litigation and the prospects of the amount
not being received in the near future. These
This judgement, though very old and regarding are certainly relevant factors for determining
valuation of shares, is applicable even today the value of the asset. When the tribunal has,
with equal force to the valuation of real estate in view of these relevant factors, assessed the
also. There are few Valuer friends who are value of the wealth at 50 % we cannot say
fond of working out the value of the property that it has adopted any wrong principle of law.
by two methods, viz. Land and building method
and rental method and then report the average Pandit Laxmikant Jha V/s. C.W.T. (1973) 90
value to the client. The knowledge of Supreme I.T.R. 97 S.C. In this similar case Supreme
courts view in this case may perhaps stop Court had allowed 35% rebate in value of the
them from adopting this novel approach. property due to pending litigation.
Seminar on Valuation of Properties - 2011 7

In both these cases, asset brought to Wealth 8.12 C.I.T. V/s C.F. Thomas . decided on 6-6-
Tax, was amount received by the owner as 2006 (I.V. Feb 2007 p- 165 )
enhanced compensation in High Court This appeal for Capital Gain Tax was filed
appeal, for the land acquired by the before Kerala High Court by I.T.department
Government.
In this case an interesting issue of Pakidi
8.09 U.S. Nayak V/s. C.W.T. (68 I.T.R. 171) . In (Also known as Pagdi-Salami-Key money-
this case owner purchased the property but Premium) was discussed by the court. Mr.
title of the property was defective. There was Thomas rented 1350 Sq.Ft. shop in
pending litigation and objecting party was in Ernakulam to Mr.Sundaresa Pai for rent of
possession. Mysore H.C. approved of the Rs. 5000/month for 20 years lease period. Rs.
view of the Tribunal that The presence of the 2 Lacs were taken as deposit. Tenant Shri Pai
objecting party and difficulty to get vacant filed sworn in statement in I.T. office that he
possesion are relevant considerations to be has paid Rs.10 Lacs Pakidi (Unaccounted
taken into account for deciding the price of money) to the Landlord (Lessor) to acquire
the house property and these would have a lease rights in shop.
depressing effect. 33% reduction in value for
defective land title was considered fair while Kerala H.C. held that Any transaction by way
fixing Fair market value of the property. of any agreement which has the effect of
transferring or enabling the enjoyment of any
8.10 Jawajee Naganathan V/s. Revenue Div. Officer immovable property is a transfer within the
Adilabad. (1994) 4 S.C.C. 595. In this case meaning of I.T.Act. The instrument in question
Supreme Court held that It is clear that the is a transfer of a capital asset and therefore
Basic Valuation Register prepared and any Profit or Gain arising out of such transfer
maintained for the purpose of collecting stamp will generate Capital Gains.
duty has no statutory base or force. It cannot
form a foundation to determine market value The court also held that The proviso to section
mentioned there under in instrument brought 92 of the Indian Evidence Act 1872, indicates
for registration. The Court also held that that the law does not foreclose the existence
The basic valuation register cannot form any of contemporaneous oral agreements regarding
basis to determine the market value of the such matters on which the written instrument
acquired land. is silent. The adjudicatory authorities cannot
ignore the ground realities and the common
8.11 Mani Singh Avtar singh V/s I.A.C. (1984) 151
course of Human Conduct and also the
ITR 233. In this case Punjab and Haryana
Mercantile Malpractices. In other words the
H.C. has held that The price of a plot built
court was of the view that Rs. 10 Lacs Pakidi
upon can not compare favourably with that of
paid and confirmed by the tenant was a price
an unbuilt plot on which the purchaser has the
paid for an asset of the owner and the said
opportunity of exercising a wide choice of
amount could be brought in as Capital Gains
building. But in the former case, he is not a
of the Landlord.
chooser. Applying that principle, the prices of
Apart from the above stated judgements,
the plot could even ratably have been reduced
which are supporting basic valuation principles,
by a reasonable percentage
8 Seminar on Valuation of Properties - 2011

there are several other judgements also on the client suffers because his property will not sell
subject of valuation. Some of these pertain to in the market in the open public bid. How many
Land Acquisition cases, some pertain to Wealth valuers are confident today that the property
Tax Act and some others pertain to Capital they value will sell in the market at the exact
Gain Tax. Valuers can refer to those price or at the price which is very close to their
judgements as and when occasion arises. reported value.

9.0 It may interest Bankers who are connected 9.10 All these goes to show that Subject of
with Debt Recovery Tribunal or with Auction Valuation is as big as an ocean. The valuer
of Non-performing assets of Banks, how should continuously update his knowledge by
properties were sold in India 2300 year ago. group discussions, by attending seminars and
The great author of Arthashashtra , Shri. by study of valuation books and journals. This
Kautilya , in his Volume 2 Chapter 21 continued education in valuation is the only
describes the procedure for sale of property way how the valuer can become an Expert
by a citizen. He writes: Owner of property Valuer.
shall call out the sale price three time in public.
If property is sold at higher price than the call 9.20 It is said by one prudent man that Thought
price, the difference will be paid to Govt. and opinion of Experts are like watches. Every
Treasury. This norm shows how precise the ones watch shows different time but everyone
valuer should be in valuation. If the valuer believes that only his watch shows the correct
undervalues the property his client suffers time.
because surplus will go for government tax. If
valuer overvalues the property then also his
Seminar on Valuation of Properties - 2011 9

GRADEA INFORMATION TECHNOLOGY PARK AT CHENNAI


PROJECT COST INFORMATION FOR MEP (SHELL & CORE)
INSTALLATIONS
B. Senthil Kumar
Grade-A IT Parks are the one which are built to meet international standards (in full compliance with Local
Authority requirements). Complete MEP infrastructure shall be made available upfront by the developer, so
that the tenants can move in with their own interior works having completed by them.

100% power back up, provision for server room, air conditioning, fire protection and prevention to keeping
up international standards, complete intelligent building management system and adequate vertical transportation
system are some of the features of a Grade-A IT Park.

The following table illustrates the estimated square foot based cost for MEP installation of a typical Grade-A
IT Park of 7 lakhs square feet.

Sl. No. MEP System Cost / Square Foot (INR)

1 Electrical installation 198

2 Diesel Generators 137

3 Plumbing, Drainage and Sanitary Installation 45

4 Fire Fighting System 47

5 Air Conditioning & Mechanical Ventilation System 160

6 Vertical Transportation (Lifts) 98

7 Air-cooled Chillers 87

8 Intelligent Building Management System 63

* The above cost estimate is subject to change due to market fluctuations, site conditions, Local Authority/
Client requirements, etc.

Director, M/s. Enkon Engineering Consultants Pvt. Ltd., Chennai


10 Seminar on Valuation of Properties - 2011

MORTGAGE VALUATION TO SATISFY SECTION 5(N) OF BANKING


REGULATION ACT, 1949
AND
DISTINCTION BETWEEN - MARKET VALUE, 'JANTRY' / CIRCLE
VALUE / GUIDE LINE VALUE, LIQUIDATION VALUE ORDERLY
LIQUIDATION/ FORCED SALE VALUE
Kirit P. Budhbhatti
1.0 Introduction seller in an arm's length transaction after proper
marketing wherein the parties had each acted
Loans from banks and other financial knowledgeably, prudently and without
institutions are often secured by the collateral compulsion.
of the borrower's property interests. The
lending may be by way of a mortgage or other The estimated amount means a price, normally
forms of fixed or floating charge. The common expressed in terms of money, payable for the asset
factor is that the lender has the power to in an arm's length market transaction. Market
recover the loan by taking control of the value is measured as the best or most probable
collateral in the event of default by the price reasonably obtainable in the market at the
borrower. date of valuation in keeping with the Market
Value definition. The estimate specifically
1.1 Valuation u/s.5 (n) of Banking Regulation excludes an estimated price inflated or deflated
Act, 1949 by special terms or circumstances such as typical
financing, sale and leaseback arrangements,
Secured loan or advance has been precisely special considerations or concessions granted by
defined in clause (n) of Section 5 of Banking anyone associated with the sale, or any element
Regulation Act, 1949, as: of special value.

a loan or advance made on the security of an asset ought to exchange means the fact
the assets the market value of which is not at the value of an asset is an estimated amount rather
any time less than the amount of such loan or than a predetermined or actual sale price. It is
advance. the price at which the market expects a
transaction should be completed on the date of
2.0 Definition and explanation of Market valuation, meeting all other elements of the
Value as per IVS Market Value definition.

Market value is the estimated amount for which on the date of valuation indicates that the
an asset ought to exchange on the date of estimated Market Value is time specific as of a
valuation between a willing buyer and a willing given date. Because markets and market

Kirit P. Budhbhatti, Practising Valuer RE and PM - Mumbai, Founder Chairman (Retd.) International Plant
and Machinery Valuation Conference Committee, Founder Secretary, Centre For Valuation Studies, Research
and Training
Seminar on Valuation of Properties - 2011 11

conditions may change, the estimated value may independently.


be incorrect or inappropriate at another time. The
valuation amount will reflect the actual market after proper marketing means that the
state and circumstances as on the valuation date, asset would be marketed in the most appropriate
not as of either a past or future date. manner to effect its sale at the best price
reasonable in accordance with the Market Value
a willing buyer means one who is definition. The length of marketing time may vary
motivated, but not compelled to buy. This buyer with market conditions, but must be sufficient to
is neither overeager nor determined to buy at any allow the asset to be brought to the attention of
price. This buyer is also one who purchases in an adequate number of potential purchasers. The
accordance with the realities of the current marketing period occurs prior to the valuation
market, and with current market expectations, date.
rather than an imaginary or hypothetical market
that cannot be demonstrated to exist or to be wherein the parties had each acted
anticipated by the market. The assumed buyer knowledgeably and prudently presumes that
would not pay a higher price than the market both the buyer and the seller are well informed
requires. The present asset owner is included about the nature and characteristics of the asset,
among those who constitute the market. A its actual and potential uses and the state of the
valuer must not make unrealistic assumptions market as on the date of valuation. Each is further
about market conditions not assume a level of presumed to act for self-interest with that
market value above that obtainable. knowledge, and prudently as to seek the best
price for their respective positions in the
a willing seller means one who is neither transaction. Prudence is assessed by referring to
an overeager nor a forced seller prepared to sell the state of the market at the date of valuation
at any price, nor one prepared to hold out for a not with benefit of hindsight at some later date.
price not considered reasonable in the current
market. The seller is motivated to sell the asset at and without compulsion establishes that
market terms for the best price attainable in the each party is motivated to undertake the
open market, and after proper marketing, transaction, but neither is under compulsion
whatever that price may be. The factual unduly coerced to complete it.
circumstances of the actual asset owner are not a
part of this consideration because the willing Having studied the provisions regarding secured
seller is a hypothetical owner. loan or advance and definition of market value,
now let us look at the international scenario in this
in an arm's length transaction means one regard.
between parties who do not have a particular or In order to ascertain the practice in different parts
special relationship (for example parent and of globe, I approached valuers across the globe. The
subsidiary companies, or landlord and tenant) that valuers from New Zealand, UK and Australia
may make the price level atypical of the market. responded to query and their replies are given
The Market Value transaction is presumed to be below:
between unrelated parties, each acting
12 Seminar on Valuation of Properties - 2011

Querry : Value regardless of the purpose of the


valuation. There have been a few recent NZ
Sub: Valuation of real estate as well as plant and cases that have supported approach the most
machinery offered as security against loan in well known is the Bpatpark case. This means
your country. that any property has only one market value
and any other value which is different is hence
Please enlighten me on following issues:
not the market value.
Y Whether Market Value as defined under
International Valuation Standards IVS-1 is alone For lending security purpose there is then a
required to be calculated by valuers or realisable security margin computed for that property
price, restricted realisable price as well as which is applied to the market value. The
Forced Sale Value are also required to be security margin takes into account the factors
calculated? such as sale under duress, the accumulation of
arrears of interest, holding and selling cost,
Y If all the above values are worked out then what fluctuations in the value of the security over
are the bases of each one? How they are worked the life of the mortgage, etc. In many instances
out after first computing market value as per IVS. the lending institutions only request the market
valuation and they decide on the appropriate
Y In the event of default by the borrower in order
security margin, in others they request a
to safeguard the interest of lending institutions
security margin and a recommendation as to
lending institutions expect that security offered
the prudent amount to lend .
should be capable of fetching outstanding
principal amount with interest. The borrower is
We do have an NZIV practice standard on
likely to go in default few years after getting loan.
valuations for mortgage security purposes.
What is line of action adopted by valuers to
safeguard the interest of financial institutions as
2) U.K.
well as his professional insurance is not
adversely affected.
Normally the valuation for loan security is done
Y If forced sale value is required to be worked as a market value. Sometimes lenders will ask
out then do valuers get sale instances of property the valuer to make some assumptions that are
sold under duress? Do valuers examine parties a bit conservative and this may create a
to transaction to ascertain whether there was no reduced market value. One of the assumptions
compulsion on either sides buyers or sellers? might be a restricted marketing period. This is
what we think is meant by a forced sale but in
Y Any other basis of valuation adopted for security the UK we do not like terminology of forced
against loan in your country. sale.

REPLIES TO QUARRY Lenders are increasingly asking for additional


information to undertake future risk
1) New Zealand
assessments, For example, with investment
We use the definition you quoted in your first property, they would like to know the expiry
paragraph. We always begin with the Market date of the leases and may tailor the loan
Seminar on Valuation of Properties - 2011 13

period so that it expires before lease expiry. a moving target. Is a forced sale one that is
But remember that in the UK we have very expected to take place within 3 months, 2
long leases and lenders tend to take much more months, 1 month, 1 week, 1 day?. Auction
notice of lease contracts and tenant covenant Value reflects the needs of the lender because
strength as well as the property fundamentals. by definition it is a value that can be realized in
Where there are generally short leases, they the relatively short time span.
only look at the property market fundamentals.

In Germany, there is an alternative Mortgage Illustration for Market Value


Lending Value but I personally think it is a
hopeless attempt to given lenders a lower A residential flat is offered for a sale. The owner of
valuation base and has no theoretical validity the flat collects the information from the market about
or practical consistency. But a lot of Europeans the market condition and as per the market he
like it and I have upset quite a few people with decides to sell the flat at an 'X' amount. Another
my criticisms of it. It is included in the European person who is looking for a residential flat comes
Valuation Standards which is called the Blue across this offer and he also collects the information
Book. from the market about the prevailing market trends
and market condition and after proper market inquiry
3) Australia establishes an opinion that the amount 'X' for the flat
is reasonable and goes ahead with the transaction
Your definition of Market Value is the one than such an amount 'X' is considered as Market
adopted in Australia. For your info the IVSC Value of that flat wherein sufficient exposure time
adopted an Australian definition!. Most lending was given to the asset (here flat) for transaction in
bodies in Australia called either for Market an open market.
Value ( as defined above) or Auction Value.
Again, it is important to note that,
Auction Value is the estimated amount that
one would expect to achieve at a properly Transaction Costs
promoted, conducted and attended auction
sale held on the site and at which substantially Market value is the value of an asset without regard
all of the assets in the inventory listing to the seller's costs of sale or the buyer's costs of
(valuation) are offered for sale at one time purchase and without adjustment for any taxes
IVS GN3 (For info the IVSC adopted my payable by either party as a direct result of the
definition!). Both Market Value and its sub- transaction.
set Auction Value must be market related if
possible. If comparables are not available the Sometimes, the Market Value and the Fair Value can
theoretical methods must attempt to replicate be used as the synonymous to each other but, it is
the market. not so. The Proposed New International Valuation
Standards defines the Fair Value and Special Value
In my view it is not possible to define Forced as under:
Sale Value because one is attempting to define
14 Seminar on Valuation of Properties - 2011

Special Value Y Selling Price - Estimated Cost of Disposal =


Net Realizable Value
Special value is an amount that reflects particular
attributes of an asset that are only of value to a special 4.0 Liquidation value
purchaser.
The definition and explanation of Liquidation
A special purchaser is a particular buyer, or a Value as per IVS is as under.
restricted class of buyers, for whom a particular asset
has special value because of advantages arising from The value of assets estimated with regard to
its ownership that would not be available to general specific circumstances under which the assets
purchasers in the market. are sold. Liquidation value describes a situation
where a group of assets employed together in a
Special value can arise where an asset has attributes business are offered for sale separately, usually
that make it more attractive to a particular buyer than following a closure of the business. Although
to the general body of buyers in a market. These associated with forced sale, these terms have
attributes can include the physical, geographic, distinct meanings. There is no reason why assets
economic or legal characteristics of an asset. Market cannot be liquidated by an orderly sale following
value requires the disregard of any element of special proper marketing.
value because at any given date it is only assumed
that there is a willing buyer, not a particular willing The value of property derived from the
buyer. When special value is reported, it should be transactions in which sellers are not willing
clearly distinguished from market value. sellers, and marketing time is not sufficient, is a
liquidation value. Buyer may be motivated by
3.0 Realizable Value the knowledge of the disadvantages the seller
suffers from.
The definition and explanation of Net Realizable
Value as per IVS is as under. Liquidation value is further classified as under
depending on urgency of sale:
Y The estimated selling prices of an asset in the
ordinary course of business, less the estimated (a) Orderly liquidation value
costs of completion and the estimated costs
necessary to make the sale. (b) Forced sale value

Y Net realizable value refers to the net amount Forced Sale Value / Distress Value:
that an entity expects to realise from the sale of
inventory in the ordinary course of business. The definition and explanation of Forced Sale as per
Fair value reflects the amount for which the same IVS is as under.
inventory could be exchanged between
knowledgeable and willing buyers and sellers A circumstance where a seller is under compulsion
in the market place. to sell and/or a proper marketing period is not
available. The price obtainable under these
In a simple word, it is circumstances will not meet the definition of Market
Seminar on Valuation of Properties - 2011 15

Value. Rather the price obtainable will depend on the nature of the pressure on the seller or the reasons why
proper marketing cannot be under taken. The price may also reflect the consequences for the seller of failing
to sell within a specified period. The price obtainable in a forced sale typically cannot be predicted, but will
reflect the particular circumstances of the forced sale rather than a hypothetical exchange where the seller is
acting without compulsion and/or the transaction occurs after a proper marketing period.

The difference in above two values is on account of urgency under which property is sold.

It is not possible to define 'forced sell value' in precise terms on account of the following reasons -

Y Is a forced sale one that is expected to take place within 6 months, 1 month or a day?

Y It is difficult to forecast values after five years or so as Indian economy is not that stable. Even expert
economist cannot forecast!

Y Scarcity of comparable sale instances of property sold under duress.

Y Urgency to sale varies from person to person.

Y What could be the bases of its estimation?

Bankers normally ask valuer to report two values Market value and Forced sale value. Bankers should
realise that value of property as on today may not remain same in future when borrower becomes defaulter.
Adequate safety margin should, therefore, be kept on reported value. When property becomes NPA, a fresh
report on valuation of security should be obtained from the valuer, before auction.

5.0 Chart showing difference between Market Value, Realizable Value & Forced Sale Value

Transaction between willing buyer and willing seller wherein both the
parties have each acted knowledgeably and prudently after proper market
inquiry and without compulsion

Market Value

Realizable Value

Seller is forced to sale the asset


The estimated selling price compulsorily with no proper
less estimated costs marketing / exposure time.
necessary to make the sale.

Forced Sale Value /


Distress Sale Value
16 Seminar on Valuation of Properties - 2011

6.0 Guide line Value/ Circle Value/ Jantri The Supreme Court of India decided in the case of
Value as per Stamp Duty Acts applicable Jawjaee Nagnathan Vs. Revenue Divisional Officer,
in Gujarat, Maharashtra, Andhra Pradesh, Adilabad, A.P. ((1994)4 SCC 595) that the Basic
Tamilnadu, Karnataka and Kerala Valuation Register maintained by the registering
authority for collection of stamp duty has no statutory
Regarding the mode of determining taxable foundation to determine market value. The provision
income for the purpose of capital gains tax, of section 47A of the Indian Stamp Act, 1899, does
there was a proposal even in the past to resort not confer power to determine market value of an
to stamp-duty guideline rates for this purpose. entire area, region or block and to maintain Basic
This proposal was however, not accepted and Valuation Register for levy of stamp duty. Neither
Mr. Yashwant Sinha (then Union Finance the Registrar nor the vendor is bound by it. Evidence
Minister) said the following in the Parliament of bona fide sales, the Court observed, between
while presenting his budget in the year 1998. willing prudent vendor and prudent vendee of the
lands acquired or situated near about the subject land
A number of representations were received possessing same or similar advantageous features
against the proposal to substitute the fair would furnish the bases to determine market value.
market value of land with the value adopted The Basic Valuation Register prepared and
by State Government authorities for the maintained for the purpose of collecting stamp duty
purpose of stamp duty for calculating has no statutory base or force and accordingly, it
capital gains. cannot form a foundation to determine the market
value of property mentioned in the instrument brought
These representations point out that as the for registration .
circle values fixed for the purpose of stamp
duty are arbitrary, a large number of bona The above observations are absolutely correct which
fide transactions shall be affected by the will be evident from cases cited herein after.
proposed amendment. The value of
properties fixed by the State Government Market Value is defined as under in Gujarat
does not always represent the fair market Stamp Manual
value. I agree with the apprehensions
expressed in this regard and hence withdraw (Extracts from forthcoming book on valuation
the proposal to calculate capital gains tax of real estate by Kirit P.Budhbhatti)
on the basis of circle rates of stamp duty.
Market Value in relation to any property which is
(Source: The Economic Times, Mumbai Edition the subject matter of an instrument of transfer means
dated 20-07-1998) the price which such property would have fetched if
sold in the open market on the date of execution
The above provision was, however, made of such instrument.
effective from 1-4-2003 in spite of having any
improvement in the method of preparation of Similar definition of Market Value is prescribed in
ready recknor in the intervening period. the Manual of Stamp Duty and Registration in Andhra
Pradesh by M.V.Durgaprasad-Third enlarged edition
Seminar on Valuation of Properties - 2011 17

Meaning of the term If sold:

If sold in the open market does not contemplate actual sale or actual state of market but only enjoins that it
should be assumed that there is an open market and property can be sold in such a market.
(Ref.: Ahmed G.H. Ariff vs. C.W.T. 76 ITR 471)

If sold, creates a fictional position which the Tax Officer must assume. It is a hypothetical case which is
contemplated by these words. Let the price be determined by economic forces sale to be open to competition.
Adequate publicity or advertisement prior to sale, are given as criteria of open market.
(Ref.: Purshottam Amarsey vs. C.W.T. 88 ITR 417)

Meaning of the term On the date of valuation:

on the date of valuation requires that the estimated Market Value is time-specific as of a given
date. Because markets and market conditions may change, the estimated value may be incorrect or inappropriate
at another time. The valuation amount will reflect the actual market state and circumstances as of the effective
valuation date, not as of either a past or future date. The definition also assumes simultaneous exchange and
completion of the contract for sale without any variation in price that might otherwise be made.

3.0 Now let us examine the principles to be taken into consideration for determination of market value (as
per Gujarat Stamp Manual,1991 and Manual of Stamp Duty & Registration in Andhra Pradesh., The
Bombay Stamp (Determination of True Market Value of Property) Rules, 1995, The Karnataka Stamp
Act, 1957, Kerala Stamp Act and Tamil Nadu Stamp Act: :

GUJARAT

Gujarat Stamp Manual, 1991 Authors comments

Principles to be taken into consideration for


determination of market value:

The Collector of the District shall while determining The manual defines market value but does not define
the true market value of a property which is the true market value.
subject matter of an instrument take into consideration
There are three approaches to value income, market
primarily the capitalized value of the property i.e. the
and cost.
amount of money whose annual interest at the highest
prevailing interest at any given time is its net annual The manual recommends income approach with other
income, and also the following factors, namely factors.
The income approach requires net rent from property
and YP to estimate value. YP is to be computed from
market-derived data from sale of similar rented
property.
18 Seminar on Valuation of Properties - 2011

Gujarat Stamp Manual, 1991 Authors comments

This is not correct on account of following reasons:


a. Most of the property transactions are of either
vacant possession (VP) or owner occupied
(OO).
b. Due to Rent Control Act sufficient data to carry
out the valuation by income approach is not
available.
c. Tenants mainly fall under two categories
- Protected under Rent Control Act
- Not Protected under Rent Control Act
d. The income from protected and not protected
tenants are different.

In view of facts mentioned above, it is not correct to


only recommend income approach.
(a) in the case of agricultural land- No comments are offered on agricultural land as book
is for immovable property other than agricultural land.
(i) Classification of land under the provision of the
Bombay Land Revenue Code, 1879;
(ii) the rate of the land revenue;
(iii) the nature of crops raised in the land;
(iv) the average yield form the land, its nearness to
road and market, its distance from village site
road to land. facilities available for irrigation and
also for transport of produce of such land;
(v) value of adjacent or land in vicinity;
(vi) any factors mentioned in the instrument which is
relevant for the purpose of determination of true
market value;
(vii) any other factors which the Collector of the
District thinks to have a bearing on the valuation
of land;
(b) in the case of non-agricultural land, -
(i) the general value of non-agricultural land in the This is vague.
vicinity;
(ii) facilities such as road, railway station, bus route, These are relevant factors.
shops, market and the like available in the vicinity
of the land;
Seminar on Valuation of Properties - 2011 19

Gujarat Stamp Manual, 1991 Authors comments

(iii) amenities like public offices, hospitals and These are relevant factors.
educational institutions available in the vicinity
of land;
(iv) development activities including, development of These are relevant factors.
industries in the vicinity of land;
(v) any factors mentioned in the instrument which is Factors under this category are neither outlined nor
relevant for the purpose of determination of true explained.
market value;
These underlined principles need to be completed as
they are to be used as guidelines in preparing a
valuation. Therefore, all the factors need to be
enumerated except a few factors, which will be
required to be taken in to consideration based on
facts and circumstance under peculiar situation.

As this has not been done, it is high time that fresh


guidelines be prepared considering all the factors
mentioned later.

(vi) any other factors which the Collector of the As above.


District thinks to have a bearing on the valuation
of non-agricultural land;

(c) in the case of buildings,-


(i) the area of construction;
(ii) the floor space index;
(iii) type and structure;
Factors mentioned under (i) to (vii) forms part of
(iv) year of construction; physical/technical factors explained in valuation
(v) kind of material used; maxims later.

(vi) locality in which constructed;


(vii) rate of depreciation;
(viii) any factors mentioned in the instrument As per (b) (v) above.
which is relevant for the purpose of
determination of true market value;
(ix) any other factors which the Collector of the As per (b) (v) above.
District thinks to have a bearing on the
valuation of building;
20 Seminar on Valuation of Properties - 2011

Gujarat Stamp Manual, 1991 Authors comments

(d) in case of any other property,-


(i) the nature and condition of the property; This is also vital for property falling under (b) and
(c) above.
(ii) purpose for which the property is being put This is also vital for property falling under (b) and
to use; (c) above.
(iii) any factors mentioned in the instrument As per (b) (v) above.
which is relevant for the purpose of
determination of true market value;
(iv) any other factors which the Collector of the As per (b) (v) above.
District thinks to have a bearing on the
valuation of property.

ANDHRA PRADESH
Manual of Stamp Duty and Registration in A. P. Authors Comments
By M. V. Durga Prasad, Ed. 2005-2006 Page
no.385

34. Calculation of market value of flats in the multi- When an ownership flat in a multi-storied building is
storied buildings and 387 of manual. purchased the transaction is for acquisition of right,
(a) Where one or more flats in the multi-storied title and interest, wherein land and building are not
building are purchased the value of the site to separately purchased and valued. Market value of
be added to the market value of the building shall building (flat) includes market value of land.
be to the extent of the rights created in favour of
the purchaser in the site together with the value Therefore, this guideline is misleading and incomplete.
of the appurtenant land, in any, on which specific
rights are created.

(b) Where the document does not specify any such From the terms and conditions of the document one
rights as above the registering officer should has to find out what is the subject matter of
ascertain by means of a deposition from the transaction and accordingly rights are to be valued.
executants / presentant the information necessary
for calculating the market value. If the information
is not ascertainable or the parties refuse to
furnish such information, clarification as
contained in the Procgs. No. MV_3/2788/85,
dated 6-12-1985 should be followed for
calculation of market value.
[I.G.s Procgs. No. MV-3/2788/87, dated
21-1-1987]
Seminar on Valuation of Properties - 2011 21

37. While arriving at the market value of the It appears that I.T.C.C. means Income Tax
property in respect of buildings and houses, the Clearance Certificate under Chapter XXC of
value mentioned in I.T.C.C. will have also to Income-Tax Act, 1961, which is withdrawn from 1st
be taken into consideration. If the value of July 2002.
property noted in the I.T.C.C. is higher than
the (i) market value assessed by the Sub-
Registrar based on the Basic Register, (ii) the
value noted in the document by the party, (iii)
18 times of annual rental value and (iv) market
value as expressed by the party under Rule 3
of M. V. Rules, it constitutes as market value
of the property and the registering public are
bound to adopt such higher value for payment
of stamp duty and registration fees. In cases
where the registering public do not pay stamp
duty and registration fee. In cases where the
registering public do not pay stamp duty and
registration fee on such higher value noted in
I.T.C.C. the Registering Officers should
impound the document. A separate column
should be opened in Annexure I-B as last
column as Value noted in I.T.C.C.

BOMBAY
The Bombay Stamp Act Determination of the Market Value of Property Authors comments
Rules, 1995

4. Annual Statement of Rates of Immovable Property


(1) The Joint Director of Town Planning and Valuation, Maharashtra State, shall
prepare annual statement of rates showing average rates of lands and buildings
situated in every Tahsil, Municipal Corporation or local body area with the
help of such other officers as may be appointed by the Government from time
to time and submit the same for approval to the Chief Controlling Revenue
Authority, latest by 31st October of each year.

(2) The data in respect of average rates of lands and buildings in every Tahsil,
Municipal Corporation or local body area shall be arranged in the annual
statement of rates as far as possible in ward wise / zone wise manner in respect
of urban properties and in respect of rural properties, Taluka-wise, village-
wise as the case may be. For the purpose of average annual rates, properties
may be divided in groups, sub-groups or classes after taking into account the
types of land, types of construction, location and situational advantages or
22 Seminar on Valuation of Properties - 2011

dis-advantages of property. While working out the average rates, of lands


and buildings, the concerned officers shall take into account the established
principals of the valuation and any other details that they may deem necessary.

(3) The Chief Controlling Revenue Authority shall by an order issue annual statement
of rates showing average rates of lands and buildings situated every Tahsil,
Municipal Corporation and local body area (hereinafter called annual statement
of rates) as soon as they are made for the first time, and thereafter, every
year on 1st day of January, taking into account the average rates of lands and
buildings prepared and submitted to him by the Joint Director of Town Planning
and Valuation, Maharashtra State.

(4) If the Chief Controlling Revenue Authority is not in a position to issue annual
statement of rates as mentioned in sub-rule (3) above, on 1st day of January in
any year due to any administrative difficulties, the rates mentioned in the annual
statement of rates for the year immediately preceding may be incremented by
the Chief Controlling Revenue Authority, in consultation with the Joint Director
of Town Planning and Valuation, keeping in view the increase in market rates
of immovable properties.

(5) The Chief Controlling Revenue Authority shall annually supply to Sub-Registrar
a copy of the above statement showing the average rates of lands and buildings
situated within his jurisdiction. Every registering officer shall cause a copy of
the above statement to be affixed outside the Registration Office.

(6) Every registering officer shall, when the instrument is produced before him for
registration, verify in each case the market value of land and buildings, etc., as
the case may be, from the above statement and if he finds the market value as
stated in the instrument, less than the minimum value, prescribed by the
statement, he shall refer the same to the Collector of the District for
determination of the true market value of the property which is the subject
matter of the instrument and the proper duty payable thereon:

*[Provided that, if a property is sold or allotted by Government or Semi-


Government body or a Government Undertaking or a Local Authority on the
basis of the pre-determined price, then value determined by said bodies, shall
be the true market value of the subject matter property] :

Provided further that, where the market value has been stated in accordance
with or more than prescribed in the statement issued by the Chief Controlling
Revenue Authority, but the Registering Officer has reason to believe that the
true valuation of the immovable property cannot be arrived at without having
Seminar on Valuation of Properties - 2011 23

recourse to the local inquiry or extraneous evidence, he may, before registering


such instrument, refer the same to the Collector of the District for determination
of true market value of property and the proper duty payable thereon.

* This proviso was substituted by G.N., R & F.D., No. MUDRANK. 1099/
2251/C.R. 676/M-1, dated 6-12-2001 (M. G. G., Pt. IVB, dt. 13-12-2001,
p. 837).

(7) All the Registering Officers shall send to the Town Planning Valuation Officers
appointed to assist the Joint Director of Town Planning and Valuation for
preparation of annual statement of rates, the extract of the register in respect
of the instruments presented for registration in which consideration for the
subject property is stated to be more than the annual statement of rates by
30th day of the following month.

(8) All the Special Land Acquisition Officers appointed under the Land Acquisition
Act, 1894, or any other Act for the time being in force in respect of acquisition
of lands and properties for public purposes shall, whenever the amount of
compensation awarded by them is higher than the one payable on the basis of
annual statement of rates issued by the Chief Controlling Revenue Authority
under sub-rule (3) and (4) above, send a copy of such award to the Town
Planning and Valuation Officers appointed to assist the Joint Director of Town
Planning and Valuation for preparation of annual statement of rates, referred
to in sub-rule (7) above, within 30 days from the date of payment of
compensation.

KARNATAKA
The Karnataka Stamp (Constitution of Central Valuation Committee for Authors comments
Estimation, Publication and Revision of Market Value Guidelines of
Properties) Rules, 2003

6. Guidelines for the estimation of the market value by the Sub-


Committee.
Each Market Valuation Sub-Committee shall prepare the statement showing
average rates of agricultural and non-agricultural lands, residential,
commercial and industrial sites in the sub-district and municipal or local
body area, on the following general guidelines as reference:
(1)(a) In the case of lands
(i) Classification of land as dry, garden, wet and the like;
(ii) Classification under various classes of soil in the survey records;
(iii) Other factors which influence the valuation of the land in question;
(iv) Value of adjacent land or lands in the vicinity;
24 Seminar on Valuation of Properties - 2011

(v) As far as practicable the nature of crop and average annual yield
from the land for five consecutive years till the determination and
nearness to road and market, distance from village site, its location in
general, level of land, transport facilities, facilities available for irrigation
such as tanks, well and pump sets.
(1)(b) In the case of house-sites
(i) The general value of house-site in the locality;
(ii) Nearness to road, railway station, bus route;
(iii) Nearness to market shops and the like;
(iv) Amenities available in the place like public offices, hospitals and educational
institutions;
(v) Development activities, industrial improvements in the vicinity;
(vi) Land tax and valuation of sites with reference to taxation records of the
local authorities concerned;
(vii) Any other features having a special bearing on the valuation of the site;
(viii) Any other special features bore-well in addition to public water supply,
lawn, garden and swimming pool.
(1)(c) Properties other than lands, house-sites and buildings -
(i) The nature and conditions of the property;
(ii) Purpose for which property is being put to use; and
(iii) Any other special features having a bearing on the valuation of the property.
(2)(a) The suggestions for the estimation of market value of non-agricultural and
industrial lands in general may also be separately indicated either as multiple
of the rate for agricultural land or square feet considering the location of
the property namely, Municipality / Corporation or village. Normally the
values for lands converted for non-agricultural use in a village not near to a
town / city, may be estimated as multiple of the rates for agricultural lands
and those near or in the vicinity of a town or city may be estimated per
square feet.
(2)(b) The value of agricultural lands may be classified as dry, wet or garden,
generally and nearness to the village may be considered for fixing the rates
to each class.
(2)(c) The rate for the lands with coconut or areca plantations may be estimated
as garden lands instead of land plus number of trees.
(2)(d) Public Works Department norms for the cost of construction of buildings
prevailing in the concerned area may be indicated.
(2)(e) Basic civic amenities such as electricity, water supply and drainage need
not be considered as special features for the purpose of cost of construction.
This should be considered as a part of the cost of construction.
Seminar on Valuation of Properties - 2011 25

KERALA
The Kerala Stamp Act, 1959 with Rules and Allied Laws Authors comments

Procedure for determination of the value or consideration and the proper


duty under [Section 45B]

5. Principles for determination of value or consideration:-


The Collector shall, as far as possible, have also regard to the following
points in arriving at the provisional value or consideration, namely:-
5(a) in the case of lands: -
(i) nature of the land or ground such as garden, wet and the like;
(ii) other factors, which influence the valuation of the land in question;
(iii) points, if any, mentioned by the parties to the instrument or any other
person which require special consideration;
(iv) value of adjacent lands or lands in the vicinity;
(v) average yield from the land, nearness to road and market, level of
land, transport facilities, facilities available for irrigation, such as tank,
wells etc.;
(vi) the nature of crops raised on the; and
(vii)* the use of land, domestic, commercial, industrial or agricultural
purposes.
* Ins. By Kerala Stamp (Prevention of Under Valuation of Instruments)
Amendment Rules, 2007, SRO No.153/2007 pub. In G.O.(P) No. 28/
2007/TD. Dt. 17-2-2007.

5(b) in the case of house-sites: -


(i) the general value of house-sites in the locality;
(ii) nearness to roads, railway station, bus route, etc.;
(iii) means to market, shops and the like;
(iv) amenities available in the place like public offices, hospitals and
educational institution;
(v) development activities, industrial improvements in the vicinity;
(vi) valuation of sites with reference to taxation records of the local
authorities concerned, if any;
(vii) any other features having a special bearing on the valuation of the
site; and
(viii) any special feature of the case represented by the parties.

5(c) in the case of buildings: -


(i) type and structure;
(ii) locality in which constructed;
26 Seminar on Valuation of Properties - 2011

(iii) plinth area;


(iv) year of construction;
(v) kind of materials used;
(vi) rate of depreciation;
(vii) fluctuation in rates;
(viii) any other features that have a bearing on the value;
(ix) property tax with reference to taxation records of the local authority
concerned;
(x) the purpose for which the building is being used and the income, if
any, by way of rent per annum secured on building; and
(xi) any special feature of the case represented by the parties

5(d) Properties other than lands, house-sites and buildings:-


(i) the nature and condition of the property;
(ii) purpose for which the property is being put to use; and
(iii) any other special features having a bearing on the valuation of the
property.

These guidelines lead us to conclude that there is no Y Amenities like swimming pool, garden, lifts,
clarity on factors to be taken into consideration in security intercom facility
estimating market value. Y Modern habitation styles, planning and
design concepts
4.0 The following Valuation maxims are Y Environmental conditions like noise and
required to be considered for estimating smoke pollution, sea or lake view, proximity
market value. of factory area, railway track, airport
Y Climatic condition: Hot, humid, dry, cool
1. Physical / Technical Aspects climate
Y Soil conditions : Poor soil, rocky soil,
Y Land characteristics like size, shape, reclaimed ground
frontage, vista, plot area, orientation, Y Natural calamities prone area: Earthquake
topography zone, cyclone typhoon areas, flooding
Y Infrastructure facilities in area -Roads, hazards
water supply, power, tele communication
Y Building characteristics like load bearing, 2. Legal Aspects
steel frame or R.C.C. frame, specifications
for tiles, doors, windows, plumbing, Y Rent Control Act - Frozen rent provisions,
electrification, etc. Aesthetics, age, exemption
depreciation, workmanship, maintenance Y Urban Land Ceiling Act - Limited holding
and repairs of land
Y Prominence : Main road, Bye-lane, Y Coastal Regulations - No development in
Remote area locations 500 M belt
Seminar on Valuation of Properties - 2011 27

Y Town Planning Act - Reservations and 7. Demand


zoning
Y Transfer of Property Act - Transfer of partial Y If there is no demand, there may be no
ownership by leasing, joint ownership / value.
co-ownership
Y Building Regulations - F.S.I. and open 8. Transferability
space rules
Y Wealth-tax Act - Statutory valuations Y The ownership and possession of the
property can be transferred by the way of
3. Social Aspects sale, gift, lease, mortgage, Will etc. and
hence it has a value.
Y Locality Poor / middle / high class
Y Proximity to civic amenities 9. Value means present worth of future benefits.
Y Neighborhood
Y Religion 10. In exercise of valuation of tangible assets
actually valuation of intangible rights of owner
4. Economic Aspects in owning the tangible assets is carried out and
not the valuation of tangible assets.
Y Demand and supply
Y State and Centres economic policies I term these valuation maxims as Ten
Y Inflation and deflation Commandments of valuation.
Y Income fetching capacity of property
Y Money market situation If we compare the above valuation maxims with the
Y Cyclic boom and recession periods in Real principles given in the manual of valuation Gujarat
Estate Market and other states, it is found that valuation worked as
Y High rentals maintainability and outgoings per principles of valuation given in manuals will be
Y Paying capacity of people in the locality or without considering all the relevant factors
region enumerated above.
Y Better alternative uses vis--vis current
inferior use Let us consider the case of six properties mentioned
Y Availability of substitute or alternative asset below. If these properties are valued as per principles
Y Employment opportunities in the area laid down in manuals then the value estimated will
not have difference of more than 15%. But in reality
5. Utility
value differs from 22.6 lakhs to 140 lakhs i.e. about
700%!! as given below:
Y Movable personal properties or land with
building have some use and hence value
There are six properties located at Chennai, with
6. Scarcity same land area with buildings. (having same area,
size, maintenance, shape, design, etc.). There is no
Y The properties which are available in limited balance potential for further development for all
quantity and are not in abundance like air buildings.
and sea water
28 Seminar on Valuation of Properties - 2011

Area of land = 400 sq. m. Date of completion of construction 1 st


Built up area of building = 225 sq. m. Aug.2010. Age 11 months.
The realizable value of each of the six properties will
Date as on which valuation is made:30th June be different.
2011.
Prima-facie it appears that value of all the property (a) Property A is owner occupied and purchaser
will be the same. Actually that is not the case on can get the vacant possession and similar vacant
account of the following facts:- property are sold for Rs. 75 lakhs; giving a
market value of Property A Rs. 75 lakhs.
(a) Property A is owner occupied. (b) Property B is fetching net rent of Rs. 15,000/
Date of completion of construction 1 st - per month. It is more than 5 years old
Aug.2010. Age .11 months (c) Property C is fetching net rent at Rs. 15,000/
(b) Property B is rented at net rent of Rs. 15,000/ - per month. It is less than 5 years old.
- per month
Date of completion of construction 1 st Prima facie it will appear that the value for both the
June.2006. Age 5yrs.1 month property B and C will be same as both property
(c) Property C is rented at net rent of Rs. 15,000/ are fetching net rent of Rs. 15,000/- per month.
- per month But that is not the case due to following reasons:-

Date of completion of construction 1 st - Tenant of property B paying a rent of Rs.


Aug.2010. Age 11 months. 15,000/- per month is protected under the rent
(d) Property D is constructed on land taken on control act.
lease and there is a covenant to pay unearned
increase at 50% in the event of sale or transfer. This building is having age more than 5years.
Date of completion of construction 1 st
Section 30(i) of Tamil Nadu Buildings (Lease and
Aug.2010. Age .11 months
Rent Control Act, 1960,exempts building from
(e) Property E is constructed on land taken on operation of rent act for a period of 5 years, but in
lease. Unexpired period of lease is 10 years. this case age of the building is more than 5 years and
There is no renewal clause. There is a covenant therefore tenant falls under the category of a
to surrender the building constructed by lessee protected tenant. Hence, land lord will not get the
to the lessor without any compensation after vacant possession of property, if tenant abides the
expiry of the lease period. provisions of rent control act applicable to Chennai.
Date of completion of construction 1 st Due to this value of right of reversion will be
Aug.2010. Age .11 months. negligible.
(f) Property F is in residential zone as per
Computation of Value.
Development Plan in force and as per revised
draft plan which will come into force shortly it
Value of rented Property
falls under commercial zone. Property is entirely
= Capitalized value of net rent received
owner occupied.
+
Value of right of reversion
Seminar on Valuation of Properties - 2011 29

= Capitalized value of net income for infinite years = [15,000x12x3.246] + [75,00,000 x


(say 100 years) = 15,000 x 12 x 100/8 0.79383]
= Rs. 22,50,000/- (a) = Rs. 5,84,280/- + Rs. 59,54,000/-
+
Value of right of reversion of property worth Note:
75 lakhs available after 100 years = Y.P. for 3 years at 8% = 3.246
Present Value of Rs. 1/- available after 3 years
Present Value of Rs. 75 lakhs available at 8%
after 100 years at 8% Present Value of Rs.1/- available after 3 years
PV of Rs. 1/- available at 8% after 100 years = at 8% = 0.79383
Rs. 0.00045
(d) In case of Property D while valuing lessees
Value of right of reversion interest 50% unearned increase payable to
= Rs. 75,00,000 x 0.00045 lessor is a liability and requires to be deducted.
= Rs. 3,375/- (b) Value of property as freeholder is Rs. 75 lakhs.
Price at which land will originally purchased is
Total Value = (a) + (b) say Rs. 10 lakhs. As per current price it works
= 22,50,000 + 3,375 out to Rs. 65 lakhs.
= Rs. 22,53,375/- Unearned Increase = 65 10
= Rs. 55 lakhs
Say Rs. 22.53 lakhs Value of property after considering 50%
unearned increase
- Tenant of property C paying a rent of Rs. = 75 (55 /2)
15,000/- per month is not protected under rent = Rs. 47.50 lakhs
control act as the age building is less than 5
years.Due to this property will be reverted back (e) Purchaser of the Property E can receive the
to the owner after the expiry of period agreed benefit for unexpired period of lease i.e. 10
as per rental agreement, value of the property years only. This will depress value considerably.
will be capitalised value of rent for unexpired After 10 years property is to be surrendered to
period of rental agreement plus value of lessor free of cost. Therefore, purchaser can
reversion of property worth Rs. 75 lakhs which expect rent for next 10 years. Current net rent
will be available after expiry of rental agreement. fetching capacity of property is Rs. 3,60,000/-
In this case if unexpired period of rental per annum.
agreement is 3 years than value of property
works out as under:- Value of Property at Y. P. at 8% for 10 years
works out to:-
Value of rented property = Rs. 3,75,000 x 6.7
= Capitalized Value of net income for 3 = Rs. 25,12,500/-
years at 8% = Say Rs. 25 lacs
+
Value of right of reversion (f) In case of Property F existing use is residential
but has a potential for commercial. Due to this,
30 Seminar on Valuation of Properties - 2011

it will command high value if commercial use is Summary of Valuation ( in lakhs)


lucrative. Let us assume that commercial use is Property A 75.00
lucrative. Current land rate for commercial B 22.53
land in locality is Rs. 35,000/- per month C 59.54
which gives a value of about Rs. 140 lakhs. D 47.50
(Value of structure is not considered as it will E 25.00
be required to be demolished). F 140.00
Seminar on Valuation of Properties - 2011 31

EFFECT OF RENTALS ON COMMERCIAL PROPERTIES ACROSS


THE CITY
Narayanaswamy B.S.
Preamble: d) The market is supreme and the estimation of market
It is a common observation made by valuers that the value by valuers may go awry and may put the
rentals for commercial Properties such as commercial related parties and them in awkward position.
shops, office spaces, malls and Tech parks varies in
different parts of the same city. It is also observed that The purpose of this article is to look for the reasons
though there is a marginal change in the market value of for such a behaviour by the market and to identify the
the land, the rents varies disproportionately with that of parameters which impact the market value of income
change in the market value of the land. The valuers fetching properties to estimate the fair market value.
might have also observed that different office spaces in
City of study
the same road fetch differential rents.
The city of Bangalore is chosen for this purpose as this
The recessionary trends in the real estate market during is one of the cities which has sizable number of
the last two years and the present recovery process commercial properties in India.
of property values across various cities, albeit slowly,
Instances
makes an interesting study in this matter. During the
Instance 1
recessionary period, the market values of properties
situated at the heart of cities and Central Business A Tech park situated just off the CBD area was
Districts have shown lesser downward variations than demanding a rent varying from Rs.45/- to Rs. 50/- per
that at the outskirts. In some cases, a marginal increase Sq.ft. during 2008 for the warm shell. The rentals were
in the land value is also noticed. On the other hand, the for the large floor plates and are occupied by MNCs
fall in rentals during this period have been more steep and other major IT companies. Earlier, the rentals were
both at central areas of the city and outskirts compared as high as Rs.75/- per Sq.ft. It is now gathered that
to that of the market value of the land. the rentals have fallen to Rs. 35/- for warm shell in the
said tech park by the end of 2010. On the other hand
Though this behavior of the market looks simple and
the market value of the larger size of land which was
cognizable, valuer may find it difficult to estimate the
around Rs.3500/- per Sq.ft during the peak period
market value of these properties. Since, these are
during 2005-06 have marginally reduced to
income fetching properties, income approach is most
Rs. 3200/- now. The present position is that the rentals
suitable one and generally it is cross verified by other
have fallen further in the vicinity but the land value
methods.
remains more or less at the same level.
The task before the valuers would be:
Instance 2
a) How to recognize the market behaviour?
b) How to study the impact of such behaviour on A commercial office space at heart of the city with land
market value of the properties. area of about 12000 Sq.Ft and built up area of about
c) How to identify the parameters required for valuing 33000 Sq.Ft and leasable area of about 25000 Sq.ft
the property under income approach? was fetching rent of Rs.60/- per Sq.ft. as per the lease

Chief Manager, Syndicate Bank, Hyderabad


32 Seminar on Valuation of Properties - 2011

agreement entered during the year 2006 for the entire Rs.13.50 Crore. While the investor was looking for a
leasable area. The lease was for 5 years with a minimum loan in the vicinity of Rs.11.00 Crore, he got a rude
lease period of 3 years. After three years, the lessee shock when the bank sanctioned loan of Rs. 8.00 Crore
vacated the premises for the reason that the rent was after considering the margin. There was a big argument
high and better premises were available at lesser rent. between the investor and the bank. While the investor
During Dec. 2010, the rents have fallen to Rs.30/- to was quoting his own instance of purchase / sale, the
Rs. 40/- in this area. The lessee had all the reasons to panel valuer of the Bank was defending his estimation
vacate the premises. On the other hand the land value based on the following factors.
had a small correction from Rs.20000/- to Rs.18000/
- for the similar and comparable land. There was a recent instance of sale wherein 175,000
Sq.ft. floor space was sold at Rs. 44.00 Crore
Instance 3 (About Rs.2500/- per Sq.ft.). Earlier comparable
land parcels of similar size were exchanged at
A tech park at outskirts of the city surrounded by Rs.2800/- to Rs.3000/-.
commercial spaces and situated on National Highway
A portion of the same building of about 90,000 Sq.ft.
was commanding a rent of Rs.45/- per Sq.ft for the
was let out on lease at ` .23/- per Sq.ft.
bare shell and the entire leasable area of more than
4,00,000 Sq.ft was leased out to two major IT giants. The rental values have fallen down from Rs.45/-
Now the rentals have fallen to Rs.25/- per Sq.ft. for per Sq.ft. to Rs. 23/- to Rs. 25/- per Sq.ft.
the bare shell where as the land value has shown a
There was oversupply of office space and rents have
downward correction of about 10%. There has been
fallen.
over supply of office space in this area and more supply
is expected in next two years. It is expected that due
Panel valuer further defended that he has taken in to
to market forces the rentals may fall further.
consideration over supply of office space, sale and rental
instances and arrived at yield, the rate of capitalization,
Instance 4
and accordingly the years purchase to estimate the value
of the property.
An investor had bought 40,000 Sq.ft. of floor plate
space in Ground floor of a large Techpark in the suburbs
Instance - 5
of a city developed by a reputed developer. The
undivided share of the land is about 18,500 Sq.ft. The
An office premises at CBD located abutting the main
developer was quoting a rate of Rs.5000/- per Sq.ft.
road with total office space of 2,00,000 Sq.ft. was
The investor engaged the services of local valuer, who
commanding a rent of Rs.40-45 since last 6 years. The
arrived at the fair market rate of the property at
rent is almost stagnated since last 6 years. This is due
Rs.4500/- per Sq.Ft. Accordingly after due negotiation
to the fact that the type of structure, quality of office
including payment terms etc., this property (floor space
space and amenities provided were outdated even
of 40,000 Sq.ft) was purchased by investor at Rs.4500/
though the location was ideal than many other premises
- per Sq.ft i.e. Rs.18.00 Crore. The investor in order
situated on the same road. Other premises situated in
to part fund this project sought loan from a Bank. The
inner roads are commanding more rent. Office premises
Bank agreed in principle to fund 60% of the value
with better ambience, infrastructure and amenities are
arrived by their panel valuer. The panel valuer of the
commanding more rent in the same road. The land
Bank estimated market value of this property at
price which was about Rs.16000/- per Sq.ft. in the
Seminar on Valuation of Properties - 2011 33

year 2004 have now moved to Rs. 25000/- despite The smaller shop at ground floor and abutting main
recession. road which has retail commercial potential are
exchanged at higher market value than a larger
Instance 6 commercial office space in upper floors of the same
building / in the vicinity.
A small shop at the ground floor admeasuring 670 sq.ft The micro economic factors and other factors
in a major commercial building built by leading impacting the local scene and the overall macro
developer abutting 80 ft road on the outskirts of the economic factors such as recession etc have been
city occupied by a Retail MNC player was commanding responsible for this behaviour.
a rent of Rs.36000/- p.m. (Rs.54/- per Sq.ft. per
month). The said shop is situated in the ground floor There is a pronounced need for the valuer to understand
facing the main road. The Developer is quoting a rate the local market behaviour and also the economic factors
of Rs.6500/- per Sq.ft. for outright sale. A Valuer has on the large canvas. Otherwise the related parties and
advised the investor to accept the rate in the vicinity of thereby the valuers may come across embarrassing
Rs.5750/- per Sq.ft. The larger Office space in the situation as mentioned in Instance No 4.
same building / road is available at Rs.2500/- per
Sq.ft. for outright sale and at rentals at Rs.25/- per Sq.ft. The analysis indicate that the fall or rise in rentals have
impact on the yield. It may also be seen that yield is
Analysis of instances not the same at Heart of the city /CBD and outskirts.
The yield is also not the same across the various
Analysis of these instances indicates that there is a need commercial properties situated in the same road/locality.
to study this divergent behaviour of the market in depth
and to adopt suitable changes to arrive at fair market What is yield?
value. Following observations can be made out of these
instances. Yield = Net annual rent / Capital value of the property.

The capital cost of the building and other amenity Yield is the tool for the valuer and it is also the defined
remain more or less same across the city, whether it path for the rate of capitalization. The concept of rate
is centrally located or in the outskirts and it is the of capitalization is to be seen in a holistic manner and
land value which makes the difference in property compared with other forms of investments.
value.
The rentals both at heart of the city and at outskirts A closer study of the above instances indicate that the
have fallen steeply when compared to the variations capital value of the properties situated at the heart of
in the land value. the city/CBD will be more than that at the outskirts as
The fall in the rentals at the heart of the city is more the cost of the buildings and the amenities provided are
steep than and at the outskirts in some cases. more or less the same at both the places. The difference
in land value constitutes the difference. Now what
The land rates have shown small correction and in happens to the yield?
some cases at city centre and CBD it has moved
upwards also. From the analysis of the instances, it is evident that the
There have been instances of variation in rentals of yields for commercial properties are more at outskirts
commercial properties situated in the same road. than at the Heart of the city/CBD. Further, the yield
34 Seminar on Valuation of Properties - 2011

varies amongst the commercial properties situated at Market value of the property is arrived by multiplying
the same place. While the small commercial shop the yield in absolute terms with number of Years
fetches lower yields, large office space fetches higher purchase.
yield. The yield for the tech park is more due to the
risk factors involved. The above discussions indicate the importance of yield.
Hence, the skill of the valuer is in correct assessment of
Yield and Year's Purchase yield.

Yield In the instances quoted above, it may be seen that the


yield at the outskirts are more than the yield at the heart
The yield is also the net income of the property of the City/CBD. More the yield lesser is the market
expressed in terms of percentage to the capital value of value and vice versa. In the instances quoted above,
the property. In other words it also is the rate of as per the valuation made, the yield at the heart of the
capitalization. Since the valuer has to estimate the City/CBD was about 3.50% to 4.50% and at the
market value of the property ( i.e. capital value of the outskirts it was as high as 10-11%. Given the same
property), he has to estimate the yield using his rentals, the market value of commercial properties at
experience, data bank and study of local and macro the heart of the City/CBD would be more than two
market. His estimate of yield becomes the key for times than that of the similar rent yielding properties at
valuation and that is the reason it is rightly said that outskirts.
yield is the pathway to rate of capitalization.
The relation between the yield and Years purchase is
What will happen to assessment of market value of graphically represented as follows.
the property if same yield is taken in to account for
commercial properties across the city? YIELD & YEARS PURCHASE

1
0.9
What will happen to assessment of market value of 0.8
)
the property if the same yield is taken in to account s
r 0.7
a0.6
e
for all types of commercial properties in same area? y
f 0.5
o
. 0.4
o
N
( 0.3
Years purchase P
Y0.2
0.1
0
The years purchase can be defined as capitalized value 1 2 3 4 5 6 7 8 9 10 11 12
paid at one time to receive annual income (Annuity) Yield (%)

Rs.1/- for a certain period at certain rate of return. The


rate of return is nothing but yield (i) which requires an It may be seen that the resultant graph is an Hyperbola
intelligent assessment. and with increase in yield, the years purchase ( In years)
reduces and consequently the market value also gets
Mathematically years purchase (YP) is YP = 100/ i, reduced given the same rentals. This concept is to be
where i is the rate of interest which is nothing but clearly understood by the valuers especially while
yield. Hence yield is an independent variable and Years assessing the market value of the property under income
purchase is dependent variable.
Seminar on Valuation of Properties - 2011 35

approach. Income approach to valuation is also known Let us analyse instance -6. The smaller commercial
as Investment method, Rental method and Rent space which has retail potential due to locational factors
capitalization method fetch more market value/rentals than the large office
space situated in the same building. The yield is low
The yield which determines the rate of capitalization for smaller retail commercial space and high for larger
behaves in a divergent manner and is very sensitive to office space indicating that the fair market value of
local factors, parameters and other macro economic smaller retail commercial space is higher than that of
factors. The market is supreme and is affected by the office space.
various micro and macro factors. It also takes in to
consideration other social, legal and environmental Yield of lease hold properties:
factors also. The interaction of all these factors will
reflect in market value and rentals. Hence, yield reflects Yield of lease hold properties are influenced by terms
the fair market value provided it is recognized and of lease such as period of lease, balance period of
assessed properly. Thus, estimation of the yield assumes lease, lock in period of lease and interest free deposits
the paramount importance. both refundable and non refundable. In instance No.2,
due to fall in rentals between 2006 and 2009, the yield
The valuer generally will not get the feed back as to the varied. After completion of the lock in period, in the
correctness of his valuation until he has knowledge of scenario of falling rents, it is advantageous for the
the sale instances of similar property in the same location. lessee to terminate lease immediately after the lock in
Till such time, he will be under the impression the period. On the other hand it is advantageous to the
valuation exercise carried out by him reflects the fair lessor, if no lock in period is specified and the period of
market value. Hence, a professional valuer must be lease is long, when rentals fall and vice versa. Interest
doubly sure to assess the yield and thereby the market free deposits are generally agreed along with minimum
value taking in to consideration the required parameters lease period. If there is any agreement to the contrary
and factors. or not specified, they will influence the yield.

How to arrive at the yield:


Now, let us analyse the instance 5 with this
background. As seen in Instance 5, the rentals have It is to be clearly understood that yield is an independent
stagnated at Rs.40 45 since last 6 years. Now variable which is impacted by time factors, location
ascertain whether the yield is same for the last six years. factors, individual preferences, legal factors, social
Prima facie, it appears that the yield is same, but it is factors, environmental factors and any other local factors
not. With the increase in the land value over the last apart from micro and macro economic factors. The
years, the market value of the property has increased yield can vary for the same property in different time
resulting in lower yield year after year. The reduction factors. The yield may vary for same type of properties
in yield results in higher Years purchase and the in the same road. Yield is thus independent of any
increased market value due to increase in land value particular behaviour and is outcome of interaction of
will be reflected. What will happen if the same yield is many factors and becomes complex. Hence, it is for
taken in to account? The estimation of the market value the valuers to first understand the various factors and
of the property will go awry. their impact on the yield and recognize the correct yield.
36 Seminar on Valuation of Properties - 2011

The skill of the valuers lies in correct assessment of the which is capable of appreciation will be low as observed
yield so that he can estimate the fair market value. in properties situated at the heart of the city/CBD.

In order to estimate the correct yield, comparison is to The yield which is expressed as a single arithmetical
be made with yield fetching investments such as Bank number in percentage terms appears very simple.
deposits, Treasury bonds, debenture and bonds of
companies, appreciable investments such as stocks, In reality, this number reflects interaction of umpteen
gold etc. The criteria of security, the risk factors number of factors affecting the market value of the
involved, the chances of capital appreciation are also property and the success of the valuer is dependent on
to be taken in to account. The yield for the properties assessing this simple arithmetical number.
Seminar on Valuation of Properties - 2011 37

VALUATION OF A PLANT FOR THE PURPOSE OF LIQUIDATION

Vr. NA. Arunn

1.0 INTRODUCTION There is a saying that An able leader should be firm


to use the word No, when he is required to use it and
With Banks or Financial Institutions taking a stringent should not kneel down or accept everything on all
step for recovery of financial assistances that they had occasions. In a similar way if a business is not thriving,
made, liquidation has become a common Term and the owner should be brave enough to close the business
the role of an Official liquidator is gaining importance. and venture in to a new one and succeed. World over,
insolvency procedures help entrepreneurs to close down
To know about liquidation, we will first understand what
unviable businesses and start up new ones.
is 'Liquidation'?
The closure may become inevitable due to change in
Liquidation means to pay off or settle a debt or claim Technology or non availability of adequate raw material
or an obligation. / labour / power, or poor management or bad business
judgement or even fraud. In such cases it is not advisable
It also means to windup the affairs of a business or a to hang around the same business. In such cases, the
bankrupt estate by determining the liabilities and applying productivity of the enterprise could be restored at a
the assets to their discharge. In short, it literally means low cost by providing more capable managerial talent
turning business assets into readily available cash. and opportunity to run it. The entrepreneur who had
lost the business may not be interested in pumping in
It also helps for avoiding further losses on the same additional resources; the best way is to liquidate and
business that erodes the capital. venture in to a new business allowing the capable man
to run the show successfully. It is nothing but turning
The value of Liquidation will be either the forced sale around of a sick firm and transforming it to be a profit
value or less and not the fair market value. It is the making unit. In fact in recent times, a lot of
entrepreneurs, even from India, have become dominant
amount that may reasonably be received from the sale
business entities internationally.
of a property within shortest period. Here, neither the
seller nor the buyer may act prudently with the It is a fact to be noted that the systems in India do not
knowledge of the product. provide any opportunity for speedy and effective
rehabilitation or for an efficient exit unless the
2.0 WHY A LIQUIDATION?
entrepreneur decides to liquidate. In our country, the
Any business may require a transformation in due course process for rehabilitation is regulated by the Sick
of time. It is nothing but to rechannalise the human and Industrial Companies (Special Provisions) Act 1985
economic resources of the country for an efficient use through the institutional structure of BIFR. The
to increase the overall productivity of the country. As procedure is a very lengthy one and it causes the delay.
the success of any business is based on efficient and It does not provide a balanced or effective framework
speedy procedures to start up, it also requires speedy for all stakeholders. As a result, the entire assets lose
procedures for exit also. their material value in the process of liquidation and
winding up.

Practising Valuer, Trichy


38 Seminar on Valuation of Properties - 2011

Similarly, for the banks and financial institutions, they (a) Liquidation under normal conditions
need to liquidate their Non-Performing Assets as early
as possible to realize more value, otherwise, the value (b) Liquidation under distress conditions
of such Non-Performing Assets reduces very quickly.
Fortunately THE SECURITIZATION AND 4.1 LIQUIDATION UNDER NORMAL
RECONSTRUCTION OF FINANCIAL ASSETS CONDITIONS
AND ENFORCEMENT OF SECURITY INTEREST
ACT, 2002 widely known as 'SRFAESI' Act has been By this method, the liquidation is almost like auctioning
introduced to provide a faster relief to some extent, on a normal course, in a reasonable time frame to the
highest bidder. It assumes an orderly sale process and
3.0 WHO ELSE CAN OPT FOR the seller can take time to sell each asset in its
LIQUIDATION? appropriate season and through channels of sale that
can fetch the highest price.
Y The entrepreneur himself for switching over to the
next profitable line of activity. 4.2. LIQUIDATION UNDER DISTRESS
CONDITIONS
Y The partners when they opt for a judicial remedy
to come out of the partnership
It is selling off the entire assets under a 'distress' condition
Y Banks/financial institutions who have offered the to any price at the same time or closer to that time.
financial assistance Generally, the buyers may be regular bidders of auction
or sellers of scrap. They look at everything on the
'Financial assistance' means any loan or advance material of construction basis and not on their functional
granted or any debentures or bonds subscribed or basis.
any guarantees given or letters of credit established
or any other credit facility extended by any bank It goes with out saying that the Liquidation under distress
or financial institution condition will always be lower than the Liquidation
under normal condition. There is no fixed percentage
Y Any Reconstruction company, (a company formed of deviation between these two conditions. Depending
and registered under the Companies Act, 1956 (1 on the enterprise and the nature of its assets, the
of 1956) for the purpose of asset reconstruction) difference between the two values may be substantial.
to whom the Bankers have assigned the job of
reconstruction for the purpose of realization of If the property to be liquidated is of perishable / fragile
financial assistance offered by them and which has variety, then liquidation under distress condition
become a non performing one. automatically sets in.

Y Securitization Company 5.0 WHAT ELSE TO BE DONE IN THE


PROCESS OF LIQUIDATION?
4.0 VARIOUS TYPES OF LIQUIDATION
5.1. The property is to be valued first to ascertain the
There are actually two types of liquidation, liquidation value
depending on the time available for the liquidation
process:
Seminar on Valuation of Properties - 2011 39

5.2. To find out a buyer who could run the unit as unit 5.1.2 IF PROPERTY IS AN INDUSTRIAL
itself or UNIT, THEN,

The functioning of the unit is to be studied first, as a


5.3. To push through the buyer, may be a scrap
running unit is valued more than any unit under
merchant, the special purpose machines and to sell the
closure.
general purpose machines to a trader of second hand
machines Valuer should remember that Value is a function of
Time: Since obsolescence plays an important role
5.4. To realize the cash as early as possible on behalf while valuing the machines, the value arrived at on a
of lenders particular day holds good on that day alone and
cannot be decided for any particular period.
5.1 VALUATION TO ASCERTAIN THE
He should also remember that Value is a factor of
LIQUIDATION VALUE
environment. Industry situated in a sea shore or in
an air polluted area suffers corrosion when compared
Whenever a Valuer is assigned a job to carry out
to the industries of other belt.
valuation of Fixed Asset for the purpose of Liquidation
the following points are to be considered. The Macro and Micro aspects of the Industry as a
whole and subject unit in particular are to be studied
5.1.1 IF THE PROPERTY IS RESIDENTIAL on the following aspects
OR COMMERCIAL PROPERTY, THEN,
5.1.2.1 MACRO ASPECTS OF THE INDUSTRY
The property is to be physically inspected. TO BE LOOKED UPON ARE:
The built up area is to be measured and the The nature of industry
depreciated value of the buildings are to be The technology used and its obsolescence with
estimated. reference to the date of valuation
The market value of the property for the land portion The profitability, average return on capital invested,
with reference to the demand and supply that prevails demand of the products, present capacity, excess
in that area is to be studied. or shortfall of supply compared to demand.
Government policy, tax and duty structure and other
The details of recent sale instances with reference
incentives available to the industry as a whole.
to the details of area, date of transaction, technical
specifications are to be acquired; the offers for a Other comparable units which fall in the same
sale, if any are also to be considered, industry group. Their product capacity, year of
establishment, investment made in fixed assets,
The plus and minus points of the property are to be Balance sheet, Profit & Loss account and other
considered to add or deduct weightages. financial ratios. This information can be had from
The time required to liquidate subject asset based the following websites, of which some are paid
on market study is to be estimated. services:
Y www.capitaline.com (Paid Service)
The market value and the value for liquidation are
then to be estimated based on the above findings. Y www.cmie.com (Paid Service)
Y www.mca.gov.in
Necessary photographs are to be taken.
40 Seminar on Valuation of Properties - 2011

5.1.2.2 MICRO ASPECTS OF THE SUBJECT Copies of financial statements for past 2 years
INDUSTRIAL UNIT ARE LIKE: Copies of all statutory permission / approvals
Location of the subject property, its connectivity by obtained
road, rail, air, sea and the distances from nearby List of machinery imported as well indigenous
major cities with the advantages and disadvantages
of the location. Copies of purchase details if the machines are second
hand and if the manufacturing plant as a whole was
If it is in an industrial estate, every information about purchased in public auction
Industrial Estate like infrastructure provided by the
industrial estate, nos. of plots available, no. of Drawings of the building
industrial units in the estate with their nature of Optimum requirement of land and details of excess
functioning. land if available and its value
Type of industry: small, medium or large scale Details of production carried out since inception of
industry the unit
The present use of the subject property, the highest Period of working of the unit with details of closure,
and best use or the available alternative use like lay off etc and the reasons for the same
godown, warehouse etc.
Requirement of financial assistance and its availability
The recent sale instances with date of transaction or
even the offers of similar property along with Area, 5.2. TO LOOK IN TO THE POTENTIAL
technical specifications etc PURCHASERS OF THE PROPERTY
The weightages to be assigned on various aspects To prepare a list of potential buyers that they might
or parameters according to importance attached have come across and submit it to the assigner
The time required to liquidate subject asset based To provide the details of brokers of real estate and
on the market study, study of demand and supply the dealers of machinery separately to enable them
The market rate based on above sales analysis and to liquidate easily
also the liquidation value The information about Real Estate Market, Agents,
Types of Plant & Machinery: continuous process or Brokers, Property available for Sale, Rent etc can be
a batch process; used for general purpose or special available from the following websites:
purpose or custom built machinery,
www.99acres.com
Physical and working condition; the approximate www.indiaproperty.com
cost of repairing and overhauling to make the www.indiaproperties.com
industry marketable www.magicbricks.com
www.indiarealestatedirectory.com
5.1.2.3 DOCUMENTS TO BE COLLECTED www.realestateonline.com
TO VERIFY THE DETAILS ON MICRO AND www.realtymart.in
MACRO LEVEL: www.realestatemumbai.com
Memorandum and Articles of association of the www.timesproperties.com
company www.persquareyard.com
Copy of fixed assets register
Seminar on Valuation of Properties - 2011 41

6.0 CONCLUSION Replacement value projected = 7880504 x123.4/100


After the enactment of 'The Securitization and = Rs. 9724542
Reconstruction of Financial Assets and Enforcement
of Security Interest Act, 2002 (SRFAESI Act)', the Life is estimated as 20 years.
procedure to take charge of the Assets and get it
liquidated becomes very fast and this will really help Salvage value is assumed as 15%
the Banks or Financial Institutions and even borrower
to liquidate the asset as quickly as possible and realize Depreciation arrived in Straight line method
maximum value, otherwise the Value of NPAs erode = 9724542 x 7/20 x (100-15)
very fast. So the valuers must study about liquidation = Rs. 2893051
and possibly about the SRFAESI Act to justify our duty
as a valuer to estimate the Liquidation Value of the asset Depreciated value = 9724542 2893051
in a much better way. = Rs. 6831490

7.0 CASE STUDY I (FOR DISPOSAL OF A Say : Rs. 68,31,000


SINGLE MACHINE OF AN
ESTABLISHMENT)
There is no offer for the machine as it is of a machine of
A manufacturing unit of HDPE woven sacks unit was specific purpose. Other manufacturers of the machine
having a Tape manufacturing unit (Extruder with cheese quote a lower rate for the machine of the same capacity
winders) with a capacity of 150 kgs of extrusion. As (The RIV of the same capacity m/c of other brands is
there were frequent unscheduled power cuts, they around Rs. 70 lakhs). There are fabricators like
preferred to increase the capacity of production during Kalimatha & Co., Bangaluru, who could sell the
the non power cut period to extrude tapes and use the machine at a very reasonable price, say for Rs. 50
same during power cut periods also in the weaving Lakhs, as they could fabricate the same with the
machines. (The requirement of power for weaving components that they bring at the site itself. They avoid
machines was manageable by augmentation of power Tax and duty elements.
through generators.)
Advertisement in local daily was in vain and the
The purchase cost of an extruder of 150 Kg/hr. information by word of mouth brought a customer, who
capacity in 2004 was : Rs. 7880504/- from his existing capacity of 75 Kg/Hr. was looking for
a plant of 150 Kg/hr.
The replacement value is estimated by using RBI indices
as Kolsite have stopped production of extruders in Both of them have adequate knowledge about the
2008 itself. machine. It is not Liquidation under distress condition.
The seller is in need to sell and the buyer has interest on
RBI Index in April 2004 for : 100 the Machine. What could be the price that could be
industrial electrical M/c negotiated?

RBI Index projected for current period : 123.4 What are the effects if the sale is not concluded? Will
on same head there be a scope for a better consideration?
42 Seminar on Valuation of Properties - 2011

VALUER'S OPINION took a project report from the detergent unit, submitted
to the bank, gave his bungalow as security availed a
The depreciated value of a similar machine of other loan and installed the unit. After 3 months of production,
brand works out as Rs. 49,17,500/- and that of the the detergent manufacturer felt that it was not easy to
fabricated one as Rs. 35,12,500/-. As it is an asset fight against his competitor and withdrew the order.
under liquidation for the present owner, the value could
be anywhere in between. The valuer prefers to reduce The entire industry was on loss and the bankers seized
10% towards obsolescence and 30% towards the rare the plant and other assets, liquidated the house and are
preference in market and estimates the value for a total struck with the spray dryer.
discounted price of 40% lesser to the depreciated value.
The valuer is unable to fix the value with all his
i.e. the value assessed on liquidation is 60% on experiences as there are no off takers.
Rs. 68,31,000
which will workout to Rs. 40,98,600 As these are specific purpose machines, a better
consideration cannot be assured and the owner may
The real consideration was ascertained to be have to suffer loss on following heads:
Rs. 32,00,000/-
1. Lower amount of consideration. (The difference
Note: If it is a general purpose machine the reduction in amount of this bid and the one that may take
factor could be 10 to 20% of the depreciated value. place after some time)

2. On payment of interest for the amount that is


8.0 CASE STUDY - II
blocked. i.e. on the amount of liquidation.
A manufacturer of detergent cake was trying to market 3. On the earnings from the liquidated amount, if it
his product in down south and wanted an entrepreneur were invested on a profitable investment
to manufacture his products alone to his requirement.
4. On safeguarding the machine with all accessories.
This made the entrepreneur to come out of his tie ups
with other brands and made him loyal to the new 5. On purchase of items that are stolen
product. The sailing was good for 6 months and the
manufacturer was interested to produce detergent 6. On establishment charges made to retain the asset
powder and compete with the popular brands existing free from loss by natural perils or made up perils.
in the market. Hence they urged the entrepreneur to
install a spray dryer at a cost of 4 crores assuring of Hence, if an asset is a non productive one, it is better to
their entire order of manufacture of detergent poser will liquidate as early as possible and the liquidated amount
be offered to him. The entrepreneur with flying colours could be invested in some other venture for profitability
Seminar on Valuation of Properties - 2011 43

DESTINATION CHENNAI

V. Nagarajan

There has been an influx of developers from city has already made an indelible mark as the Detroit
other cities leading to setting international of South India with major automobile units setting
standards and offering lifestyle hitherto unheard up shops and expanding their existing operations.
of in Chennai.
The government is speeding up the IT expressway
As the fourth largest city of the country, Chennai has work on the 20-km stretch on Old Mahabalipuram
been focusing the attention of global investors in road from Madhya Kailash to Siruseri. The second
varied areas. The city has played a major role as phase will extend upto Mahabalipuram. The
the hub of economic, geographic and political CMDAs outer ring road covering 62 km on BOOT
scenario in the country. However, it has been a late mode is yet another boost to the citys infrastructure
entrant to realise the potential engulfing IT sector development. There will be flyovers at five locations
unlike neighbouring cities like Bangalore and along inner ring road. The airport will be expanded
Hyderabad. on 1440 acres. Project development work for 92
km on NH 205 from Tiruvallur to Renigunta will be
When the thrust was laid on IT sector by way of a big boost to millions of pilgrims visiting Tirumala.
fiscal sops and liberal FSI norms, Chennai took the
lead and since then it has been luring corporates and The national highways authority of India has been
MNCs to its fold. The inherent strengths are constructing grade separators at the Kathipara,
availability of human resource and real estate. No Koyembedu and Padi junctions, a flyover near the
wonder the city is the nerve centre of development Chennai airport and an underpass, connecting
centres, high-end BPOs in financial services, health Tirusoolam railway station and the airport. It is said
care and other back office operations. Twelve IT that the traffic volume in terms of the number of car
SEZs have been approved which is expected to units was 0.27 million a day at the Kathipara junction,
increase the availability of space in the city. 0.25 million at Koyambedu and 0.14 million at Padi.

Around 26 million sqft of IT/ITES space is under The western part has already seen electronic
construction ever since the liberalisation of FSI hardware corridor with land prices zooming along
norms for construction of IT buildings. Additional the corridor. The stretch of mega units with Hyundai-
20 million sqft has been proposed in the city. The Nokia-Saint-Gobain-Motorola-Foxconn-Dell-
IT sector contributes about 90 per cent of the total Samsung-Flextronics-Caparo lining the corridor is
demand for office space in the city. Demand exceeds ample testimony to the major industrialisation drive
supply for built, plug-and-play quality office space undertaken by the government to give a virtual boost
in the city. In fact there is an acute shortage of quality to the citys all-round development.
office space for new entrants in CBD areas. The

V Nagarajan, Editor and Publisher, Priya Publications, Flat 10, Shyam Durbar Apartments, Old No.26,
New No.21, South Mada Street, Sri nagar colony, Saidapet, Chennai 600015. Tel. 42043857,
98400 27139. Email: priyapublications@gmail.com Web: www.priyapublications.com
44 Seminar on Valuation of Properties - 2011

After a considerable lapse of time, north Chennai is The city is well connected by air, rail and road
gearing up for development. The widening of Ennore network. With a comfortable power situation and
expressway for 6.8 km stretch at an estimated cost telecom infrastructure, cosmopolitan outlook,
of Rs 1.5 billion will give a new lease of life to the Chennai is all set for a major turnaround with world-
area. class housing planned in suburbs by established
property developers in association with global
realtors.
Seminar on Valuation of Properties - 2011 45

WRITING A REPORT
REAL ESTATE VALUATION
- P.T.Hardikar
1.0 INTRODUCTION requires reasonable command over the language in
which a report is written.
A report, in general, is a device to communicate facts,
information & data etc. together with opinion and 2.2 A valuation report, no doubt, requires sound
advice if called for. The process of writing a valuation technical knowledge & skill. However for presenting
report usually starts after the process of valuation is information & data, its analysis and communicating
completed. Writing a report on valuation is often conclusion, opinion & advice efficiently (i.e.
neglected or taken lightly by valuers mainly because conveying maximum meaning in minimum words) and
once the main work of valuation is completed the effectively & convincingly also requires
enthusiasm and energy with which the work has been communicative skill which needs to be acquired and
persued gets slackened and writing a report about constantly improved. One may have sound technical
the work already done becomes a burden. knowledge and expertise but it may not be of any
Considerable inertia may have therefore to be practical use if he is unable to present information &
overcome in explaining & repeating whatever has data, its analysis & conclusion and express his opinion
been already done. Further most of the financial & advice in an effective & convincing manner. On
institutions insist on valuation reports in the the other hand one may have acquired skill of
prescribed proforma. The valuers are therefore communication but his opinion and advice may be
accustomed to such format reports which require misleading and erroneous if the same are not based
answers to the points on the check list in the form of on sound technical knowledge. A bad technical work
short notes. Such format reports hardly leave any will never produce a good report while even a high
scope for explaining logical reasoning for arriving at quality technical work will be endangered by poor
the conclusion and opinion and therefore tend to be reporting causing harm to the reputation of a valuer
arbitrary. since the work of a professional valuer is usually
judged by clients and others through his valuation
2.0 COMMUNICATIVE SKILL AND reports. Technical skill and communicative skill are
TECHNICAL SKILL thus complementary to each other and not substitutes
for one another.
2.1 A report on valuation is a communicative device
not merely to communicate information & data its 3.0 FORM OF A REPORT
analysis and opinion & advice but most importantly
to communicate it efficiently, effectively & 3.1 The physical appearance of a report together
convincingly. Report writing is basically a technique with its from, style of presentation as well as analysis
primarily concerned with 'HOW' to communicate of information & data by application of technical
rather than 'WHAT' to communicate though 'what' expertise, arrangement and phrasing of concepts &
to communicate is also important. It therefore ideas leading to conclusion all play an important role

The article gives glimpses into some of the important aspects of the subject covered in the forthcoming book
"Writing Valuation Reports:Real Estate Valuation" by Shri P. T. Hardikar, Pune
46 Seminar on Valuation of Properties - 2011

in understanding technical contents of a valuation opinion and (iii) possible consequence of such
report. A valuation report has therefore a distinct actions. The opinion should always be accompanied
form indicated by objectivity, structure and style & by detailed reasoning and explanation for better
tone of writing which distinguishes it from other forms appreciation of the opinion. A characteristic structure
of communication. of a report helps a reader to use a report efficiently
and effectively.
3.2 Objectivity
3.4 Style
A report should be objective and not subjective with
any preconceptions or bias including bias favourable 3.4.1 A valuer may have all the technical expertise
to the clients. The information & data should be but it will be of no practical use if he is not able to
collected by carrying out rigorous and comprehensive present the information & data, its analysis and
(i) search of records (ii) inquiries and (iii) site opinion clearly. Clarity is one of the most important
inspection without any preconceptions or bias. The requirement of a valuation report which can only be
information & data collected should be analysed and achieved by careful attention to the style, tone, choice
evaluated objectively by application of professional of words & phrases and layout or arrangement of
expertise leading to sound conclusion and the report.
professional opinion. A report will then have
professional credibility generating confidence in the 3.4.2 A valuation report is an essay on technical
mind of a client or a reader. subject written in simple non-technical words. The
style of a report should therefore be that of an essay
3.3 Structure with simple words and simple, short & complete
sentences. Complex complicated & long, flowing or
3.3.1 Writing a report is preceeded by the process winding sentences which often create confusion,
of valuation, which includes (i) receiving instructions should be avoided. At the same time incomplete &
from a client (ii) inspection of relevant documents, disconnected sentences giving impression of short
site inspection and inquiries on & off the site for notes or specifications should be avoided.
collecting information & data (iii) analysing
information & data and (iv) arriving at conclusions. 3.4.3 The style adopted should be impersonal style
This process is reflected in the form or structure of a using passive voice, instead of active voice. Repeated
valuation report and in deciding logical & systematic use of "I", "my", "me", "we" is often annoying to a
sequence of the contents of a written report forming reader and may sound egoistic. A report written in
structure or framework of a report. A report should first person may also give impression of lack of
ideally stand as a document in itself. objectivity and may result in loss of interest on the
part a reader. Instead of saying "I inspected the
3.3.2 The final part of a report will comprise of property" it is better to say "the property was
drawing inferences from analysis of information & inspected". The use of 'simple past tense' however
data etc., resolving balance of alternative solutions, suggests that the action of inspection was completed
by using technical expertise and identifying possible long back in the past. It is therefore better to always
effects of such alternatives. This process leads to (i) use 'present perfect tense' and say: 'the property has
the conclusion and final opinion. (ii) recommendations been inspected' suggesting that the action has been
regarding actions to be taken for implementation of completed just before writing the report.
Seminar on Valuation of Properties - 2011 47

3.4.4 Language is developed by formal & informal 3.6.2 Clarity


education and continuous efforts for improvement.
It is therefore difficult to suggest a good style. A valuer Clarity is one of the most important qualities of a
needs to develop his own style by the continuous good report. Use of inappropriate & ambiguous
efforts to improve his command over the language words or improper constructionsof sentences often
and by study & analysis of well acknowledge reports results in lack of clarity. Choice & use of proper and
in the past. appropriate words conveying the exact and precise
meaning is therefore crucial for a good report.
3.5 Tone Whatever is to be stated in a report should be stated
clearly in simple, plain & straight forward words &
A valuer writing a report should primarily have sound sentences.
technical expertise and be considered as having
authority in the concerned field. A report should not 3.6.3 Use Simple, Short Words
however be written in a highly authoritative, dictating
or official tone as it may carry sense of arrogance Good English or any other language requires not only
annoying a reader. At the same time a report should good command over the language but also requires
not be written in too casual tone as it will not be rich vocabulary and clear understanding of the
effective, will give impression of lack of expertise precise meaning of words. The best effect of a report
and will not generate confidence in the mind of a is usually achieved by using simple, short and plain
client or a reader. A report should therefore be written words rather than by using long, pedantic words.
in the style of a Demi-Official (D.O.) letter which is Pompous expressions and long uncommon &
neither too authoritative nor too casual. It is also complicated words used for creating impression of
necessary to preserve certain dignity of tone so that scholarship often result in creating bad impression.
the information & data, its analysis, conclusion and Further with use of longer, bombastic words there is
opinion can be expressed effectively & convincingly. a tendency of exaggeration which needs to be
avoided in a report.
3.6 CHOICE OF WORDS
3.6.4 Words and Phrases To Be Avoided
3.6.1 Effective Use of Words
When a writer fails to appreciate correct meaning of
In a written report words and words alone are the words a report may become unnecessarily repetitive
basis of communication. When used properly they & verbose. Use of technical terms redundant words,
describe the object / property and actions of analysis repetitive words, hedge words, trite or common
& drawing conclusions etc correctly. When used place expressions, pointless phrases, false, elegant
poorly they lead to poor communication often & bombastic words which make a report
misleading the reader. In writing a valuation report unnecessarily verbose should be avoided.
every word should therefore be chosen with utmost
care so that the maximum & precise meaning is 'Quite', 'relatively', 'exceptionally' 'in my opinion' etc.
conveyed with minimum words. are some of the redundant words while 'basic
principles' 'fundamental principles' [Principles are
48 Seminar on Valuation of Properties - 2011

basic or fundamental] 'exactly identical' 'true facts' 4.2 Systematic and Methodical
etc. are examples of repetitive words. 'Appears',
'seems' 'possibly' etc. are hedge words. A valuer is The report should be systematic & methodical. This
not paid to say what it 'appears'. He is paid to say implies that before writing a report the exact order
what it 'is'. in which it is proposed to present information & data,
analysis, findings and conclusions should be decided.
"At your earliest conveience" [i.e. "soon"] & "Please Such order should follow a clear, definite, systematic
find enclosed herewith" [i.e. enclosed is/are] are some & logical sequence. The presentation of information
of the trite or common place expressions while & data, e.g. description of a property should not be
"Interesting to note" "needless to say" are a few shifting from one aspect to another at random, and
meaningless phrases. analysis & conclusion should follow logical sequence.

Further a client being a common man not familiar A report should be a logical sequence of simple, short
with technical terms, the general rule is that no & complete sentences. Each sentence should
technical terms, should be used in a report & communicate only a single element or point of
explanatory sentences should be used instead of information and each group of sentences i.e. a
technical terms. More often technical words can be paragraph or subparagraph should cover only one
substituted by simple words. In case where technical aspect of information & data or analysis etc. The
terminology is unavoidable it should be defined & information & data, its analysis and findings should
explained in simple words when first used. be given in a series of paragraphs, each paragraph
or sub paragraph being independent and yet logically
Care needs to be taken to ensure that adverbs are following the earlier paragraph or sub paragraph.
placed in correct position, usually as near to the verb
as possible. Do not say "The Property needs repairs 4.3 Continuity
badly" (may mean that repairs are to be carried out
badly) when it is intended to say "The property badly A report should be such that it can be read as a
needs repairs". complete and continuous reading from start to finish,
without any break in its reading. There should not
4.0 ESSENTIAL QUALITIES OF A GOOD be any jerkiness and there should not be any break
REPORT felt between any of the parts of a report. It is
therefore imperative that schedules, tables,
4.1 General statements, maps, plans, graphs, charts, photographs
etc. should not be included in the main body or text
A valuation report should be an essay on technical of a report but should be attached to the report as
subject written in a simple language for easy annexures. A reader who may be interested in any
understanding by a common man. It should create details can find them in the annexures while a reader
interest in the mind of a reader so that he is tempted who may be interested only in the findings or
to read it right from the beginning to the end in one observations of the annexures and who may not be
sitting like a good story or a good novel. In addition interested in details is not compelled to go through
to the quality regarding its form i.e. style, tone and the details.
choice of words, a valuation report should also have
certain specific qualities which are essential to make
it a good report.
Seminar on Valuation of Properties - 2011 49

4.4 Well set out 5.0 GENERAL STRUCTURE OF A REPORT

A report should be arranged in such a manner that a 5.1 General


client can easily refer to any particular part of a report
at any time without any difficulty. Headings given to 5.1.1 The basic or general structure or framework
a group of sub paragraphs covering a particular of a valuation report follows the system or process
broad aspect and sub headings given to a sub- of valuation. It therefore remains almost same with
paragraph or a group of sentences covering a variations in the details according to the purpose of
particular part of such broad aspect, will serve this valuation and according to the type of property to
purpose. Such headings and subheadings can be be valued.
given either in the margin or just to the right of margin
in a broad type so that they can be seen easily. 5.1.2. A valuer may be required to report on
valuation of different types of properties for various
It is also a good practice to give serial numbers to purposes. Every report on valuation of any specific
headings and subheadings. Headings may be property for any specific purpose should be built upon
numbered in single or two digits e.g. 2 or 3.1 while certain basic framework, so organised as to lead the
sub-headings can be numbered in two or three digits client or a reader smoothly from defining the problem
such as 2.2 or 3.1.4 etc. Such numbering should through descriptive information & data, analysis
invariably be given in the margin. thereof and eventually to the reasoning process
producing specific conclusion & opinion.
4.5 Definite & Concise
5.2 Definitions, Assumptions & Limitations,
A valuation report should be concise and definite. Method of Valuation
The length of a report should be such as may be
necessary to convincingly convey the information & 5.2.1 Definitions
data, analysis and opinion with reasoning. It should
be no longer & verbose and no shorter than It is not necessary to give definition of market value
required. There should be no ambiguity any where in a report. As a general rule where no definition of
in the report. The opinion and advice given in a report market value is given in a report the standard
must be definite and should be expressed in clear definition of market value held by the Supreme Court
and unambiguous words so that only the meaning of India and also adopted by the International
intended is drawn and any meaning other than what Valuation Standards Committee (I.V.S.C.) is
is intended cannot be drawn. presumed to have been adopted in a report. In cases
where value estimate is other than the standard
4.6 Legible definition it should necessarily be clearly defined in
a report.
The report, if handwritten, should be legible. In the
present era of computer typing not only this essential 5.2.2 Assumptions and Limitations
qualify of being legible is ensured but a report can
be neatly arranged & well set out without any No assumption should be made in a report, as far as
difficulty. possible, unless necessary by compulsion which is
rare.
50 Seminar on Valuation of Properties - 2011

Where a valuer is prevented or obstructed in 5. Supporting material i.e. Statement, Plans,


collecting & verifying information & data either due Graphs etc.
to physical circumstances or due to legal
circumstances the limitation should be stated in the 5.4 Introduction
report in the paragraphs relating to data collection
or recital of facts. 5.4.1 A short forwarding letter addressed to the
client and title of the report constitute introduction
There may be limiting conditions regarding use of a part of a report.
report. These should be clearly stated in the form of
caveats as also the limitations regarding leading 5.4.2 Forwarding Letter
expert evidence in the Court or Tribunal.
A report is usually sent to the client under a short
5.2.3 Method of Valuation forwarding letter written on the letter head of a
valuer. It should briefly refer to identification of the
A valuer cannot be compelled to adopt any particular property being valued and the purpose of valuation.
method of valuation. The method of valuation or The letter should include a concise statement of
approach to valuation should be evident from conclusion about value, recommendation & advice,
collection of data, its analysis and reasoning leading if any, and the date on which the value is estimated.
to conclusion & opinion. It is therefore redundant to It may point out that the value estimated is subject
make specific mention regarding method of valuation to certain conditions, if any. It may also enclose the
adopted. In case the valuer makes specific mention bill of fees.
of method of valuation adopted he is required to give
reasons for the same. 5.4.3 Title of Report

Further it is not necessary to estimate fair market A report should begin with the title comprising of (i)
value of a property by adopting two or three identification of the property being valued i.e. C.T.S.
approaches to valuation unless the valuer himself No. or F.P. No. & TP Scheme No. locality, city /
desires to have cross - check. In any case, however, town etc. (ii) the purpose of valuation and (iii) the
temptation to arrive at an average value should always date of valuation. The purpose of valuation & the
be scrupulously avoided. date of valuation both are very important and the
title will be incomplete without any of these three
5.3 Parts of a Valuation Report elements.

5.3.1 General 5.5 Data Collection and Recital of Facts

Every report should comprise of five parts, viz. 5.5.1 General


1. Introduction, which includes a forwarding letter
and the title of a report. A valuation report should give sufficient information
2. Recital of facts, after collection of information about the property to be valued and other data
and data. relevant for estimating its value so as to be of
3. Analysis of information & data assistance to a client or to those for whom a report
4. Conclusion, opinion and advice if needed. is intended. The type of information and data to be
Seminar on Valuation of Properties - 2011 51

collected, quantitatively & qualitatively, depends on 5.5.4 Date of Valuation


and is rather determined by (i) the instructions given
by the client (ii) method of valuation and (iii) the date The material date of valuation sets the time limit since
of valuation. the information & data to be collected should be as
on the date of valuation or in the recent past. The
5.5.2 The Instructions postdated information & data including instances of
sale & rentals which have taken place after the
Instructions given by the client (a) identify the material date of valuation are irrelevant and are not
property and interest to be valued (b) define the considered as admissible evidence.
purpose of valuation (c) state the date of valuation
and (d) suggest the scope and limitations, if any. 5.5.5 Factual Data

The information & data to be collected should be in It is the duty of a valuer to give all relevant & true
respect of (i) details of the property to be valued facts, information & data to the client. He must always
including the interest to be valued and (ii) instances be prepared to prove and defend his evidence about
of sales & rentals in respect of properties in the the information & data in the Court of Law whenever
vicinity & comparable to the property to be valued needed. He must therefore himself ascertain,
with details thereof. personally verify and get himself satisfied that the
information & data given in his report is true &
The purpose of valuation & the type of property to correct. The information & data given in a report
be valued determines the type or nature as also the should be first hand evidence and not hearsay
extent of details of the information and data required evidence.
to be collected.
5.6 Sources of Information and Data
5.5.3 Method of Valuation
5.6.1 General
The method of valuation adopted also determines
the nature of data to be collected in addition to the Information & data can be collected from various
data regarding the property to be valued including sources. These sources mainly include
its location.
1. Record of Govt. and Municipal Offices - City
The cost approach requires data regarding the Survey office or talathi's office can provide
prevailing schedule of rates of construction and recent information about survey No., area, tenure, title
instances of sale of comparable open lands in the ownership & occupier, encumbrances if any as
nearby areas. The market approach to valuation will recorded on P. R. cards or VII / XII extracts
require information & data regarding transactions of and city survey maps or village maps etc.
sale of similar and comparable properties nearby Instances of sale & now instances of leases &
while Income approach will require information & rentals can be obtained from Index II maintained
data regarding instances of rentals as well as instances in the Registrar's or Sub-Registrar's office.
of sale in respect of comparable properties in the Information about planning proposals, D.C.
locality for the study of the local Real Estate Market. Rules & building bye laws, & F.S.I. permissible,
development / building permission granted or
52 Seminar on Valuation of Properties - 2011

otherwise, part plans of development plans, & conditions of occupancy etc. and easement rights
municipal taxes etc. can be obtained from & encumbrances etc. 2) Land : area, shape, level,
municipal offices. foundations etc. 3) access to property 4) Location
& situation with reference to social amenities &
2. Publications and Economic Journals provide facilities, transport facilities etc. 5) Type of locality,
information about (a)various economic indices social & economic status of local residents 6) Utility
like Capital Market Indices i.e. B.S.E., N.S.E. services provided in the Locality etc.
or Nifty indices, foreign exchange rates, inflation
indices, rates of interest on Central & State During site inspection observations should be made
Govt. securities, fixed deposits, bank lending about the structures in the property i.e. main &
rates, G.D.P. growth rate etc. Similarly subsidiary buildings, if any, including engineering &
information of per capita income (P.C.I.), architectural aspects etc. in a systematic order, utility
domestic savings & its investment in various services provided to the property itself, common
forms including investment in Real Estate, Net facilities & services provided in the property, state
Fixed Capital Formation (N.F.C.F.) in economic of maintenance & repairs etc. During site inspection
sectors especially in Construction sector (supply economic aspects of the property should also be
side) & Real Estate sector (demand side) can inquired into.
be obtained from such economic journals
Instances of rentals should be collected and verified
including R.B.I. journals & journals of Central
by inquiries during site inspection especially
& State Govt's. Bureau of Economics &
regarding rate of rent and the date of letting.
Statistics, C.M.I.E. journals etc.
Instances of sale collected from Registrar / Sub-
Registrar office should also be verified by inquiries
Census books provide information regarding
with vendors & purchasers etc., particularly about
population, growth & changes in population,
the price actually paid including unaccounted money
housing conditions, changes & growth in
which requires to be elicited by using skill of inquiry
occupancy i.e. employment patterns etc. which
and winning confidence of the person being
give some idea about property values, in a town
interviewed. Enquiry should also be made regarding
/ city.
preferences, priorities & importance attached by the
local residents to various actions like comparison of
5.6.2 Site Inspection
the properties let out, sold & property to be valued.

The third and most important sources of data & A valuer should also ensure that all the original field
information is site inspection. The principle 'Inspect notes are preserved as record for further reference
before Value' should therefore be strictly, followed. if & when required.
A valuer can observe many important facts and gather
information & data by careful & rigourous site 5.7 Analysis
inspection and inquiries on site. He has therefore to
do perfect homework and decide a definite 5.7.1.. Analysis of information & data collected
systematic order for inspection of site. At the time of involves
site inspection he can collect information about (1)
Identification of property i.e. its survey No or CTS 1) Estimating fair market rent or fair market value
No. etc., its boundaries , owner & occupier etc. terms or land value.
Seminar on Valuation of Properties - 2011 53

2) Finding out net annual income, or depreciated properties let out with properties sold and the
cost of construction and property to be valued is the most crucial & integral
3) Estimating appropriate rate of interest for part of any valuation report. A sales plan showing
capitalization. the instances of rentals (usually in green) instances
of sale (usually in blue) and property to be valued
5.7.2 Estimating Fair Market Rent of Fair (usually in red) in terms of rate of rent, area let out
Market Value or Land Value or sold, T.W.S. & date of letting / sale / valuation is
much helpful in estimating fair rate of rent or fair
Estimating fair market rent or fair market value or market value or fair land value.
fair land value involve comparison of properties let
out, sold & property to be valued. Such comparison 5.7.3 Net Annual Income
can be done by directly comparing the properties
taking into consideration various factors of Gross annual income can then be estimated by
comparison. Such comparison often becomes multiplying rate of rent per sq.m. per month by area
subjective. In order to eliminate the element of of the property & 12 months. Deducting annual
subjectivity and compare the properties as outgoings from gross annual income gives net annual
objectively as possible it is advisable to adopt Total income of the property sold or property to be valued.
Weightage System for comparison of properties. As
many as 60 or more factors of comparison grouped 5.7.4 Rate of Interest for Capitalisation
in seven broad categories viz. (i) Land (ii) Location
(iii) Buildings (iv) Utility Services (v) Social (vi) In Income approach to valuation Y.P. is estimated
Economic and (vii) Legal alongwith their 5 levels can by dividing price actually paid by net annual income.
be identified. Appropriate weightages are to be In case of properties having future life of 60 years or
assigned to these factors (as may be applicable in a more remunerative rate of interest is equal to 100 /
particular case) and their levels according to priorities Y.P. In case of properties having future life less than
& importance given by local residents. Multiplication 60 years remunerative rate of interest is found out
of weightage assigned to a factor of comparison & with the help of Valuation tables considering the
weightage assigned to its level gives weightage score accumulative rate of interest at 2 1/2% to 3% and
of that particular factor. Total of such weightage the future life of the property sold. The appropriate
scores of all the factors of comparison as may be rate of interest for capitalizing the estimated net
applicable in a particular case gives the Total annual income of the property to be valued as on the
Weightage Score (T.W.S.) of a property. date of valuation, is estimated by comparing rates of
Comparison of properties on the basis of such interest yielded by the properties sold with the
T.W.S. tends to be objective depending on economic indices especially B.S.E. & N.S.E. indices
weightages being appropriate & nearer to reality. and foreign exchange rates etc.

Properties can then be compared on the basis of (i) 5.8 CONCLUSION


Total Weightage Score (ii) area let out or sold and
(iii) the date i.e. the date of letting, or the date of Every valuation report should conclude with opinion
sale and the material date of valuation and fair market and advice if called for. The opinion and advice must
rent or fair market value or fair land value can be be definite & clear and must be expressed in
estimated. Discussion regarding comparison of unambiguous words.
54 Seminar on Valuation of Properties - 2011

5.9 SUPPORTING MATERIAL description of a property will vary according to the


purpose of valuation.
Valuation of properties is becoming increasingly
subject to greater scrutiny and analysis. It has 6.1.2 Though the details of information & data about
therefore become necessary to collect comparatively the property to be valued will vary according to the
large quantity of information & data. A text alone is purpose of valuation and the type of property, the
inadequate for presentation & analysis of such data details of the information & data required for
which contributes to formulation of professional estimating fair rent or fair market value i.e. information
opinion. The narrative part of the report is therefore & data regarding instances of rentals & sale will
supported by a wide range of annexures such as however be more or less the same irrespective of
statements, schedules, tables, maps, plans, graphs, the purpose of valuation and will vary according to
diagrams, charts, and photographs etc. which are the basis and the method of valuation to be adopted
appended to the main write-up of a report. Graphs, for estimating fair market value as per requirements
bar charts, plans, photographs etc. creating visual of a particular purpose and as per requirements of
impact are helpful in achieving effective the provisions of the relevant legislation for taxation
communication. etc.

Each of the supporting material must have the same 6.2 Purpose of Valuation
title as that of the main report as heading and must
have sub-heading indicating the contents of the 6.2.1 Reports on valuation of properties for various
annexure. All the annexures should be arranged in purposes can generally be classified in three
Logical sequence with annexure number at the right categories on the basis of details of information &
hand top corner. The annexures which need folding, data required to be collected, mainly for description
should be folded in such a manner that the Annexure of the property.
number, heading & sub-heading and job number
should be seen easily & clearly. 6.2.2 Purposes requiring all the Details

It is advisible to write, at the end of each of the a) Purchase for self occupation or taking on lease:
annexures, statements of findings or observations. Since the purchaser / lessee intends to occupy
Such statements arranged in logical sequence may the property for his own use he would like to
make writing of the narrative text of the report easier. know all the details.
b) Insurance: All details are required for claiming
6.0 DATA & INFORMATION AND ANALYSIS full damages, when insured for fair market value.
ACCORDING TO PURPOSE OF VALUATION c) Land Acquisition: For claiming compensation &
damages for financial reinstatement.
6.1 General
6.2.3 Purposes requiring fair or reasonable
6.1.1 A valuation report mainly consists of two major details, but not in minute details
parts viz (1) Description of the property to be valued
& (2) Analysis of information & data leading to a) Purchase as an investment. (b) Sale or letting on
conclusion. The general framework of every valuation lease (c) Mortgage (d) Management of Estate /
report will be the same but the details regarding Property.
Seminar on Valuation of Properties - 2011 55

In these cases the purchaser or owner or lessor is 7.0 DATA AND INFORMATION REQUIRED
not intending to occupy the property & hence minute ACCORDING TO THE TYPE OF PROPERTY
details are not required. However since the purpose
involves financial aspect, all reasonable details having 7.1 General
effect on income fetching capacity of a property are
required. 7.1.1 A valuer may be called upon to report on
valuation of various types of properties for different
6.2.4 Purposes where Brief Description of the purposes. While the extent of details of information
Property is sufficient & data vary according to the purpose of valuation
and the analysis of information & data varies
a) Rating or Municipal Taxation (b) Income Tax according to the method of valuation adopted, the
c) Estate Duty & Wealth Tax, d) Capital Gains Tax. aspects relating to which the information & data is
The owner knows his property and the taxation required is to be given in a valuation report vary
authority is not interested to know the details. according to the type of property to be valued.
Valuation of agricultural properties will require
6.3 Valuation for Mortgage information & data regarding certain specific aspects
like quality of soil etc. while valuation of residential
Valuers are often instructed by financial institutions, properties will require information & data on different
including banks, to report on valuation of properties specific aspects. Similarly aspects about which
for purpose of mortgage and hence it requires special information & data is required for valuation of
mention. In addition to reasonable details of commercial properties will be different than the
information & data regarding usual aspects the nature aspects to be considered in valuing industrial
of mortgage necessitates consideration of some properties.
special points in a valuation report. Since the
property being mortgaged has to be considered as It is necessary for a valuer to develop his own logic
security for loan & interest the possibility of the & reasoning regarding the aspects that would be
mortgagor failing to pay interest and repay the loan relevant or irrelevant for valuation of different types
and the client mortgagee eventually being required of properties and regarding the extent or degree of
to take possession of the property needs to be taken details of information & data required to be collected
into consideration. In such cases it needs to be in respect of such aspects according to the purpose
ensured that the actual & realizable net annual income of valuation.
from the property is sufficient to cover annual interest
as well as annual installment for repayment of 7.2 Residential Properties
principal loan amount. Further the fair market value
may also give fair idea about the value realizable in 7.2.1 Residential properties are expected to provide
distress sale which however can hardly be estimated. utility or satisfaction for the occupants seeking to
A valuer should also point out to the client the defects maximize utility for them. The important aspect
in a property as security and possibility of sudden & therefore is the type & quality of residential
violent depreciation in its value. Where there is accommodation itself which include (i) type & quality
possibility of slow but steady depreciation of value of construction, structural safety & specifications for
periodic revaluation should be advised. material & workmanship (ii) convenient arrangement
of rooms, rooms sizes, light & ventilation etc. (iii)
56 Seminar on Valuation of Properties - 2011

type of quality of utility services & (iv) amenities & out and occupied by several occupants, in addition
facilities provided within the property & (v) the state to usual aspects, the schedule of accommodation
of maintenance & repairs. showing (i) areas let out with terms & conditions of
lettings, (ii) existing rents (iii) unexpired period of
7.2.2 The next important aspect is the locality, its lease (iv) time of renewal & revision of rents
type & quality of environment in general i.e. type of indicating potential for increase in net income etc. is
residents, their social & economic status, habits very important.
especially regarding cleanliness, type & quality of
access road and general surroundings. 7.5 Special Properties

7.2.3 The situation with reference to social facilities Special properties like residential hotels, Cinemas
& amenities is also important but secondary to the & Theatres, Petrol Pumps etc. also require profit
first two aspects. maximizing locations and the important aspects to
be considered in valuation report are almost the same
7.2.4 The importance given to these aspects will as those in case of commercial properties with
vary in respect of residential bungalows & flats. additional aspect of various other sources of income
in addition to the main source of income.
7.3 Commercial Properties
7.6 Industrial Properties
The main consideration in commercial or business
accommodation is maximization of profits. 7.6.1 Locational aspects have strong effect on
Maximisation of profits depend on accessibility and profitability i.e. profit earning capacity of industrial
location which are therefore most important aspects concerns. In a report on valuation of industrial
in valuation of commercial properties. It is therefore properties the following seven locational aspects
essential to describe, in detail, the situation with acquire importance.
regard to central business district (C.B.D.) & other 1. Nearness to the sources of raw materials and
commercial areas, other shops & their types, focal nearness to market for finished products -
points, main road on which the property has frontage reducing cost of production including transport
with vehicular & pedestrian traffic on such road etc. costs.
The area of accommodation (size), shape, frontage 2. Type & quality of transport facilities for raw
to depth ratio, arrangement of space, parking materials and finished products, loading &
facilities, provision for window shopping etc. in case unloading facilities etc.
of shops, are also important aspects. The type &
3. Accessibility by road and / or rail.
quality of construction, utility services especially
electric supply are also important but secondary to 4. Availability & dependability of infrastructure
location & accommodation itself. services i.e. electric supply, water supply,
facilities for disposal of effluents & waste
7.4 Properties in Single Ownership with materials, facilities for prevention of air & water
Several Occupants pollution etc.
5. Availability of labour, especially skilled & semi
In case of residential & commercial properties in skilled labour, tendancies & behavior of labour
single ownership with flats, shops and / or offices let & possibilities of labour trouble.
Seminar on Valuation of Properties - 2011 57

6. Availability of complementary services like fire 2. A factory building built for a particular process
fighting, banking, communication facilities etc. of production is hardly suitable for other
7. Surplus assets like vacant land, not utilized in processes & products unless suitable changes
production at present. are made, which may be rare & costly.

7.6.2 Reports of Specialists 3. It is difficult to find a purchaser who may use


the factory buildings as they are, with minor
In practice one valuer is not sufficiently an expert in alterations. Considerable time is lapsed in finding
various aspects of valuation of industrial buildings to such a purchaser, if any, and in the mean time
make a complete report. This is especially true in factory buildings become an encumbrance on
respect of large or uncommon industries with land.
electricity generating plant or powerhouse or
industries with elaborate water supply system, a boiler 8.0 CONSEQUENCES OF A REPORT
house with large number of boilers etc. In such cases,
the special parts of valuation work are given to the 8.1. General
specialist in the respective fields as sub contracts.
The task of a valuer in such cases is to incorporate 8.1.1 The role of a valuer, many a time, does not
reports of these specialists in his own general report end with sending to the client, his report on valuation
on valuation of such industrial buildings. He should of a property. On receipt of a valuation report the
however never attempt to paraphrase such reports client proceeds to take further action regarding
or make precise thereof and absorb them in his management of his property. A valuer may then be
report. He should however help the client by called upon to participate in meetings with concerned
including, in simple words, the general conclusions parties or authorities & present his report. He may
& recommendations given by the specialists which be required to negotiate with other parties alongwith
may be given in technical language. The reports of his client or representing his client. A valuer may also
specialists must invariably be appended to the be required to lead evidence in the Court or Tribunal
general report as they have been received from the as expert. Sometimes if the client is dissatisfied with
specialist experts, without any changes. a valuation report and / or suffers loss due to actions
taken on the basis of his report, a valuer may have
Same is the case of reports received from to face charges of professional negligence with
horticulturist in respect of valuation of orchards. liabilities for damages. It is therefore necessary for a
valuer to know & bear in mind the possible
7.6.3 Mortgage of Industrial Building consequences of his report while writing a report.
This will, enable him to take all the necessary care
Industrial buildings are generally considered as poor (i) in collecting detail information & data in respect
security for mortgage mainly for the following of all the aspects relevant for valuation of a particular
reasons. type of property for a particular purpose (ii) in
1. Obsolescence : A product as also production analyzing the same (iii) in drawing logical conclusio
technique & process may become obsolete & ns and (iv) in giving his opinion & recommendations.
outdated in the era of fast developing technology. It is necessary for a valuer to equip himself for dealing
The machinery may also become outdated with with any of the consequences of his report.
almost no resale value or may have scrap value.
58 Seminar on Valuation of Properties - 2011

8.2 Meetings 8.3.4 One of the skillful and shrewd tactics employed
by the opponents is to create confusion. When there
A meeting can be termed as multi-party negotiations is confusion in the mind of a valuer he becomes less
attended by more than two parties and their advisors certain about his facts & data making his case weak.
etc. A valuer may be required to present his valuation It is therefore necessary for a valuer to clearly know
report in such a meeting and participate in the strengths & weaknesses of his case and that he
discussions. The participants & valuers attending a should be absolutely certain about his facts for which
meeting are expected to have carefully studied the the information & data should be first hand and
notes on the agenda circulated earlier as also all the personally verified by him.
relevant aspects related to the subjects on the agenda
before expressing their views. Presentation of views 8.3.5 Care should be taken to ensure that the
and discussions should be to the point and brief. It is discussions are calculated, measured and well
also necessary to express views with due courtesy reasoned. The style of conducting discussions should
and at the same time, with firmness. Interests should be polite, firm and confident. Professional differences
be advocated in all fairness. of opinion, if any, should not be allowed to reach the
level of arguments and nothing should be done which
8.3 Negotiations may result in losing opponent's goodwill and his
interest in the mutual purpose. A valuer may put
8.3.1 Negotiations is a meeting usually between two
forward and indicate disagreement but should always
or more individuals in respect of a particular issue.
avoid confrontation.
Negotiations means to communicate or to meet for
the purpose of resolving some matter by mutual
8.4 Expert Evidence
agreement and to clear and mutually settle some
differences. A valuer may be called upon to represent
8.4.1 General
his client. In such a case his implied authority is limited
to making statements about the property and to
A valuer may be required to give evidence as expert
prepare the stage for agreement & contract. Unless
i.e. to lead expert evidence in the Court or Tribunal
expressly authorized a valuer cannot make binding
etc in several cases of valuation such as those in
contract or any transaction in respect of his clients
respect of claims for compensation in acquisition of
property.
land, claims in cases of insurance, estimation of fair
8.3.2 Prior to starting of negotiations a valuer must rent or fair market value for municipal or Govt.
be fully briefed in all aspects about the property, the taxation and other purposes. Generalisation about a
client & his intentions. Thereafter some tactics have valuer's report or his conduct in the Court of law or
to be decided by the client & his valuer. This sets Tribunal etc is therefore difficult.
the background and the framework for negotiations.
8.4.2 Basic Rules for Leading Expert Evidence
8.3.3 An important rule in negotiations is that a valuer
should be fully aware of the strengths and weaknesses In leading expert evidence it is most important to
in his own case and use them to achieve maximum observe three basic rules.
benefit. It is also advantageous to find out and know 1. The foremost important rule is that "Honesty is
the strengths and weaknesses in the opponents case the best Policy", especially in examination in chief
and use them to the advantages of his client. and in cross - examination. If some thing is not
Seminar on Valuation of Properties - 2011 59

done, be honest and say so, invariably giving 1. No forwarding letter is required in the case of
reason why it has not been done, otherwise it valuation report for leading expert evidence since
may amount to negligence losing value of expert it is directly submitted to the Court during the
evidence. proceedings.

2. Evidence by an expert is evidence of opinion & 2. The report for leading expert evidence begins
is distinctly different from the evidence of facts. with professional qualifications and experience
In giving evidence of facts a witness is required of a valuer. It is broken down into numbered
or rather compelled to give answers in 'yes' or paragraphs with the intention to indicate those
'no'. An expert deposing his 'opinion' evidence being answers to questions put to him in the
has privilege of giving answers with reasoning examination in chief through which a valuer is
and cannot be compelled to answer questions led. Such informative paragraphs are not
only in 'yes' or 'no' without explaining the required in a normal valuation report.
reasoning since any opinion without reasoning
will have no value. It is advisible that the expert 3. The main & most important difference in the two
should scrupulously insist on exercising this reports is that the normal valuation report given
privilege while leading expert evidence. to the client can be based on the information,
data and instances of rentals & sale collected &
3. It is the duty of a valuer leading expert evidence verified by the valuer and considered to be true,
to assist the Court or Tribunal in properly correct & genuine while report submitted by the
interpreting the evidence regarding value valuer while leading expert evidence is required
produced in the Court or Tribunal proceedings to be based only on the information, data and
and thereby in arriving at the just and fair value. transactions of sale duly proved in the Court.
As an expert his duty is not to defend his client The expert valuer is therefore usually leads his
or anybody and though his fees may be paid by evidence after factual evidence is over.
client, his duty is to assist the Court or Tribunal
in arriving at the just & fair value. Once a valuer 8.4.4 Instances of sale can be proved in the
enters the witness box and is giving his opinion Court in three ways viz.
to the Court he must give his honest opinion.
The valuer will deceive himself and also the Court 1. By producing original registered document or
if he fails to give his honest professional opinion. certified true copies thereof & by examining the
He therefore needs to distinguish between his vendors or purchasers or a persons having direct
report in leading expert evidence and his normal & personal knowledge of the transaction. Mere
valuation report given to his client when he production of registered document or certified
legitimately took a view for achieving best results true copy therof is not considered as enough
for his client. proof of execution of the transaction.

8.4.3 Normal Valuation Report & Report for 2. By agreement by the parties to the litigation,
leading Expert Evidence accepting the sale transaction to be true &
correct.
A valuation report for leading expert evidence differs
from normal valuation report in the following respects. 3. By personal and direct involvement or
60 Seminar on Valuation of Properties - 2011

experience of expert valuer acting for one of the 8.5.3 Legal Action
parties to transaction when such expert valuer
can be examined in the Court. Legal action for negligence can be taken under the
Law of Contract or under the Law of Tort. An action
8.5 Professional Negligence in tort can have two advantages. Firstly tort permits
greater extent of damages to be considered.
8.5.1 The action or inaction on the part of a valuer Secondly the Limitation period which is normally six
in the valuation work or in the management of years, starts in contract from the moment of breach
individual interests in properties / estate can be whereas in tort it starts from the moment when
subject to criticism by clients and / or third parties damage could reasonably have been discovered and
on account of several types of misconduct. known.
Intentional or deliberate mistake done causing harm
to interest in a property is fraud, which would be a 8.5.4 The Standard of Care
criminal matter. The client who may be dissatisfied
with the services provided by a professional valuer The rule about the standard of care is that "the test
will be able to sue the valuer for damages on the is the standard of the ordinary skilled person
basis of contract or under statute law. A valuer may exercising and not that of a professional expert
have been negligent in common law when the client claiming to have specialized skill". A professional
as well as third party will have remedy in damages. valuer need not possess the highest expert skill; it is
sufficient if he exercises the ordinary skill of an
8.5.2 Basic Principle ordinary competent valuer. His knowledge must
however extend to a reasonable and up to date grasp
Valuation is a matter of opinion. A valuer is certainly of the principles of valuation including law. If however
not guilty of negligence only because his valuation a valuer claims himself to be a specialist and is
figure does not tally with reality in the real estate consulted as such then the standard of care of
market. The basic and the first principle in specialist is expected.
professional negligence is that 'negligence is not
judged by the figure or the result alone but is judged Further irrespective of whether the work is
by the way or route by which the figure or the undertaken charging no fees or reduced fees, the
conclusions have been reached. standard of care remains the same as that of a
competent professional irrespective of the amount
A valuer cannot be compelled to adopt a particular of fees.
method of valuation. The important factor in deciding
professional negligence is that whether a valuer has 8.5.5 Measure of Damages
correctly followed the method of valuation he has
adopted and whether he has taken or not taken all In considering the correct approach to damages, the
the necessary care that is required to be taken courts have concentrated on three main issues. Firstly
according to the method adopted, in collecting the the relative merits of a cost approach and value or
information & data, in analyzing it & in reaching at income approach as the measure of damages.
the conclusion. Secondly the extent to which the plaintiff could have
or should have mitigated his loss. Thirdly the date at
Seminar on Valuation of Properties - 2011 61

which the damage should be estimated or calculated. profession and should be true to his own conscience
while performing & completing the task & also while
The principle objective underlying the rules for writing a valuation report. A valuation report written
assessment of damages or compensation is to place by a valuer exercising & adhering to these essential
the plaintiff / owner in the same financial position, as prerequisites will have duly observed the requirements
far as possible, as he would have been, had he not of the International Valuation Standard 105: Valuation
suffered the loss or deprived of his property. Reporting (2010)

9.0 CONCLUSION In report writing it is basically presumed that a valuer


satisfies the above essential prerequisites. The thrust
Firstly it is basically necessary that a valuer writing a is therefore given on developing good and effective
valuation report has already acquired necessary communicative skill. Once an inquisitive & analytical
technical knowledge and experience in the field of mind is committed, with enthusiasm, to the task
valuation and has developed sound analytical mind. coupled with good & effective communicative skill,
Secondly a valuer must have honesty, integrity to better quality report is bound to be obtained.
62 Seminar on Valuation of Properties - 2011

LENDERS' INDEPENDENT ENGINEER

1.0 SCOPE OF WORK 3.0 DRAWDOWN SCHEDULE

The assignment would broadly covers: The Lenders' Independent Engineer will review the
drawdown schedule and comment on whether each
Y Verifying the project parameters vis--vis the cash draw down amount is consistent with the Project
approval / notification from BOA / SEZ rules / schedule and Contract requirements.
norms, sale deed and other project-related
documents, verifying approvals for building plans 4.0 REVIEW OF PERMITS AND LICENSES
and other requisite permissions / approvals
Y Comment on the debt drawal schedule and its The Lenders' Independent Engineer will review the
consistency with the project requirements permitting schedule and all available permits or permit
applications.
Y Monitor project implementation
Y Verification of the capital cost incurred vis-- Independent Engineer will
vis estimated costs
Y Assess the capability of the Project as designed
Y Provide regular (or as and when desired by the
to meet the technical requirements specified in
lenders) project review reports
the Project's permits licenses
2.0 INITIAL REPORT Y Establish contact with the appropriate
environmental or energy regulatory agencies for
Initial Report shall consist of following: the purpose of independently identifying and
determining the current status of the major
Y Current status of project permits and licenses to construct and operate
the Project
Y Review of scheme (Design & Construction) with
brief Y Identify what major permits have not been
obtained and comment on the likelihood that they
Y Review of construction schedule with respect
may or may not be able to be obtained in a timely
to phasing, sequencing of construction activities
manner to support the Project schedule
and site logistics
Y Comments / suggestion on construction 5.0 ENVIRONMENTAL IMPACT
methodology ASSESSMENT
Y Review of project budget and cost incurred till
date The Lenders' Independent Engineer will review the
permits and licenses required for the construction
Seminar on Valuation of Properties - 2011 63

and operation of the facility. The typical Engineer will issue a communication covering
environmental issue to be addressed is provisions their expert recommendation on the request for
for disposal of wastes. draw of loan funds.

6.0 CONSTRUCTION MONITORING 7.0 PERIODICAL REVIEW REPORT

The project will be monitored on a regular monthly The periodical review report shall comprise of:
basis through successful construction completion,
which will include: Executive Summary

Y Reviewing quality control reports and field Overview of the status of the project
laboratory test reports Indicating critical activities
Y Prepare and submit quarterly written reports to Highlighting major variations from the business
lenders plan, schedule, costing and cash flow
Y Monitoring of the capital cost incurred and certify Progress photographs
drawdown request in line with budgeted capital
expenditure requirement Regulatory Compliance
Y Assessment of variation in orders & contracts
and its implication on the technical aspects of Update on approvals and their impact, if any
the project and capital cost estimates Health and Safety statement of accident on site
Y Regular reporting of physical progress in
construction up to commencement of Schedule
commercial operation in line with the capacity
build-up plan and schedule for implementation. Summary of progress to date, Milestones, Critical
Y Highlight / report deviations from field map / path
approved drawings to the Company and the Comparison with base schedule
corrective action need to be initiated thereat Highlighting delays and their causes as well as their
within stipulated time frame, if any impact on overall completion
Y Reporting and certifying physical performance
testing and acceptance provisions Costing / Budget
Y Monitoring and certifying compliance
Summary of expenditure to date
Y After every quarterly visit the Lenders'
Independent Engineer would prepare a progress Review of budget with respect to construction
report outlining all activities completed and those practices. We shall compare the budget with the data
scheduled for the coming visit and an available with us from our various projects and any
independent assessment of the status. On major inconsistency shall be reported
conclusion of the site visit the Independent
64 Seminar on Valuation of Properties - 2011

Comparison with Benchmark Cost Performance Testing


Statement on extension of times / claims / disputes,
etc. 8.0 COMPLETION REPORT

Quality Completion Report to include:


Y Facility design versus actual installation
Comments / suggestion on quality of work Y Compliance with regulatory requirements

General Y Confirmation that required governmental permits


and licenses have been obtained for construction
Review of change order records and operation

Random review of daily site records Y Certification of Performance Test Results

Review of Site Safety/HSE Status report


Seminar on Valuation of Properties - 2011 65

PROBLEMS FACED IN VALUATION BY PRACTITIONERS

Mr. Kirit Budhbhatti

DOCUMENTATION Y If plans / approvals are not provided then it is


difficult to comment on the legality of the
Property related DOCUMENTs play a major structure.
role in the valuation of a property.
Y At times the drawings which are shown in the
agreement do not tally with the approved plans
 Asking for related documents for valuation of
and there are may be major deviations between
Y Residential Flat the two.

Y Open Land
If the said property is occupied by person other
Y Land and Building than the owner, it is difficult to ascertain if he is
a licensee or a tenant protected under Rent
Y Industrial Properties
Control Act or is there any dispute in the said
Y Commercials offices / Retail Shops property or he is an illegal occupier.

Y Owner occupied/rented / tenanted/ 2. Land / Industrial Properties


leased / licensee

 Unfortunately this part is mostly neglected by Y For the above valuations if agreement copies are
bankers saying that please provide us with the not provided, the valuer is not able to understand
value of the property and documents will be seen the rights of the owners in the said property.
by our legal cell . Y It is observed that the most of the times industrial
 Implications of providing partial documents are land is leased or may be acquired under Land
very serious. A few examples are given in the Acquisition Act, documents related to the same
next slide to show how the valuation and the if not available can have a major impact on the
security being mortgage gets affected in case of valuation.
inadequate documents. Y If plans / approvals are not provided then it is
difficult to comment on the legality of the
1. Flat Valuations / Commercial Offices / Shops structure.

Y For the above valuations if agreement copies are Y At times the drawings which are shown in the
not provided, the valuer is not able to understand agreement do not tally with the approved plans
the rights of the interested parties in the said and there are may be major deviations between
property. the two.

Kirit P. Budhbhatti, Practising Valuer RE and PM - Mumbai, Founder Chairman (Retd.) International Plant
and Machinery Valuation Conference Committee, Founder Secretary, Centre For Valuation Studies, Research
and Training
66 Seminar on Valuation of Properties - 2011

Y If the said property is occupied by person other 3) If conditions of N.A. are not fulfilled by the
than the owner, it is difficult to ascertain if he is owner, it becomes very difficult to identify the
a licensee or a tenant protected under Rent plot.
Control Act or is there any dispute in the said
4) In case of under construction properties where
property or he is an illegal occupier.
the building is partially complete and valuation
Y At times it observed that the land is agricultural is asked for flats which are yet to be constructed,
and industrial activities are still functioning on the valuer has to inadvertently believe that the
the same, in the absence of N.A. Order / PR flat will be constructed as per agreement details
Card latest / Collector Order on change of land or brochure/ plan.
use if not available will have a huge impact on
At times Plans are approved only to limited floors
the value of land.
and flats are sold beyond the same and it is informed
that the plans will be amended by loading TDR
Observation (Transfer of Development Rights). So practically the
property under valuation doesnt even exist.
This will just give you a glimpse of the problems faced
by the valuer in absence of the sufficient documents
 Carrying out the site visit
and when reports are prepared by valuer based on
partial documents provided by the bank, it is
1. Many a times bank insists on valuations of
needless to say that the implications of the same are
properties which the owners or the occupiers
seen by the banks when the accounts becomes NPA.
are unwilling to co-operate.
Hence, it is very essential to provide all the required
documents as asked by the valuer to give a correct
 Collection of data at site
opinion.

1. At times the owners are unwilling to handover


HALF KNOWLEDGE IS DANGEROUS
documents in case of any observations.
FIELD / SITE PROBLEMS
 Cooperation at site
Site inspection plays a major role in valuing a
1. Many a times during the site inspection, the
property
owners or the occupier of the property do not
part with any information if asked.
 Verification of site address:
2. Co-operation at site is very essential specially
1) At times it becomes practically impossible to in case of valuation of movable assets as
verify a site mostly lands when only Survey Nos. identification of movable assets is difficult in the
and CTS Nos. are mentioned in the documents. absence of proper serial no. and linkage with
the invoices.
2) Many a times there are no boundaries or even
markings or any sign board at site.
Seminar on Valuation of Properties - 2011 67

3. At times it is observed that the movable assets  Most of the empanelment letter that bank have
are shifted from one location to another location a conditions of fee limitations and removal from
without informing the Bank and linking the panel without giving any reasons, valuers are well
correct asset with the invoice, this becomes a wishers of the banks and work to provide an
tedious task in the absence of co-operation from independent opinion to the bank in the interest
site personnel. of the bank. Hence, they deserve a proper
hearing and should be treated as a professional.
OTHER PRACTICAL ISSUES
 So to conclude Bankers and Valuers should
mutually co-operate on the above issues so that
 At times when the valuer comments or puts his
they can
observations or any adverse comments after site
inspection, it is treated as if the said comments
WALK TOGETHER, TALK TOGETHER
are outside the scope of assignments and valuer
AND ROCK TOGETHER.
should limit only to their role as decided by the
banks.
68 Seminar on Valuation of Properties - 2011

RECOVERY OF DEBTS DUE TO THE BANKS &


FINANCIAL INSTITUTIONS
Er. S.S.K. Bhagat

VALUATION REPORT OF PROPERTY Economics Wizard had been vigilant and taking
UNDER SRFAESI ACT 2002 precautionary measures to kerb this menace to the
minimum. But since it is global phenomenon, this
(A case Study) infection has crept in our society.

It is well known that any entrepreneur whether an Coming to Indian Scenario, application for loan and
Industrialist, Business magnate, or even Aam Admi sanctioning by the Banks or Financial Institution is a
require loan or financial aid in one form or the other regular practice. The Sanctioning Authorities after
from the Banks or Financial Institutions. The present verifying the reputation, antecedents & Paying back
system is such that even though one may have capacity etc of the borrowers take the risk defray
sufficient money in his possession, still opts for loan the loan after carrying out the Pre-Sanction Loan
since he does not have the amount in books. procedure well known to all of us. Main criteria are
Sometimes need are genuine e.g loan for housing, TRUST & FAITH. The amount thus released is of
business & trade, vehicle, education, visa, social course is returnable on the basis of proper agreement
purposes et el. executed by both the parties.

In foreign countries every need of life, whether At times, borrower may fail due to any reason fail to
essentials or luxuries are on loan basis and that is follow the mode of paying back, the loan becomes
why most of them are under debt. NPA (Non Productive Account).The banks or
Financial institutions have to take all measures to get
One of the major causes of recent economical crunch the due (principal & interest accrued) recovered by
and collapses of the banks has been due to non return adopting various methods say of Persuasion,
of the loan by the borrowers and failure to abide by Reconciliation, One Time Settlement before going
the terms of agreement in payment of EMIs at to the Court. As you know litigation through the Court
specified intervals. The Developing countries have is not lengthy, cumbersome but costly also .Looking
still not recovered from the Melting down resulting at the gravity of the situation and enormous un-
in under employment or even retrenchment adversely realised loan amount, the Govt. setup special courts
affecting the workers majority of who are immigrants. known as Debt Recovery Tribunals .But these too
did not prove to be that effective with respect to
This epidemics has affected our country also but not promptitude as was expected. Thus Govt. taking
to the scale of Western countries. Our controlling into accounts the magnitude of the problem and the
authorities like Ministry of Finance, Reserve Bank interest of the Public involved promulgated SRFAESI
of India and the Prime Minister who himself is an ACT 2002 (perhaps the longest name

Er. S.S.K. BHAGAT, F.I.E (India), F.I.V.,M.I.C.A., Chartered Engineer, Approved Govt. Valuer & Surveyor
Vice President, Institution of Valuers, (India), Retd. Chief Engineer, N.D.M.C.
Seminar on Valuation of Properties - 2011 69

The Securitisation and Reconstruction of Financial 30.04.2011 in presence of MR.XYZ, Banks


Assets and Enforcement of Security Interest Act)- Enforcement Officer & ABC, Banks Guard to assess
2002) its present valuations (Fair Market Value, Distress
Sale Value, Value as per Guideline under the
SECURITIZATION AND Securitization and Reconstruction of Financial Assets
RECONSTRUCTION OF FINANCIAL and Enforcement of Security Interest Act., In this
ASSETS AND ENFORCEMENT OF regard, the information made available by the Bank
SECURITY ACT (in short SRFAESI Act)-2002 and that collected during inspection has been taken
as true and correct. Besides, the following documents
SRFAESI Act, it is evident that the said Act was have also been examined:
enacted to enable the banks and other financial
institutions to realise the long-term assets, manage 1. Photo copy of Sale Deed dated 19.02.2002
problems of liquidity, asset liability mis-matches and by M/s. MN Pvt Ltd in favour of M/s. XYZ
improve recovery by exercising powers to take (P) Ltd in respect ot the subject property
possession of securities, sell them and reduce non- for Rs. One Crore plus Stamp Duty of
performing assets by adopting measures for recovery Rs.12, 50,000/-.
or reconstruction. The object of the Act is to improve
the recovery process by vesting the powers with the 2. Photocopy of Deed of Rectification Dt.
banks and financial institution powers to take 23.05.2003 by M/s. MN Pvt.Ltd in favour of
possession of secured assets and sell them in case M/s. XYZ (P) Ltd ratifying th actual area of 1
the borrowers commit default in repayment of the Bigha, 19, Biswa & 6 Biswansi = 5944 sq.yds
loan. as delineated in Col. 9 of the proforma
Some of you must have been assigned such job &
3. Photo copy of Memo No. 4417 DTP(G)
must have carried out the exercise by actually visiting
dt. 25.10.1996 from Distt. Town Planner,
the site again, taking the photographs and other
Chairman Building Plans Committee for
relevant data for undoubted identification of property,
Controlled Area ,Gurgaon to Original Owners
so that there should be no confusion in conducting
conveying approval of Building Plans in respect
any of the modes.
of CLU case Khasra No. xxx/949/2/1 (part),
I take this opportunity of sharing my experience in 9xxx/2/2 (Part), Village ABC Controlled Area,
one case, which after having become NPA and fallen Gurgaon subject to terms & conditions
in the grip of SRFAESI underwent certain anomalies enumerated therein including inter-alia obtaining
thereby affecting the character of the property as Occupation Certificate from the competent
would be evident from the summary given below:- authority before occupying the above Building.
No such report has been produced.
Basis of valuation
4. Photo copy of Letter No. 975 Ginyo(E0) dt.
Annexure 1 24.03.2011 by the Distt Planner Enforcement
addressed to State Bank of India, Stressed
Pursuant to the request from the Chief Manager, Assets Management Branch endorsing Letter
Stressed Assets Management Branch, State Bank No. 148 dt.13.01.2011of RTI stating that
of India, the above property been inspected on validity of CLU of this property has expired.
70 Seminar on Valuation of Properties - 2011

5. Photocopy of letter dated 14-01-2011 by the In view of the above facts and circumstances, the
owner Company addressed to Deputy General instant property suffers from some technical
Manager SBI SAM Branch mention therein the infirmities & legal implications. The latter to be
outstanding bill of Electricity for Rs.15,20,783/-. examined by the Legal Counsel.

As explained above, the property comprised of land Technical infirmities as evident are
bounded by compound wall & two big gates having
Industrial building designed for baking of Breads for a) Obtaining Occupation/Completion Certificate
which necessary plant & machinery was stated to b) No objection Certificate from Fire Safety
have been installed. Enquiry from the site revealed Department after providing fire safety system
that the factory has been since closed, most of the appliances
machinery removed by the owner. At present, the
building and all its ancillaries are in utterly ill c) Providing rain harvesting system
maintained condition, electric supply disconnected d) Construction of Effluent Treatment Plant
tube well out of working order, vegetation grown in
e) Revocation of CLU by payment of dues
the compound and main building & rooms are sealed
intimated by the Department for
by the Bank on taking over the property under
Rs.1,86,67,680/- plus overheads.
SRFAESI.
f) Restoration of power connection after clearance
P Plant & machinery At Site of arrears along with restoration charges known
to the owner
The following item of machinery could be found at g) Clearance from Pollution Control Department
site in neglected condition without operating
h) Commissioning of abandoned tube well &
connections as such their performance, usefulness
internal services apart from general clearance,
and efficacy cannot be checked. In their present
removal of jungle growth & maintenance etc
condition, these cannot be more than scrap for
disposal and shall be valued as such:- i) Procurement of water Softening Plant

1. Swing tray Oven of 30 tray In view of the above, Valuation has been
assessed under different options after taking
2. Bread Proving Chamber of 61 swing tray
clues from current Circle rates of Gurgaon
3. One Mixer with 3 SS bowls Administration w.e.f. 01-04-2011 and
4. Dough Hoist replacement value of the existing Structures
as under:
5. Blenders-2No
6. Misc. Lot of worn out parts like motors, OPTION I
switches,V.Belts, Bearings etc. stacked in sealed
store Treating the property as Agricultural
7. Old furniture, fixtures in Office (Sealed)
The land use as per original sale Deed is Zarai
The value of these items per my opinion, may be meaning agricultural. In that case, value shall be land
taken as Rs. 4, 50,000/-P bounded by compound wall, gates, tube well, guard
Seminar on Valuation of Properties - 2011 71

room and at the most small temple built inside. Main Rate proposed in view of non
factory structure would become un-authorized & availability of peripheral services
shall have no value. The Value shall be as under:-
@ 80%of Rs. 10000/sq.yd
OPTION-II = 0.80x5944x10000 = Rs. 4,75,52,000/-

ASSUMING THE LAND USE REVOKED AS Deduct


INDUSTRIAL
Revalidation of CLU = Rs.1,86,67,680/-
Circle rates for Industrial plots having
Arrears Of Electricity = Rs. 15,20,783/-
peripheral civic services like public
Total = Rs.2,01,88,463/-
sewers, water supply net, drainage etc.
Net = Rs. 2,73,63,537/-
= Rs. 10,000/- per sq.yd
Say Rs.2,73,64,000/- D
Though the plot in question does not
have these services and the owners have E. Value of Factory Building
to provide their own arrangements for
Built up area = 37480 sft
Water supply through tube well, sewer
@ Rs. 500/- per sft = Rs. 1,87,40,000/-
disposal through septic tank etc. this
rate may not be fully applicable. and
Less
may be taken yet by applying as
Depreciation @ 2.25 % p.a
80% of it and assuming that the
taking life as 40 years & Age 6 years
CLU has been obtained and other technical
=13.5 % = Rs. 25,29,900/-
infirmities (as highlighted above ) settled,
Net = Rs. 1,62,10,100/-E
the valuation would work out as under:-

D. Land Add
Ancillary Structures as Per B
Area of Plot
= Rs. 4,96,000/- B
1 Bigha, 19 Biswa & 6 Biswansi Plant & Machinery as per P
= 5944 sq.yd. = 1.228 Acre = 4953 sq.mt. = Rs. 4,50,000/- P
Circle rate for Indutrial CLU plots Thus total value = D + E + B + P
with Peripheral civic services In = Rs. 4,45,20,100/-
accordance with Circle Rates Say Rs. 4,45,20,000/-
notified by Deputy Commissioner (Rupees Four crores forty five lacs and twenty
Gurgaon = Rs. 10,000/- per sq.yd thousand only)
72 Seminar on Valuation of Properties - 2011

OPTION III
Thus total value = F + E + B + P
As is Where is Basis = Rs. 3,79,60,100/-

Considering the property in its present position Say Rs. 3,79,60,000/


without the Legal Implications & Technical Infirmities (Rupees Three crores seventy nine lacs and
sixty thousand only.)
F. LAND
Certificates
Area of Plot = 1 Bigha, 19 Biswa & 6 Biswansi
= 5944 sq.yd = 1.228 Acre = 4953 sqm 1) The information furnished in part-I is true and
correct to the best of my knowledge and belief.
Circle rate for Gair Mumkin
2) I have no direct or indirect interest in the
Plots e.g. plots without CLU property valued.
@ Rs.3,500/-per sq yd 3) I have inspected the property on 30.04.2011in
Thus Value = 5944x3500 presence of Mr. ABC, Banks Enforcement
= Rs.2,08,04,000/- F Officer.
4) If this property is offered as collateral security,
E. Value of Factory Building. the concerned financial institution is requested
to verify the extent of land shown in this valuation
Built up area = 37480 sft report with respect to the latest legal opinion.
@ Rs. 500/- per sft = Rs. 1,87,40,000/- 5) Value varies with the purpose and date. This
report is not to be referred if the purpose is
Less different other than mentioned in the report.
Depreciation @ 2.25 % p.a Taking 6) This valuation work is undertaken by this valuer
life as 40 years & Age 6 years based upon the request from the Chief
= 13.5 % Manager, SAMB, State Bank of India

= Rs. 25,29,900/- 7) The report is based on Valuers opinion as on


the date of valuation. The Bank is free to accept
Net = Rs. 1,62,10,100/-E it fully or partially or completely ignore.

Add
Ancillary Structures as Per B Signature of the Approved Valuer
= Rs. 4,96,000/- B
Place : New Delhi
Dated :02.05.2011
Plant & Machinery as per P
= Rs. 4,50,000/- P
Seminar on Valuation of Properties - 2011 73

LEGAL ASPECTS OF VALUERs INTEREST :


ON VALUE, VALUATION AND VALUERS
N. Somasundaar

Sir, as a man advances in life, he gets what is Valuers are persons who have mastered the technical
better than admiration judgement to estimate know how of valuing things fairly and justly. It
things at their true value Johnson in Life of would be illogical to expect from the said experts
Samuel Johnson, 1775 an exact value of things since value of things would
differ based on its utility, need, demand and supply.
The ultimate acumen is knowing the exact value There are periods of boom, fall in prices and periods
of things The maxims of La- Rochefoucauld, where there would be no demand at all. The value
1959 arrived at the period of boom will pale into
insignificance and would look irrelevant at a period
Everyman by common sense has an assessment, when there is no demand at all. Valuers are
estimation and judgement of the worth of things, possessed of that expertise to value a thing by
goods and assets he deals with and this basic forecasting of a price that the said thing could fetch
knowledge which a person acquires by experience on a later relevant date, for the purpose of which a
in dealing with people and properties, profits him in valuation has been sought from him by his clientele.
his day to day affairs. Valuers are experts who
have mastered the art of valuation. The concept Valuers are today, much sought after experts by
value is a word of different meanings when individuals, corporates, government authorities and
employed in different situations. A man is known courts. Since their opinion by way of valuation
by his values (virtues and vices). A man who knows precedes the decision of parties to enter into a
to arrive at nearly the exact value of things is a contract either of small nature or high magnitude,
valuer. It has always been a subject of debate as they have a great role to play, with responsibility,
to whether the valuation is an art or science and our integrity and accountability. With rapid
Honble Apex Court had in its famous Bank industrialization and economic development, the
Nationalisation case had coined the word arenas requiring valuers participation in the guidance
valuation science. Valuation is a scientific analysis of commercial ventures and economic activities have
of available data and information gathered from vastly expanded.
market source and it is an art of judgment and
forecast. As mathematical certainity is not possible A valuation is sought for creation of security interest
to be attained and as two valuers arriving at the in favour of banks, sale and purchase of land and
value of one and same thing would come out with buildings, estimation of value of leasehold rights, life
different value despite the availability of same datas, estate and reversionary interest, to a tourist for visa
valuation is more an art based on logical reasoning purposes, in a court of law for arriving at court fee
and cannot be an exact science. payable on determination of market value, in a rent
control proceedings for arriving at fair rent based on

Practising Advocate, Thirupur


74 Seminar on Valuation of Properties - 2011

market value, for obtaining succession certificates, under the said Act. As per Sec.35 (2) of The
probating of will, partition of properties, settlement Architects Act, the architects registered under
of properties, for arriving at residual value of the the Act is to gain preference in the matter of
assets of properties at the time of constitution / appointment as architect under the Central or
dissolution of firms, winding up, merger and State Government or in any other local body or
amalgamation of companies, for arriving at or institutions which is supported or aided from
assailing upset price in a court auction sale, for fixing the public or local funds or in any institutions
of reserve price of saleable assets before DRT or by recognized by Central or State Government.
Authorized Officer under SRFAESI Act, in the mater The Architects Act had provided the qualification
of auction sales by revenue / statutory authorities for registering as an architect. For a valuer, there
under Income Tax Act, Revenue Recovery Act and is no regulatory statute unlike The Advocates
the like Acts, for ascertaining the value of goodwill, Act for lawyers, the Chartered Accountants Act
shares, stocks and rental value, by Insurance for Auditors and the like. The Institution of
Companies for assessing of loss suffered by fixed Valuers, Delhi confers the title of approved
assets, incidences of fire, theft, damage to goods and valuers and the Central Board of Direct Taxes
vehicles, for assessment of value of properties of Income Tax Department confers the title
purchased and constructed to facilitate an assessee registered valuer. It also appears that the
to file his IT returns and for substantiating its true Universities in India are yet to have a separate,
value and for assessment of the value of intellectual specialized degrees on valuation. Professional
property rights, copy rights and patent rights. private organizations like RICS is learnt to have
introduced a professional qualification viz. Assoc
The aforesaid challenges and opportunities puts the RICS, MRICS and FRICS for professionals in
valuer in pivotal, indispensable position and to cope land, property and construction valuation.
up with the demands of the present day world, the
valuer has to equip himself with up - to date 2. With the passing of SRFAESI Act, the valuers
knowledge, proficiency in his field of practice and had come to be statutorily recognized as
advanced methodology and tools. Knowledge on approved valuers under Rule 2 (d) of The
various legal aspects is a should and must for every Security Interest (Enforcement) Rules 2002. As
valuer not only for arriving at fair and just valuation per the said Rule, approved valuer means a
but for protecting himself from having to face any person registered as a valuer under Sec.34
legal consequences due to his wrong acts, AB of The Wealth Tax Act 1957 and approved
commission or omission. by the board of directors or board of trustees
of the secured creditor as the case may be.
FEW LEGAL ASPECTS THAT ARE WORTH Under Rule 8 (5) of the said Rules it has been
DELIBERATING : provided . Before effecting sale of
immovable property referred to in Sub Rule
1. Qualification for valuation has not been 1 of Rule 9, the Authorised Officer shall
legislated so far and there is no separate law to obtain valuation of the property from an
regulate the conduct of the valuers as of now, in approved valuer and in consultation with the
the lines of The Architects Act 1972 which secured creditor, fix the reserve price of the
specifically prohibits use of said title of architect property and may sell the whole or any part
by any person other than an architect registered of such secured asset by any of the four
Seminar on Valuation of Properties - 2011 75

methods prescribed thereunder. It is schedule III to the said Act, the price or other
therefore obvious that in fixation of reserve price consideration for which the property may be
of the secured asset to be sold, obtaining acquired by or transferred to any person under
valuation from an approved valuer is a sine qua the terms of a deed or trust or through or under
non a mandatory legal requirement. While a any restrictive covenant in any instrument of
valuation is required to be obtained from transfer shall be ignored. If the assessee is
approved valuer to facilitate the Authorised carrying on a business for which accounts are
Officer to fix the reserve price in consultation maintained by regularly, the net value of the assets
with secured creditor, if the auction do not of the business as a whole, having regard to the
fructify for want of buyers, the banks are balance sheet of such business on the valuation
unilaterally reducing the reserve price in respect date is to be taken as value of such assets.
of subsequent auctions without obtaining further
valuation report and this matter was a matter of 4. The Full Bench of Honble Supreme Court of
challenge in M. Chokkalingam Vs. The India in its land mark judgement while dealing
Authorised Officer, Indian Bank and another with medical negligence under the Consumer
before Division Bench of Honble Madras High Protection Act in Indian Medical Association
Court reported in CDJ 2009 MHC 3087. Vs. V.P.Shantha and others (Civil Appeal
Although the said Judgement is sub silentio as No.688 of 1993) had in para 22 of the said
to whether a fresh valuation need to be taken judgment observed as follows : In devising
before reducing reserve price every time, there a rational approach to professional liability which
seems to be some force in the argument put forth must provide proper protection to the consumer
by the petitioners counsel in the light of well while allowing for the factors mentioned above,
settled legal proposition that the property cannot the approach of the Courts is to require that
be sold for less than reserve price and not even professional men should possess a certain
at the reserve price without the consent of the minimum degree of competence and that they
borrower. The approved valuers valuation should exercise reasonable care in the discharge
being pivotal in fixing the reserve price, why a of their duties. In general, a professional man
fresh valuation should not be insisted before owes to his client a duty in tort as well as in
deciding on reducing the reserve price is a contract to exercise reasonable care in giving
debatable position. advice or performing services. [See: Jackson &
Powell (supra), paras 1-04, 1-05 and 1-56].
3. Under the Wealth Tax Act, registration is granted Immunity from suit was enjoyed by certain
under Sec.34 AB after a valuer satisfies the professions on the grounds of public interest.
rules made under the Act. As per the said Act, The trend is towards narrowing of such
the value of any asset, other than the cash shall immunity and it is no longer available to
be estimated either by the assessing officer architects in respect of certificates
himself or by the valuation officer if reference is negligently given and to mutual valuers.
made to him under Sec.16 A of that Act. In
either case, the value shall be estimated to be 5. A valuer being a professional is expected to be
the price which the asset would fetch if sold in an expert / specialist in his field and any wrong
the open market on the valuation date. For the opinion or advise given by him could attract
purpose of determining the market value under penal consequences and suit for damages in a
76 Seminar on Valuation of Properties - 2011

civil court or for deficiency of service under above, the court finds that no prima facie
Consumer Protection Act. After the case has been made out as against the
amendment passed to Consumer Protection Act petitioner. The allegation made against the
which is effective from 15.03.2003, hiring of petitioner, who is only an approved valuer are
services for commercial purpose had been found to be baseless. Absolutely no materials
excluded from the ambit of said Act and as such, are available for charging him for the
the corporates, banking companies and financial offences alleged against him. The Honble
institutions hiring the services of valuers for their High Court had further held that it is
commercial purpose cannot seek legal recourse definitely not the duty of the valuer to look
under Consumer Protection Act. However a into the authenticity of the documents. For
suit for damages as against valuer for his assessing the value of the property which is
negligent act would lie before a civil court. the prime job of the valuer, he is not supposed
to identify and meet the owner of the
6. A valuers prime duty is to value a property property. However in para 7 of the judgement,
assigned to him for valuation and for arriving at the Honble High Court had observed
the correct value, he needs to physically inspect Even if the branch manager
the property. In cases where, the valuer had locates the property for the purpose of
identified a wrong property, given a wrong inspection by the approved valuer, the
valuation report resulting in loss to his client who valuer is supposed to identify the property
had engaged his services and is guilty of fraud with the schedule found in the copy of the
and misrepresentation in collusion with the document furnished to him for valuing the
borrower, he may be prone to criminal property. In the light of T.N. Ravis case
prosecution for offences of cheating under (Calender Case No. 63 / 2004, Inspector of
Sec.420 I.P.C. and to other charges under Police, SPE/CBI/ACB, Chennai Vs.
Sections 467, 471 and Sec.120 B of I.P.C. The J.S.Prabhu and others) handled by CBI
Honble Madras High Court had in L.N. which has ended in conviction on 19.05.2009
Rajagopalan Vs. State by Additional and also in view of the aforesaid observations
Superintendent of Police, SPE/CBI/ACB/ of the Honble Madras High Court it could be
Chennai reported in 2010 (1) MLJ (Crl) 709 safely concluded, it is the duty of the valuer to
held : The poor approved valuer, well qualified identify the property although he has no duty to
in assessing the value of the property, has look into the authenticity of documents and is
been wrongly implicated having found that not supposed to identify and meet the owner of
no properties were in existence as though he the property.
committed cheating punishable under Sec.420
of I.P.C. Likewise, the other charges under 7. A valuers evidence in a court of law is
Sections 419, 467, 468, 471 read with recognized as expert evidence in the matter of
Sec.120B of I.P.C. have also no basis. The valuation. The Division Bench of Madras High
valuers are supposed to contact the branch Court in Mehrunnissa Begum and another Vs.
manager and not the borrowers. None of the Begum Nathu Bibi and others reported in 1989
witnesses has spoken to the fraudulent or (1) MLJ 461 held . if the parties seek
dishonest role played by petitioner in to place evidence through an expert, such
fabrication of the documents. In view of the as an Engineer with regard to value of Item
Seminar on Valuation of Properties - 2011 77

4 and as well as other items whose value unmarried daughter and minor grandsons and
may require assessment through an expert, unmarried grand daughters in the male line if their
certainly that should be permitted and we parents are dead. A family comprising of 5
are giving directions to that effect in the persons is entitled to hold 15 standard acres and
present judgement. together with an additional 5 standard acres for
every member of the family in excess of 5 and in
8. A valuer who may have to assess the value of any case it shall not exceed 30 standard acres.
an agricultural land shall be conversant with the
ceiling limits of permissible land holdings as Under Sec.5(3)(b), in calculating the extent of land
envisaged under Tamilnadu Land Reforms held by a family or by an individual person, the
(Fixation on Ceiling) Act 1961. share of the family or of the individual person in
the land held by a firm, society or association of
After the date of commencement of the Act individuals (whether incorporated or not) or by
namely The Tamilnadu Land Reforms (Fixation a company (other than a non-agricultural
of Ceiling on land) Act 1961 i.e. 15-02-1970 company) shall be taken into account.
as well as its notified date i.e. 02.10.1970,
holding of lands in excess of Ceiling Area by Sec.7 of said Act provides that on and from date
any person is hit by the provisions of the said of commencement of the said Act, no person
Act. shall, except as otherwise provided in the said
Act, be entitled to hold land in excess of the
Under the said Act, a person includes any ceiling area.
company, family, firm, society or association of
individual whether incorporated or not and under It is to be pointed out here that as per Sec.10(4)
Sec.5, ceiling has been fixed for holding of lands of the said Act if any person has specified the
on and from the date of commencement of the particulars of the land which he desires to retain
said Act. within his ceiling limits, the authorized officer shall
as far as practicable declare the same land as
Under Sec.5(1), the ceiling area in case of every comprised within his ceiling area.
person (which definition includes a company)
other than the educational institutions, students Sec.20(1) of the Act provides that if as a result
hostel, agricultural school, orphanage and public of any transfer of land either by sale, gift or
trust of charitable nature, shall be 15 standard otherwise effected on or after the notified date,
acres. The extent of one standard Acre vary the extent of land held by transferee exceeds
from Ac.0.80 cents in case of wet lands to Ac the ceiling area, then the right, title or interest
4.00 in case of dry land in proportion with the accrued in his favour by virtue of such transfer
land assessment value. in the land in excess of the ceiling area, shall as
a penalty for contravention of Sec.7, be deemed
Under Sec.5(2) of the Act of 1961, all the lands to have been transferred to the Government
held individually by the members of a family or with effect from the date of such transfer, on a
jointly by some or all of the members of such declaration made by the authorized officer with
family shall be deemed to be held by the family. in whose jurisdiction such excess land or the
A family comprises of husband, wife, minor son, major part thereof is situated. The authorized
78 Seminar on Valuation of Properties - 2011

officer shall record in writing the reasons for such (Fixation of Ceiling on Land) Rules 1962 had
declaration and the declaration shall be made been passed which became effective from
after giving reasonable opportunity to the 25.11.2010.
transferor as well as transferee. No suit or other
proceeding by the transferee shall lie in any court In view of the amendment made to Sec.37 A,
for refund of the consideration for any such any industrial or commercial undertaking holding
transaction. However the said vesting of land any land in excess of ceiling area acquired before
with Government would not be automatic but 25.11.2010 shall make an application to the
after declaration as aforesaid. Government within 180 days from 25.11.2010
and such undertakings holding any land in excess
If any industrial or commercial undertaking of ceiling area acquired after 25.11.2010 shall
desires to hold or acquire any land in excess of make an application to the Government within
the ceiling area, it shall make an application under 180 days from the date on which such lands
Sec.37A of the Act seeking prior permission for were acquired through registered documents.
holding or acquiring such land.
Similarly pursuant to the amendment passed to
Under Sec.37 B of the Act a public trust of Sec.37 B of the Act, the public trust created
charitable nature created before 01.03.1972 before and after 01.03.1972 can now make an
was to hold only 5 standard acres and if the said application to the Government to hold the lands
trust was desirous of holding or acquiring more acquired by it within 180 days from 25.11.2010
than 5 standard acres, it was to be done only if the acquisition was made prior to 25.11.2010
with prior permission of the Government by and within 180 days from the date on which the
making an application to it. The public trusts said land had been acquired through registered
created after 01.03.1972 could not hold any documents if the said acquisitions were made
land without prior permission of the Government after 25.11.2010. The Government in revenue
and the word land would mean agricultural department have also been empowered to admit
land. Ignoring the said provisions of the Act, an application presented after expiration of the
several public trusts created before and after prescribed period, if they were satisfied that the
01.03.1972 had acquired lands in excess of the applicant concerned had sufficient cause for not
ceiling area without obtaining prior permission presenting it with the said period.
of the Government and when they had
approached the Government for post facto 9. It is indispensable for a valuer assessing school
permission, the Honble Madras High Court had or college property to be conversant with the
held obtention of such permission to be illegal. provisions of Tamilnadu Recognised Private
The Tamilnadu Government had with a view to Schools (Regulation) Act 1973 and Tamilnadu
validate such holdings or acquisitions had passed Private Colleges (Regulation) Act, 1976.
Tamilnadu Land Reforms (Fixation of Ceiling on
Land) Amendment Act 2008 which received the In respect of a private school property there is
assent of the President on 06.06.2010 and the a restriction for alienation or mortgaging the same
said amendment was published in Tamilnadu under Sec.31 of Tamilnadu Recognized Private
Government Gazette on 10.06.2010. Further to Schools (Regulation) Act, 1973 unless prior
that, amendment to the Tamilnadu Land Reforms permission is obtained from competent authority.
Seminar on Valuation of Properties - 2011 79

Otherwise the transfer of property would be held Madras High Court in Association of
null and void. The competent authority to grant University Teachers, National College
permission for pre-primary, primary and middle Unit, Trichy Vs. AICTE, New Delhi and
schools is District Educational Officer and the others reported in AIR 1999 Mad. 164. It is
appellate authority is Chief Educational Officer. also held by our Honble High Court that
The competent authority to grant permission for the TN Private School Act shall not apply
high schools is Chief Educational Officer and the to Matriculation Schools and the TN
appellate authority is Joint Director of School Private Colleges Act shall not apply to
Education (Secondary Education). The Engineering Colleges.
Competent authority to grant permission for 10. A valuer having to assess the value of assigned
higher secondary schools is Chief Educational lands by grant of Government shall take care to
Officer and the appellate authority is Joint read the entire conditions governing the
Director of School Education (Higher Secondary assignment since the assignment deeds normally
Education). provide clauses prohibiting the assignees from
alienating or encumbering the assigned lands
In respect of a private college property there within 10, 15 or 20 years as the case may be
is a restriction for alienation or mortgaging the from the date of assignment and till such time
same under Sec.27 of Tamilnadu Private the entire value of the land is fully paid to the
Colleges (Regulation) Act, 1976 unless prior assignor. In certain assignment deeds which are
permission is obtained from competent authority. exclusively given to landless poor or certain class
Otherwise the transfer of property would be held or community people, there are specific clauses
null and void. As per the Rules, the college would requiring the assignee to dispose of the lands to
mean and include Arts & Science College, such class or community people alone. Hence
Teachers Training College, Physical Education the property setout under assignment deeds are
College, Oriental College, School or Institute of acceptable as security only if he mortgagor is
Social Work and Music College maintained by able to produce no due and no objection
Educational Agency and approved by or certificates from competent authority i.e.
affiliated to the University. The competent Assistant Commissioner of Land Reforms or
authority to grant permission for Arts & Science such authority as prescribed and only if the time
Colleges is Deputy Director of Collegiate stipulated in the assignment deed has expired.
Education and the appellate authority is Director
of College Education. 11. It would be advantageous to a valuer having to
assess the value of the building to be abreast of
If the property stands in the name of the the following land mark decisions of Honble
Trust or Educational Agency or individuals Supreme Court and Honble Madras High
and if the school / college property is to be Court rendered under Tamilnadu Town and
constructed therein, it shall be construed Country Planning Act, 1971:
to be the school / college property, the
school or the college as the case may be Planning permission was obtained under Section
being its beneficial owner although not real 49 of the Act. Section 49 of the Act deals with
owner and the said Acts would apply to all control, development and use of the land and it
such properties as was held by Honble has nothing to do with the building regulation.
80 Seminar on Valuation of Properties - 2011

Building permission and planning permission @ The appropriate planning authority de hors the
should be obtained. [A.M. Kandasamy & power under Section 56 of the Act has also got
others vs. Corporation of Madras & others, powers to order demolition of unauthorized
AIR 2002 (Mad) 318 : (2002) 1 MLJ 599] development under Section 85 (1) (a) of the Act.
[Chairman, CMDA, Chennai
@ In a primary residential use zones, building S.Radhakrishnan & another, (2006) 1 MLJ
carrying on commercial activities in violation to 181 (Mad).
Rule 7 of the Development Control rules for
Madras Metropolitan Area appended to the Act @ When notice under Section 56 (4) of the Act, is
of 1971 are prohibited. [Suriya Sweets V. State issued with regard to the demolition of an illegal
of T.N. by its Secretary, Housing & Urban construction, the aggrieved person may submit
Development Dept. (2008) 5 MLJ 547]. a revised plan to the Authorities concerned.
Notice under said section shall not take effect
@ When, subsequent to approval of the sanctioned pending final order on the revised plan.
layout no further steps have been taken and the [M.Senniappan V. Director of Town &
approved plan was not given effect to. In view Country Planning, Chennai & Others (2007)
of Section 50 of the Act of 1971, after 3 years 1 MLJ 228.
from the date of permission, the existence of
sanctioned plan itself would come to an end and @ The period prescribed under Section 50 of the
the question of allocation of portion for public Act is only for the compliance of the conditions
purposes does not arise. As such, without even imposed for the sanctioned of the layout and also
giving notice to the owner, the municipality could for the owner to make the layout within the
not come to any conclusion that the ear marked prescribed period. In case if the land owner is
portions are encroached upon and that no notice unable to make the layout within the prescribed
is required for removal of such encroachments. period of 3 years, the proviso comes into play
[Raja Baskara Shanmugam @ N. Kumaran which empowers the planning authority to extent
Sethupathy V. Director of Town & Country such period for such times as it thinks proper
Planning & another, (2008) 2 MLJ 824]. but not exceeding a period of 3 years on an
application made on that behalf before the expiry
@ The object of the Act is not to permit of the prescribed period of 3 years. The
unauthorized and illegal development activities provision is only an enabling provision for the
in contravention of the planning permission land owner to strictly comply with the sanction
issued by the appropriate authority. The offence of the layout. Hence, it cannot be construed
has to be treated as continuing one till the that by expiry of the period of 3 years, the
offending construction is either demolished or sanction itself is lapsed and consequentially the
altered. The planning Authority is perfectly conditions imposed under the sanctions layout
right in issuing a notice to proceed against also lapsed and cannot be enforced.
unauthorized development even after the [Vilupuram Municipality by its Commissioner
expiry of 3 years and taking appropriate V. M. Subramanian and others,(2002) 3 MLJ
steps for demolition of the unauthorized 375].
construction. [Chairman CMDA, Chennai V.
S.Radhakrishnan & another, (2006) 1 MLJ
181 (Mad).
Seminar on Valuation of Properties - 2011 81

@ Section 49 of the Act is confined to minor MLJ 666 : (2005) 1 CTC 573 (Mad) : (2005)
violation only. Unauthorised constructions which 2 LW 285].
are illegal have to be demolished and they cannot
be regularized under Section 49 of the Act. @ A building constructed in deviation of building
[Madras Race Club V. CMDA & others rules regulations etc., should result in the order
(2006) 4 MLJ 1 (Mad]. of demolition. [Salahudin Babu Vs. P.T.
Prabakar and another, (2005) 1 LW 643:
@ Reserving a site for street, open space, parks, (2005) 1 CTC 385].
schools, temples etc., near layout or plan is only
for public purposes. The reservation should be @ The plot in question was allotted for the purpose
used by the public in general. The legal owner of community hall. If it is used for the purpose
would cease to have any right over it. of constructing a kalyana mandapam, the same
cannot be questioned, in view of the settled
@ The object of the Act is to enable the residents proposition that Kalyana mandapam can be used
of the area to have a common land for public for community purposes [Co-operative Nagar
purpose. An alternative site is given and handed Welfare Association by its President
over, the purpose has not been defaulted. V.Director ofTown & Country Planning,
[Maharani Avenue Kudiyiruppor Chennai and others (2008) 1 MLJ 1285].
Nalasangam, Coimbatore V. The
Commissioner Town Planning Authority, @ The petitioner was directed by the court to put
Chennai and others (2006) 3 LW 963). up construction as per the sanctioned plan dated
03.04.2000 on furnishing undertaking to the
@ The requirement contained in various statutes effect that construction will be according to the
with regard to provision of open spaces and sanctioned plan and if there is any deviation, the
other basic necessities for the community are all applicant will demolish the construction made
made in the interest and welfare of the with deviation. However contrary to the
community at large taking the spirit of Article sanctioned plan and undertaking, building was
21 of the Constitution of India. [President, constructed with deviation. Revised plan was
Thanjavur Dioces Society V. Nirmala Hagar filed and refused on 19.07.2001. In the mean
and others, (2008) 3 MLJ 723]. time the Government brought in a scheme to
regularize unauthorized constructions. The bench
@ The planning Authorities should keep in mind the of the high court made it clear that no
zonal classification before considering the regularization or ratification was permissible.
application and granting permission [ P.T. The construction was contrary to the orders of
Prabakar V. Member Secretary, CMDA, the High Court and undertaking of the petitioner.
Chennai, (2006) 5 CTC 449]. The non compliance of the court order amounts
to willful disobedience. [Kumaran Silk Trade
@ Unauthorised building can be demolished even Ltd. by its Director T.K.Chandran, Chennai
without notice. A building put up in an area less V. Dro Devandra and others (2002) 4 CTC
than 80 sqm. no notice under Section 56 of the 525].
Act is issuable. [CMDA by Member Secretary
V. P. Muthukrishnan and Others, (2005) 1
82 Seminar on Valuation of Properties - 2011

@ The person not having a house and owning a construction was made by the Housing Board
plot with his hard earned money, sought for in clear and flagrant violation and disregard of
equitable remedy from court for condoning some FSI and upheld the order of demolition of
deviation in the constructions. The construction all the eight floors as ordered by the Bombay
was in his own land without encroaching Municipal Corporation. While dismissing the
anybodys right. Therefore it could be condoned special leave petitioner, the court observed as
without affecting public interest. [Dilip Before parting with the case, we would like to
Bhandari & others V. The Commissioner observe that this case should be a pointer to all
Corporation of Chennai & another (2004) 5 the builders that making of unauthorized
CTC 481]. constructions never pays and is against the
interest of the society at large. The rules,
@ Petitioner having got a plan sanctioned for regulations and bye-laws are made by the
construction of a basement, parking, ground Corporations or development authorities taking
floor and three floors,had not only flouted the in view the larger public interest of the society
permission and the plan in respect of construction and it is the bounden duty of the citizens to obey
of those floors including not providing of any and follow such rules which are made for their
parking space, but even while the litigation was own benefits.
pending be carried on the construction, disregard
of the approved plan and constructed additional I have placed before the esteemed Valuers, few
floors 4, 5 and 6 without any regard to building Judicial Verdicts and Wisdom of the Honble
by laws and other relevant enactments and Apex Court of India and the Madras High Court,
showing scant respect to the order of court. profoundly believing that the knowledge of said
Such conduct does not entitle the petitioner to verdicts will be beneficial to the valuer fraternity
any order by way of indulgence or discretion to conduct themselves in the most appreciable
[Kumaran Silk Trade V. Devendra & others manner in their field of knowledge and expertise,
(2007) LW 747 (SC)] without having to invite Judicial Interdict. I wish
the workshop of valuers a tremendous success
@ In Pratibha Co. Operative Housing Society and place on record my heart - felt appreciation
Ltd. Vs. State of Maharashtra AIR 1991 SC to the Institution of Valuers of Chennai Chapter
1453 : (1991) 3 SCC 341, the Honble Supreme for having given me the privilege of attempting
Court came down heavily on the Housing Society at this maiden effort. IOV is doing a wonderful
which made construction in violation of the Floor job. To impart knowledge is a greatest act of
Space Index. The Court said that such unlawful charity. Best Wishes always.
Seminar on Valuation of Properties - 2011 83

WHETHER GUIDELINES ARE PRESCRIBED BY THE BANKERS


FOR VALUATION OF PROPERTY?
Pala. Ramasamy

Bank before advancing money on secured interest 6) Individual FMB Sketch


want to ascertain the valid marketable title of the 7) Subdivision Statement
secured interest offered. Bank, if it is satisfied with
the secured asset offered for the loan to be granted 8) Chitta, Adangal for latest fasalie
and other terms being accepted by the borrower then 9) Patta
a contract is concluded. For the loan to be availed
10) Kist Receipt
the security interest is charged. Hence on the date
of contract, the banker, on default can fall back on 11) Certificate from VAO
the security offered and similarly, the mortgagor is
aware that on default his secured interest will be sold b) In respect of Plot in Revenue Survey
and money will be realized thereby. 1) Survey Land Register Extract
2) Settlement Register Extract
Thus the term of contract is that, on default of
repayment the security interest will be enforced. 3) Copy of Updated Registry
Hence to ascertain title, marketability and value of 4) Village Map
the secured interest the bank is depending on
5) Combined FMB Sketch
experienced professionals guidance for marketability
and value. It gets the assistance of the advocate 6) Individual FMB Sketch
regarding marketability and the valuer for ascertaining 7) Approved Layout
the value of the property.
8) Sub-division Statement
The advocate, on perusal of the records made 9) Chitta, Adangal for latest fasalie
available to him gives his opinion. It is preferable, 10) Patta
that the lending institution gives instructions to all the
persons who are involved in the process about its 11) Kist Receipt
requirement and the documents which are to be 12) Certificate from VAO
produced and verified inter alia the following viz.
c) In respect of Plot in Town Survey
a) In respect of Agricultural Land 1) Survey Land Register Extract
1) Survey Land Register Extract 2) Settlement Register Extract
2) Settlement Register Extract 3) Town Survey Register Extract
3) Copy of Updated Registry 4) Topo Sketch
4) Village Map 5) Town Survey Sketch
5) Combined FMB Sketch

Practising Advocate, Madurai


84 Seminar on Valuation of Properties - 2011

6) Approved Layout f) In respect of Flat


7) Subdivision Statement 1) Survey Land Register Extract
8) Vacant Site Tax Receipt 2) Settlement Register Extract
3) Town Survey Register Extract
d) In respect of Site and Superstructure in
4) Topo Sketch
Revenue Survey
5) Town Survey sketch
1) Survey Land Register Extract
6) Approved Layout
2) Settlement Register Extract
7) Subdivision Statement
3) Copy of Updated Registry
8) Property Tax Receipt
4) Village Map
9) Approved Building Plan
5) Combined FMB Sketch
10) Allotment Letter
6) Individual FMB Sketch
11) Joint Venture Agreement
7) Approved Building Plan
12) Construction Agreement
8) Sub-division Statement
13) Sale Agreement/Sale Deed
9) Chitta, Adangal for latest fasalie
14) List of Plots allotted
10) Patta
11) House Tax Receipt The availability of the revenue records is to be made
12) Certificate from VAO mandatory. These requirements have to be informed
by the institutions to the customer at the earliest point
e) In respect of Site and Superstructure in Town of time so that it will be made available to the
Survey advocate by the customer as turnaround time (TAT)
is important. These records maintained by the
1) Survey Land Register Extract authorities will throw much light to verify and
2) Settlement Register Extract compare the property offered as security as stated
in the registered documents in respect of
3) Town Survey Register Extract
4) Topo Sketch 1) Location
5) Town Survey Sketch 2) Extent
6) Sub-division Statement 3) Measurement
7) Approved Building Plan 4) Four boundaries
8) Property Tax Receipt 5) Subdivision of Survey Number
9) Water Tax receipt
The perusal of the revenue records will reveal,
10) Drainage Maintenance Charges Receipt whether the property offered is a patta land or
11) EB Receipt government land? The comparison of subdivision
statement and survey sketch with layout and
Seminar on Valuation of Properties - 2011 85

registered documents will enable for proper i) Bank Pass Book


identification of property and its extent. There are j) EB Card
several instances where survey number is wrongly
quoted as 412 instead of 421 and sometime North and an opinion is obtained by the customer. The
South measures inadvertently stated as East valuer takes the opinion and value the property
West. identified by the customer or his representative or
some times on his own identification. The degree of
If the revenue records were not made available, then co-operation of the customer for valuation before
the professional is left with no other option except lending will not be similar at the time of revaluation
to scrutinize the records made available and to give during sale. Any clarification sought by the
his opinion. There are instances where housing loan professional will be replied with readymade answer,
was given for construction of new building after we are ignorant and we have placed the property on
demolishing the old structure in Government as is where basis is. There are instances where
Porombokke. In Madurai Town abutting Vaigai River credit officer compares the legal scrutiny report and
there were many constructions erected and the valuation report and finds that, discrepancy in extent
owners of the said buildings were possessed with of secured interest. Advocate furnishes the report
the following viz. gathering the data available from the documents and
gives the measurement. But the valuer measures the
1) Electricity Service Connection property and prepares his report. The discrepancy
2) Water Tap Connection is subsequently rectified.
3) Property Tax Assessment
In a case handled by me where the property is not
4) Approval of Building Plan identified properly and its valuation has induced the
5) Family Card issued by Civil Supply bank to give substantial loan amount. The list of the
Corporation case is as follows:
6) PAN Card by Income Tax Department
Property offered by Mrs.Umadevi wife of
Mr.Sadasivam. The description of property is
Such an owner of the building will be able to provide
as follows:
following documents

a) Registered Sale Deed Madurai District, Madurai North Taluk,


b) Encumbrance Certificates for a period of Andarkottam, in S.No.38/3A1 measuring 9 cents.
50 years The party identified the property, as it is lying on the
outskirts of Madurai Corporation abutting four way
c) Electricity Receipts lane. It is valued at Rs.16 lakhs. On classification as
d) Property Tax Receipt NPA it was revalued as Rs.1,60,000/-. The report
e) Water Tax Receipt of the second valuer along with the site map reveals
different location in same Taluk. The property is
f) Underground Drainage Receipt interior village in a different direction.
g) Building Plan
The first valuer valued the property as identified by
h) PAN Card
the borrower. He has not verified and compared the
86 Seminar on Valuation of Properties - 2011

property on ground as that found in the instrument. property and also identification of owner of the
He must have stood along with the mortgagor or his property. In paragraph 7 of the order of the Honble
representative and measured the property. By using High court it is found as follows:
the commonsense along with reasonable prudence
by local enquiry he could have confirmed the village The guidelines prescribed by the bank had not
without the assistance of V.A.O. A glimpse of village been produced by the investigating agency. The
map, top sketch and survey sketch would have petitioner takes a stand that there was no
helped him for verifying and comparing the property guideline prescribed for valuation of the
identified by the mortgagor and as found in the property by the approved Valuers. A Valuer is
instrument. In the absence of the same, it is supposed to estimate the marketability of the
convenient for the customer to show a different property referred for valuation by the bank based
property other than that what is found in the on the documents provided by it. Of course, the
instrument. Here comes the issue what is the role of Valuer is bound to inspect the property referred
the valuer? or what is the contract between the valuer by the bank for valuation purpose. The Valuer
and the bank for availing the service of valuation? cannot simply go to a location of a property on
his own. For the purpose of identification, he
The bank hires the service of the valuer for getting takes the Branch Manager or some authorised
fair valuation of the property, so that they can take a agent to the location at the time of inspection
prudent decision before lending the public money. It for the purpose of identifying the property. The
is the requirement of the bank that the secured Valuers take this precaution just to avoid any
interest to be assessed properly on identified by the mistake in the identification of the property.
mortgagor. Once it is identified by the mortgagor, Even if the Branch Manager locates the
the valuer has to use his expertise and confirm before property for the purpose of inspection by the
valuing the property identified by the mortgagor is approved Valuer, the Valuer is supposed to
as that found in the instrument by which he acquires identify the property with the schedule found
title and it tallies with that on found on the ground. in the copy of the document furnished to him
The task of the valuer will be easy if the aforesaid for valuing the property.
documents are made available by the customer at
the instruction of the institution to the advocate for Thus the judgment speaks about identification of the
getting legal opinion itself. It is the financial institution owner of the property and not identification of the
who hires the services of the advocate and the valuer property to be assessed. But it is important to note
for expertise and it has to give necessary instructions that, no guidelines were prescribed by the
to the customers to place necessary documents nationalized banks handling public money in crores,
before the professionals. A proper guidance is for valuation of the property by the approved valuers.
required for getting opinion from the experts and it
has to be made available to the customers at the first To have a guideline for valuation will lead to better
instance to keep up turnaround time (TAT). In the understanding and to draw limitation by the banker
absence of such guidance, blaming each other will and professional. The guideline will ensure the
not get satisfied result. professionals in determining the value of the secured
interest and mitigate the risk.
In criminal R.C.No.1063 of 2008 Lordship Justice
M.Jeyapaul has dealt with the identification of the
Seminar on Valuation of Properties - 2011 87

APPLICATION OF GPS (GLOBAL POSITIONING SYSTEM) IN THE


PROFESSION OF VALUATION

S.K. Ramesh Mani

What is GPS? How the GPS Works and what it does?

The Global Positioning System (GPS) is a satellite- GPS satellites circle the earth twice a day in a very
based navigation system made up of a network of 24 precise orbit and transmit signal information to earth.
satellites placed into orbit by the U.S. Department of GPS receivers take this information and use triangulation
Defense. GPS was originally intended for military to calculate the users exact location. Essentially, the
applications, but in the 1980s, they made the system GPS receiver compares the time a signal was
available for civilian use. GPS works in any weather transmitted by a satellite with the time it was received.
conditions, anywhere in the world, 24 hours a day. The time difference tells the GPS receiver how far away
There are no subscription fees or setup charges to use the satellite is. Now, with distance measurements from
GPS. a few more satellites, the receiver can determine the
users position and display it on the units electronic
The GPS satellite system map.

GPS satellites transmit two low power radio signals,


There are 24 satellites that make up the GPS space
designated L1 and L2. Civilian GPS uses the L1
segment that are orbiting the earth about 20,000
frequency of 1575.42 MHz in the UHF band. The
kilometers above us. They are constantly moving,
signals travel by line of sight, meaning they will pass
making two complete orbits in less than 24 hours. These
through clouds, glass and plastic but will not go through
satellites are travelling at speeds of roughly 11,500
most solid objects such as buildings and mountains.
kilometers an hour.
A GPS receiver must be locked on to the signal of at
least three satellites to calculate a 2D position (latitude
and longitude) and track movement. With four or more
satellites in view, the receiver can determine the users
3D position (latitude, longitude and altitude). Once the
users position has been determined, the GPS unit can
calculate other information, such as speed, bearing,
track, trip distance, distance to destination, sunrise and
sunset time and more.

What is Latitude and Longitude?


GPS satellites are powered by solar energy. They have
backup batteries onboard to keep them running in the ` Latitude
event of a solar eclipse, when theres no solar power. ` It is used to express how far north or south,
Small rocket boosters on each satellite keep them flying relative to the equator.
in the correct path.
` If we are on the equator our latitude is zero.

Practising Valuer, Chennai


88 Seminar on Valuation of Properties - 2011

` If we are near the North Pole our latitude is take advantage of WAAS. Users can also get
nearly 90 degrees north. better accuracy with Differential GPS (DGPS),
` If we are near the South Pole our latitude is which corrects GPS signals to within an average
almost 90 degrees south. of one to 2 meters. More expensive systems are
available that can provide accuracies to within a
` To remember, the imaginary lines parallel to X centimeter.
axis are latitude
Applications of GPS in General
` Longitude
` It shows the location in an east-west direction, ` In air - navigation by general aviation and
relative to the Greenwich meridian. commercial aircraft
` Places to the east of Greenwich (such as Middle ` At sea - navigation by recreational boaters,
East, India and Japan) have longitude angles commercial fishermen, and professional mariners
up to 180 degrees east.
` Places to the west of Greenwich (such ` For Surveyors - cost savings by drastically reducing
as the Atlantic and North and South setup time at the survey site and providing
America) have angles up to 180 deg incredible accuracy
west.
` To remember, the imaginary lines How it is useful for Valuers in Valuation Profession
parallel to Y axis are longitude
Now, we come to know that, the Central Registry is
Graphical representation of Latitude and requesting the bankers, to enter the latitude and
Longitude (Latlong) longitude of the property, under mortgage for their
records. We, the valuers, are visiting and inspecting the
property physically so we can get the latitude and
longitude from the location, by using GPS, and it can
be entered in our report. These coordinates may be
used by the bankers for the records.
` 1. Get the current position (Latlong) of the
inspected site at its location
` 2. To calculate the approximate area of the larger
How accurate is GPS? extent of site, if all the turn points of the land are
accessible
` Certain atmospheric factors and other sources of ` 3. Compare the shape of the land with the FMB
error can affect the accuracy of GPS receivers.
GPS receivers are accurate to within 15 meters ` 4. To calculate the distance from the property to
on average. the seashore / river, where the Coastal regulation
zones are applicable
` GPS receivers with WAAS (Wide Area ` 5. Make a track to access the land from the known
Augmentation System) capability can improve point or from a landmark
accuracy to less than three meters on average.
` 6. To reach the property, in future, by using Mobile
No additional equipment or fees are required to
navigator (Google maps, Mapmyindia and
Seminar on Valuation of Properties - 2011 89

Satguide) for on road sites and for off the road sites with some other mobile applications such as GPS Status,
Maverick etc.
` 7. In Google maps, view the development of the surrounding locality
` 8. To prepare a key plan using Google map, in which the property situated in a complicated road and location
and which may not have the proper access roads

How it is useful for Bankers

` In future, using this facility, any bank official can easily locate the property and will be able to reach the property,
by using their Mobile navigator
` Further by using Google maps/Google Earth, view the development of the surrounding locality of the site, by
the higher officials who may not have to visit the site, from their work place

Note:

To enjoy these applications, the internet facility must be active.

Application in Detail

` Application 1
` Get the current position (Latlong) of the inspected site at its location

a. Open Google Maps


b. Enter the above latitude and longitue ie. 13.00644, 80.25465
c. Click on Search, it will show the location of the property in Adyar, Chennai
` Application 2

` To calculate the approximate area of the larger extent of site, if all the turn points of the land are accessible
a. Get the Latlong of a point P1 to P12 in the anti-clockwise or clockwise direction.
b. Enter those values in Excel as below
90 Seminar on Valuation of Properties - 2011

Now see, how the above values are to be used in calculating the area of the land

c. Open GE Path in your Computer

d. Copy the above values (Red color only) from Excel table
e. Select the command button Paste grid from Clip board
f. Select the method in GE path as Small area
g. Give some file name and store this in a required location
h. Click on Run on the Menu bar, the picture will looks as below
Seminar on Valuation of Properties - 2011 91

The result is The perimeter is 10133.26 feet and the area is 109.00 Acres, the allotted area is 104.23
acres, by APIIC

Note:
1. To calculate the area of the larger extent approach all the turnout point of the site in clockwise direction or
anticlockwise direction and store the latlong at their respective locations and also record it in a paper by
making a proper tabulation with necessary remarks
2. Before running this application, kindly ensure that, the Google Earth / Google Earth Pro and GE path
software are installed in your system and mainly the internet facility is active.
3. For this application, the above exercise is not necessary if we have the Google Earth Pro licensed version.
In this version, we can calculate the area and perimeter by selecting polygon command, in a Show Ruler
Select Polygon Select the unit - Click on all the turn out point of the location. View the result on the
Ruler Form

Application 3
` Compare the shape of the land with the FMB / Topographical Survey Map

This the topographical survey map of a land


92 Seminar on Valuation of Properties - 2011

The same is viewed in Google earth

Application 4
` view the development of the surrounding locality
` Enter below values in Google Maps in the search bar and click on search, it will take us to the location and
view it in Satellite mode.
A. Co-ordinates
x 13 05 8.8, 79 52 55.5
This is the only house in this layout, and the other developments are at about 400 to 500 m. Zoom
in and Zoom out to view the development of the surrounding locality.
B. Co-ordinates
13.12759, 79.32421
This property is located at the foot of the hill, one side of the boundary is Andhra. There is no
development
Of course, we know physically at site, but you cannot see where is the nearest developed location in the surrounding
locality.
This Application will also help the bankers

Application 5
` Distance of a property from the Seashore / River
a. Open Google Earth
b. Enter the above co ordinates and search
c. Select Ruler
d. Select Line
e. Select the units to Metres
f. Draw a line towards the Seashore/River, in a straight line, from the property location
g. View the distance, we can see the distance simultaneously by stretching the line towards the sea or river.
Seminar on Valuation of Properties - 2011 93

Note: Other Applications

If the distance from the seashore is above 200m or The other applications such as creating track to the
500m, we cannot come to the conclusion that this inspected property and track back to the inspected
property is within or not in the respective CRZ. This property from the current location are practical oriented,
has to be finalized by the Ministry of Environment and so these details are not explained in writing. Once you
Forest get in the use of GPS, we learn everything, because
nothing is impossible.
Application 5
Some useful software
` Location of wind mill 1. Google Earth / Google Pro Trial or Licensed
Version
The windmills are located in remote locations
and there will lots of windmills. How can we 2. Bhuvan (Developed by ISRO)
know, to which we assessed the value, even 3. GE Path
the owner, may not know which wind mill he
owns and its location, only the AMC
contractors know the location and showing it Some of the useful links
to us. If we know the latitude and longitude of For Google Maps
its location we can easily locate the wind mill.
http://maps.google.co.in/maps?hl=en&tab=wl

A. Co ordinates
For Downloading Google Earth
10.6701077, 77.33881
http://www.google.com/earth/download/ge
This is the location of the wind mill near
Udumalpet, Near Coimbatore, Tamil Nadu.
For Bhuvan
http://isrobhuvan.in/
B. Co ordinates
8.26150,77.53665
others
This is the location of the wind mill near
Kanyakumari, Tamil Nadu http://www.gpsvisualizer.com/calculators
94 Seminar on Valuation of Properties - 2011

LEGITIMATE ASSETS UNDER INDIAN ENVIRONMENTAL


LEGISLATIONS
Mr. P Kirupanantha Rajan

Keywords: Developmental Projects COASTAL REGULATION ZONE


Environmental Clearance Awareness Coastal NOTIFICATION 1991 / 2011
Zone Impact Assessment Environment Age
The Coastal Regulation Zone Notification 1991 is
INTRODUCTION one such notification with almost 20 times either
corrected or amended in the span of 12 years and
India, as a signatory of the United Nations finally superseded with the revised Coastal Regulation
Conference on Human Environment at Stockholm Zone Notification 2011. The area between the HTL
in 1972, made leaps over the Environmental issues and 500 metres onwards landward side is generally
thereafter. The Water Act 1974, 42nd Amendment known as Coastal Regulation Zone. The Coastal
to the Constitution in 1976 in making the state to Zones are classified into I, II, III as sensitive area,
endeavor for protection and improvement of the developed area and undeveloped area respectively.
environment, the Air Act 1981 and the Environment The developmental activities in CRZ-I are prohibited
Act 1986 are few of the major legislation adopted with certain exemptions regulated towards the
by the Parliament. landward side of the existing road in CRZ-II and
beach resorts / hotels only permitted between 200
The developmental projects with certain threshold m to 500 m from HTL with certain restrictions in
limits or along the coastal area are being regulated CRZ-III.
under the provisions of Environmental Impact
Assessment Notification and Coastal Regulation It is certain that no thumb rule is so far arrived in
Zone Notification since the early nineties. The ascertaining the legitimacy of the developmental
improper awareness of these rules among the structures along the coastal stretches. The vision of
developers, builders and officials is creating complex more than a kilometer from the seashore with the
situations by means of violations which can lead to naked eye may actually be lesser than 100 meters in
demolition, legal proceedings, etc. the CZMP maps approved by the Government of
India. To simplify the subject any developmental
The Adarsh Building controversy opens the eyes of projects either originally constructed or altered or
the common public on these coastal provisions to modified on or after 19.02.1991 along the coastal
certain extent but however the oversight violations stretches may be verified with the environmental
due to non exposure of these notifications are still clearance obtained under Coastal Regulation Zone
continuing over almost all part of India. Notification 1991. The notification empowers the

Mr. P Kirupanantha Rajan, ME, District Environmental Engineer, Tamilnadu Pollution Control Board,
Nagercoil, Tamil Nadu
Seminar on Valuation of Properties - 2011 95

officials to accord permission only rather than 20,000 square meters and less than 150000 square
ratification. meters of built up area must have obtained prior
environmental clearance from the Ministry of
Any structure constructed over and above 9.0 metre Environment and Forests, Government of India, New
from the ground level in Village Panchayats from the Delhi.
date of the Notification can have no value, as the
same is considered as violation, likewise, lesser than Likewise the category 8 (b) stipulates the Townships
0.4 Hec. of plot size, floor space index exceeding and Area Developmental projects covering an area
0.33, provision of bore well with 200 m from the greater than 50 Hec. and / or built up area greater
HTL etc. has not been permitted. than 1,50,000 sq. meters. must have also obtained
the Environmental Clearance from the Government
Industries except the one which requires water front of India. The developmental projects like Industrial
activity is strictly prohibited in all the CRZ I, II and Estates, Parks, Complexes, EPZs, SEZs, Bio-Tech
III. Parks, Leather complexes with certain conditions
shall also be accorded clearance under the
The parks, play fields, gardens etc are permitted in Notification.
CRZ-III and any project having investment more than
Rs.5 crores must have obtained prior environmental CONCLUSION
clearance from Ministry of Environment and Forests,
Government of India, New Delhi. The Environment (Protection) Act 1986 which is
considered as an umbrella Act for the cause of
ENVIRONMENTAL IMPACT ASSESSMENT safeguarding the Environment ensures the regulation
NOTIFICATION 1994 / 2006 of developmental activities in India with certain
degree of freedom. The violations though are very
The industries and the other developmental projects temporary in nature, will be identified and appropriate
like mining, hydro power projects, ports and harbors, action will be taken against the defaulters. The
tannery, boundaries, electroplating, certain High Way violations viz. total violations, partial violations and
projects etc are regulated since 27.01.1994 through the permissible violations will be dealt as per the
the Environmental Impact Assessment Notification. merits of law. The legitimacy of the assets developed
The procedure for the clearance took a series of steps in 1991 or after could be valid only and only if the
of preparatory EIA reports, Public hearing etc. The same satisfies these provisions of these Notifications
revised EIA Notification 2006 has also modified like obtaining Environmental Clearance, compliance
certain industries and added the building / of the stipulated conditions etc. In this Environment
construction projects / area development projects Age, the conservative approach on the valuation of
and townships since 14.09.2006. assets from the point of the encumbrance certificate,
approval from the incompetent authority, architectural
The category 8(a) of the schedule of the EIA marvel etc are all superseded with one magical word,
Notification 2006 made it mandatory for the building that is Environmental Clearance.
and construction projects exceeding more than
96 Seminar on Valuation of Properties - 2011

Vr. JANAK S MEHTA Vr. SAPPANY PILLAI M


Vr. BAABU R S
VICE CHAIRMAN VICE CHAIRMAN
CHAIRMAN
32, Santhinath Apartment Old no. 146, New No. 299,
No.3A, Third Floor, J P Tower
Old No. 3, New No. 7 Peters Road, Thousand Lights
7/2, N H Road
Rajamannar Street, T.Nagar, Chennai 600 086
Nungambakkam, Chennai 600 034
Chennai 600 017 98410 37806
94440 71453
98403 39826 sappanypillai@yahoo.com
hitechsurveyors@gmail.com
janaks@hotmail.com
Vr. SARAVANAN T S
Vr. THIRUMAL R Vr. PRASAD K K
JOINT SECRETARY
SECRETARY JOINT SECRETARY
Flat No.5, 3rd Floor,
B-5, Vasanth Apts, No.2, 3rd Street, Paari Nagar
Betala Apartments,
19/25, South mada Street, Srinagar Ambattur, Chennai, 600 053
6th Trust Cross Street, Mandavali,
Colony, Saidapet, Chennai 600 015 98842 13145
Chennai 600 028.
93606 73666 Kkprasad6666@gmail.com,
94442 52003
thirumalvaluer@gmail.com
saravanan.tss@gmail.com
Vr. BABU J C E Vr. GOPALAKRISHNAN C
Vr. VEERARAGHAVAN K EXECUTIVE COMMITTEE MEMBER EXECUTIVE COMMITTEE MEMBER
TREASURER First Floor, Old No. 200 (New No. 90) Old No.207, New No.7, 6th Block
Flat No. D, Ground Floor, Dev Apts I Floor, Solaiappan Street Mogappair East
Door No.15, Hanumantha Road Old Washermanpet, Chennai 600 037
Royapettah, Chennai 600 014 Chennai 600 021 98416 15923
92821 38118 98401 01442 gopalkrish_22@yahoo.com
erragavancivil@yahoo.com babu_jce@hotmail.com
Vr. MOHANDAS G
Vr. MADHAVAN B
Vr. KANAGARAJ G EXECUTIVE COMMITTEE MEMBER
EXECUTIVE COMMITTEE MEMBER
EXECUTIVE COMMITTEE MEMBER 2/611, Nakkesan Salai
No.46/91, Kutchery Road
22, Aranganathan Subway Road Mogappair East
Near Kutchery Post Office
I Floor, Kaveri Nagar, Saidapet Chennai 600 037
Mylapore, Chennai 600 004
Chennai 600 015 94444 07539
94443 60610
94440 10304 g.mohandos@sify.com
madhav_bakth@yahoomail.com
govindkanakaraj@yahoo.com
Vr. RAMESH MANI S K Vr. RAVICHANDRAN K
Vr. RAMASWAMI T A H EXECUTIVE COMMITTEE MEMBER EXECUTIVE COMMITTEE MEMBER
EXECUTIVE COMMITTEE MEMBER Flat No.6, 2nd Floor B3/6, Bhuvaneswari Apartments
Flat B, Kanakadhara Dwaraka, Bakthavatchalam Nagar Bharathidasan Colony
Plot No.541, 20th Street,4th Sector Adyar, Chennai 600 020 K K Nagar, Chennai 600 078
K K Nagar, Chennai 600 078 94440 48440, 94440 84375
94440 71520 skrameshmani@gmail.com kravi_04@yahoo.com
rkrithi@md4.vsnl.net.in
Vr. GOPALAN M K Vr. ELANGO N D
Vr. KATHIRESAN S EXECUTIVE COMMITTEE MEMBER EXECUTIVE COMMITTEE MEMBER
EXECUTIVE COMMITTEE MEMBER 35/18, Flat No.1-C, Jamals Akila 9/5A, Cholan Street,
No. 8/6, Santham Colony, Neelakanta Mehta Street Anbalagan Nagar
Anna Nagar West Extn T Nagar, Chennai 600 017 Perambur, Chennai 600 011
Chennai 600 101 94448 20697 94443 24025
98411 67558 gopalan_shanthi@yahoo.com elangovaluer@gmail.com
sakthikathiresan@yahoo.com
Vr. BERNARD SHAW V
EXECUTIVE COMMITTEE MEMBER
Vr. PERUMAL A A V Plot No.101, 4th Main Road
EX OFFICIO MEMBER Krishna Nagar, Perumbakkam
New 279/19, Jawahar Colony Chennai 600 100
4th Avenue, Anna Nagar (W) 94442 27576
Chennai 600 040 bernardshaw@jbjay.com
98410 94850 / 94440 04850
perumalaav@hotmail.com

You might also like